Sunteți pe pagina 1din 153

PEV 105

VERBAL ABILITY III

1
Unit 1 - Sentence correction:
modifiers, parallelism, pronoun-antecedent error, verb-
time sequence error, comparison error
Sentence completion
types of questions - single and double blanks, eliminating
options using verbal clues
Unit 2- Voice and accent:
introduction to vowels and consonants, introduction to
accent,
stress and intonation
Unit 3- Narration:
direct and indirect speech, conversion of one speech to
another
Vocabulary enrichment:
pair-define-explain, vocabulary bingo

2
Unit 4 Essay Writing:
idea elaboration, writing an introduction, logical
sequencing of ideas, generating points or supporting
ideas and examples. concluding the essay
Reading comprehension passages:
SQW3R method, types of questions
Unit 5 -Cover letter:
key elements of cover letter, useful words and phrases
for cover letter, format of cover letter
Para jumbles:
types of para jumbles, fixed and moving para jumbles,
verbal and logical clues to solve para jumbles
Unit 6 Critical reasoning :
concepts - premise, assumption, conclusion,
strengthening statement, weakening statement,
types and patterns of questions, tips and tricks to
understand and solve critical reasoning, indicators to
identify basic concepts of critical reasoning

3
UNIT 1 PAGE NO.

SENTENCE CORRECTION 5-21

SENTENCE COMPLETION 22-31

UNIT-2

VOICE AND ACCENT 32-59

UNIT-3

DIRECT-INDIRECT SPEECH 60-69

VOCABULARY ENRICHMENT 70-79

UNIT-4

ESSAY WRITING 80-87

READING COMPREHENSION 88-113

UNIT-5

COVER LETTER 114-118

PARA JUMBLES 119-130

UNIT-6

CRITICAL REASONING 131-153

4
SENTENCE CORRECTION

SENTENCE CORRECTION
SPOTTING ERROR AND IMPROVING SENTENCES
SCHOOL OF PROFESSIONAL ENHANCEMENT

Introduction
Words, phrases and sentences are the bases of any language.

Knowing the common structures and the nuances of the language is crucial

Sentence correction is a type of question

Two variants - spotting error and improving the sentence

Involve errors based on grammar concepts of the language being tested

Eight point strategy


Is the VERB in accordance with the SUBJECT?

Is the TENSE OF THE VERB correct?

Is the SUBJECT or DOER close to the PARTICIPLE?

Are the NOUNS and VERBS in their PROPER FORMS confirming PARALLELISM?

Are the ARTICLES, DETERMINERS and ADJECTIVES correctly used?

Are the forms of PRONOUN correct and do they agree with the related nouns and other pronouns?

Are the PREPOSITIONS correctly used?

Are the SPELLINGS and PUNCTUATIONS in place?

5
ERROR OF SUBJECT VERB AGREEMENT (Is the VERB in accordance with the SUBJECT?)
Agreement in number

Sentence 1: Taking her basket she goes to the sea shore to collect shells. She is happy with what she earns.

Sentence 2: Taking their baskets the women go to the sea shore to collect shells. They are happy with what they
earn

Agreement in tense

Rewriting sentence 1 in past tense,

Taking her basket she went to the sea shore to collect shells. She was happy with what she earned.

Identify subject

Cattle is a plural noun which can neither be written as cattles nor used as singular.

eg, Cattle are grazing beside the lake.

Other such nouns, that are always used as plural without any change in them, are : People,
police, poultry, gentry, peasantry

Scissors, binoculars, spectacles, pincers, pliers, trousers, jeans, oats, outskirts, premises,
quarters, stairs, spirits, surroundings, thanks are used as plural.

Some nouns are used both ways without any change in them, such as : A sheep, many sheep, a
deer, many deer, a fish, many fish, also many fishes meaning groups or species of fishes, a
space craft, many space craft, an air craft, many air craft.

All uncountable nouns like milk, furniture, bread, food, ice, salt, butter, grass, hair, wheat,
rubbish, coffee, chalk, paper, sugar, dust, chocolate, soap are always used as singular.

All abstract nouns like applause, homework, advice, wealth, happiness, research, money,
evidence, hospitality, music, pollution, trash, waste, statistics, AIDS, disease, measles,
information, knowledge, refuge, rubbish are always used as singular.

Jury, committee, government which are used both ways. Other such nouns are : Association,
audience, board, commission, company, council, crew, department, government, jury, party,
public, staff, family

Words with ware suffix are singular : Software, hardware, silverware, chinaware, brassware,
glassware, middleware.

6
The following, although appear plural, are used as singular: Mathematics, economics, measles,
apparatus, wages, species, politics.

Some nouns never change into a plural form even if used as pluralapparatus, scenery,
jewellery, machinery, offspring, information, stationery, aircraft, dozen, hundred, furniture.

The pronouns each and one make the subject singular.

Somebody, nobody are singular but every and none can be used both as singular and plural.

The conjunction and makes the subject plural when combining two or more nouns or
pronouns.

Sometimes the subject is a non-finite verb which is always singular.

Smoking is injurious to health. Non-finite (Gerund)

To err is human, to forgive is divine. Non-finite (Infinitive)

ERROR OF TENSE (Is the TENSE OF THE VERB correct?)

Common verbs

Is, am and are are the present forms and their past forms are was and were.

Has and have are the present forms and their past form is had.

Does and do are the present forms and their past form is did.

Will, shall, can and may are the present form modals and their past forms are would, should,
could and might respectively.

Conjugation of verbs

ed in past tense

Remain same in all forms

Same past and participle form

Different forms

The second forms of the verb are the simple past forms and the third forms of the verb (or the past
participle) are the past perfect forms used with has, have or had. The third forms are also used in
the passive voice sentences.

7
Those which take ed in their past and past participle forms such as work-worked-worked,
enquire-enquired-enquired, lie- lied-lied or try-tried-tried. Some more such verbs areplay,
request, flow, die, please.
Those which remain same in all their forms such as cast-cast-cast, burst-burst-burst, cut-
cut-cut or shut-shut-shut. Some more such verbs areput, hit, spread, let, set.
Those which take same past and past participle forms such as buy-bought- bought, say-
said-said, hold-held-held or win-won-won. Some more such verbs areteach-taught-taught,
catch-caught-caught, make-made-made, bleed-bled-bled, keep-kept- kept, dig-dug-dug.
Those which are different in all the three forms such as see-saw-seen, sing-sang-sung; bear-
bore-borne, write-wrote-written. Some more such verbs areeat-ate-eaten, drink-drank-
drunk, break-broke-broken, bite-bit-bitten, take-took- taken, begin-began-begun.

Sense verb

Some verbs express not actions, or happenings but sense perceptions such as see, hear, smell, taste, feel
or possession such as own, possess, have, hold or some insight or idea such as believe, realize, obsess
etc. Such verbs are usually not used in the continuous tense. Sometimes when they are used in the
continuous tense they lose their usual meaning.

The judge is hearing the case at the moment.

ERROR OF CONDITIONALS (Where tenses are involved(Continuation of error of tense)

Conditional I is the first form (the present) and the first form of future (will) indicating some
future intentions.

Conditional II is the second form (the past) and the second form of future (would) indicating
hypothesis or supposition in present meaning.

Conditional III is had + third form (the past perfect) and would have + third form (future
perfect) some conditional regret about the past

Usage of verb have

I am having my lunch at the moment. It means I am taking.

He has some industry in this town. It means He possess or own.

We have done our work. It means work is completed and have denotes the perfect tense.

ERROR OF PARTICIPLES (Is the subject or doer close to the participle?)

Function of ing verbs

What function do the ing verbs do in the following sentences?

8
Smoking is injurious to health.

Swimming is a good exercise.

He has given up smoking.

She has started swimming.

They are taking lesson in swimming.

They function as the subject or object in the sentence. If you ask the question what is injurious or a
good exercise or what has he given up or started or is taking lesson in you will get the ing words as
answer. You should ask what questions before the verb to know the subject and after the verb to know
the object in the sentence.

In the sentences above the words smoking and swimming dont answer the question what entered or who
crossed but tell how he entered or how they crossed.

We can also say:

While he was smoking he entered the hall.

He entered the hall when he was smoking.

These are different ways of combining two sentences.

Smoking is a participle here and the main function of a participle is to combine two sentences.

The two sentences, in this case, are :

He was smoking. At that very same time he entered the hall. Or,

He entered the hall. At that very same time he was smoking.

Consider the other sentence:

While swimming they crossed the flooded river.

If we split the same, we can say :

They swam. This is how they crossed the river. Or,

They crossed they river by swimming. Or,

By swimming they crossed the river.

However, we cant say the above sentences in the following manner :

Smoking the hall was entered by him. Or, (Incorrect)

Swimming the flooded river was crossed by them. (Incorrect)

9
Why?

Because the one who smoke was he and not the hall; or, those who swam were they and not the
flooded river.

Consider the following sentences:

Writing a novel, his name became famous.

Having been a very good scientist, they made him the president.

Ask who wrote the novel his name or some person? It was some person. The sentence, therefore,
should be Writing a novel, he became famous.
In the second sentence, who was a scientistthey or the president? It was the president. Therefore, the
sentence should be Having been a very good scientist, he was made president by them.
It is clear from the above discussion that the doer of the participle verb should not be distanced from it
or interrupted by any other subject, noun or verb.

CONCEPT OF PARALLELISM (Brief introduction and usage)

Parallelism is a rule of English grammar that demands consistency in a sentence's structure.

Some examples:

activities: running, biking, and hiking

places: the store, the museum, and the restaurant

ideas: how to read, how to write, and how to learn

descriptors: quickly, quietly, and happily

In each list, whatever grammatical form is applied to one item is applied to all items.

Incorrect: He likes to swim, sail, and to dance.

Correct: He likes to swim, sail, and dance.

Correct: He likes to swim, to sail, and to dance.

Comparison words/phrases

When you see comparison words or phrases such as "more than", "less than", "although", "rather
than", check that the things being compared are grammatically parallel.

Incorrect: The professor published more papers last year than were published by all his
colleagues combined.

Correct: The professor published more papers last year than all his colleagues combined.

10
Comparison of pronouns and nouns

Sometimes, you'll come across comparisons between multiple pronouns or a noun and a pronoun. In
many cases, in order for the pronouns to be parallel, the pronouns must be identical.

Incorrect: Those who exercise in addition to maintaining a healthy diet are likely to be in better
health than the people who maintain a healthy diet but don't exercise.

Correct: Those who exercise in addition to maintaining a healthy diet are likely to be in better
health than those who maintain a healthy diet but don't exercise.

Correlative conjunction

When you use correlative pairs of conjunctions in a sentence, make sure that the words or groups of
words immediately following each conjunction are in the same form.

Incorrect: Either I will attend the show, or they will be attending.

Correct: Either I will attend the show, or they will. (or)

Either I will be attending the show, or they will be attending.

Incorrect: The captain has assigned to the case not only all his men, but also a private detective.

Correct: The captain has not only assigned all his men to the case, but also hired a private
detective.

ERROR OF NOUNS (Are the NOUNS and VERBS in their PROPER FORMS confirming PARALLELISM? )

In a sentence when a few nouns or verbs occur in a group or list, they are supposed to be in the same form
either all infinitive (to go), all participle (going) or all noun (gait).

Measures have been taken to generate energy, to conserve it and for its equitable distribution.

Correct form

Measures have been taken to generate energy, to conserve it and to distribute it equitably.

Measures have been taken for generating energy, conserving it and distributing it equitably.

Measures have been taken for the generation of energy, its conservation and equitable distribution.

Example

Incorrect: Infant is the first stage of life, boyhood the second and adolescent the third.

Correct: Infancy is the first stage, boyhood the second and adolescence is the third.
11
Incorrect: The breadth was five centimeter, the length ten and it was three centimeter high.

Correct: The breadth was five centimeter, the length ten and the height was three centimeters.

CONCEPT OF MODIFIERS (Understanding the role of adjectives and adverbs)

Modifiers are words, phrases, or clauses that restrict or provide extra information about other words, phrases,
or clauses. Adjectives (the red car, the happy child) and adverbs (he runs quickly) are modifiers.

Examples

Adjectives modify nouns or pronouns.

Adverbs modify verbs or adjectives.

Entire phrases can also be used as modifiers.

Modifying phrases function in the same way as single-word modifiers, but because they're often buried
in an already complicated sentence, they can be harder to spot than adjectives and adverbs.

Position of modifiers

Modifiers must be placed as close as possible to the word or group of words they're modifying.

Incorrect: Finally thinking clearly, the book was able to be understood by Rebecca.

Correct: Finally thinking clearly, Rebecca was able to understand the book.

Misplaced modifiers won't always occur at the beginning of sentences; any descriptive phrase or clause
is a potential misplaced modifier.

Pronouns like which, that, who, whose, whom often indicate modifying phrases.

ERROR OF ARTICLES, DETERMINERS AND ADJECTIVES

ARTICLES

Usage of the

For genetic or class references:

The dog (kind of animal) is a faithful animal.

Dogs are faithful animals.

The elephant is a sensible creature.

Elephants are sensible creatures.

Dog is a faithful creature. (Incorrect)


12
The dogs are faithful creatures.

Elephant is a sensible creature.

Usage of a/an

The countable singular noun takes a or an.

an in order to keep the two vowel sounds

The only difference between a and an is that a noun or adjective beginning with a vowel soundthe
sound of a, ai, aa, e, ei, ee, ai, ia, o, ou etcsuch as apple, iodine, end, air, heir, ex act, eye, oriental, owl
takes

DETERMINERS

Some, any = indefinite number or quantity

Much, many = big quantity (uncountable) or big number (countable)

A little = small quantityuncountable

A few = small numbercountable

More is a comparative of much and many

Less is a comparative of little;

Several

ADJECTIVES

Earlier- Later

Former Latter

Farther Further

Older elder

ORDER OF ADVERB

The order of the adverb

MPT = MannerPlaceTime

Manner = slowly, steadily

Place = here, there, up, down


13
Time = before, ago, now, today

ERROR OF PRONOUNS (Are the forms of PRONOUN correct and do they agree with the related nouns and
other pronouns? )

FORMS

Subjective forms: I, we, you, he, she, they, it.

Objective forms: me, us, you, him, her, them, it.

Possessive forms: my/of mine, our/of ours, your/of yours, his/of his, her/of hers, their/of theirs, its/of it.

Order of personal pronoun

For all good things:

You, he and I have achieved remarkable success.

You, he and I; You and I or He and I

For all bad things :

I, you and he are responsible for the failure of the project.

I, you and he; I and you or I and he

EXERCISE
Choose the correct option for the underlined phrases.

1.The three friends, Max included, was supposed to meet for dinner later that night.
A) was supposed to meet
B) was supposed to have met
C) were suppose to be meeting
D) were supposed to meet
E) they were supposed to be meeting

2.The number of students chosen for the prestigious medical internship have more than doubled in the past
fifteen years.
A) have more than doubled
B) have been more than doubling
C) has more than doubled
D) has been more than doubling
E) has doubled even more

14
3.Following intense debate, the faculty has approved the measure to increase class size by 15% over the next
four years.

A) the faculty has approved the measure to increase


B) the faculty has approved the measure and increased
C) the faculty have approved the measure to increase
D) the faculty have given their approval to the measure to increase
E) the faculty, having approved the measure to increase

4.Without proper funding and a better campaign strategy, there is no chances that our candidate will be
elected to office.

A) there is no chances that


B) there can be no chance for
C) there is no chance that
D) there are no chances for
E) there will be no chances for

5.Some members of the tribe has been protesting the recent passage of hunting laws applying to indigenous
populations.

A) members of the tribe has been protesting


B) members of the tribe have been protesting
C) tribe members has been protesting
D) tribe members will have been protesting
E) members of the tribe, having protested

6.After she attended the career fair, many more resources were at Philippas disposal, including job boards,
new contacts, and numerous books and pamphlets to help her improve her resume and cover letter.

A) many more resources were at Philippas disposal


B) at Philippas disposal were many more resources
C) there were many more resources at Philippas disposal
D) Philippa, at her disposal, had many more resources
E) Philippa had many more resources at her disposal

7.The results of the study clearly indicates a reduction in the number of useable pounds that can be salvaged
from an average ton of recyclable goods.

A) indicates a reduction
B) indicates that a reduction
C) indicating a reducing
D) indicate a reducing
E) indicate a reduction

8.The president of Costa Rica, along with two vice presidents, are elected for a four-year term by the people.

15
A) are elected for a four-year term by the people
B) are elected, by the people, for a four-year term
C) is elected for a four-year term by the people
D) are elected for four-year terms by the people
E) is elected for four-year terms by the people

9. The Egyptian Museum in Cairo, which contains 120,000 objects from prehistoric times through the Greco-
Roman period, are home to one of the most impressive collections of ancient Egyptian artifacts.

A) which contains 120,000 objects from prehistoric times through the Greco-Roman period, are home to
B) which contain 120,000 objects from prehistoric times through the Greco-Roman period, are home to
C) containing 120,000 objects from prehistoric times through the Greco-Roman period, are home to
D) which is containing 120,000 objects from prehistoric times through the Greco-Roman period, are home to
E) which contains 120,000 objects from prehistoric times through the Greco-Roman period, is home to

10.Neither of our schools students nominated for the national spelling bee were able to win the competition

A) Neither of our schools students nominated for the national spelling bee were
B) Neither of our schools students nominated for the national spelling bee was
C) Neither of the students from our school nominated for the national spelling bee were
D) Neither of the students nominated for the national spelling bee from our school were
E) Neither one of our schools students who was nominated for the national spelling bee was

11.The professor's consistent late arrival is offset somewhat by the remarkable quality of his lectures.

A) The professor's consistent late arrival


B) The consistent late arrival of the professor
C) The professor's consistently late arrival
D) Lately, the professor's arriving consistently
E) The professor's consistent late arriving

12. The concerto sounds more sophisticatedly in the 200-year-old concert hall than it did in the practice
room, which has decidedly inferior acoustics.

A) sounds more sophisticatedly


B) sound more sophisticatedly
C) sounds with greater sophistication
D) sounds more sophisticated
E) sound more sophisticated

13. Previously thought to have been extinct, a team of biologists rediscovered the New Caledonia crested
gecko in 1994.

a) a team of biologists rediscovered the New Caledonia crested gecko in 1994.


b) a team of biologists, in 1994, rediscovered the New Caledonia crested gecko.
c) in 1994 the New Caledonia crested gecko was rediscovered by a team of biologists.

16
d) in 1994 a team of biologists rediscovered the New Caledonia crested gecko.
e) the New Caledonia crested gecko was rediscovered by a team of biologists in 1994.

14. Erasmus's tomb lies inside the Basel Munster, located in Switzerland, an architectural monument which
having survived medieval earthquakes, and remains one of Switzerland's most well-known buildings to this
day.

A) Erasmus's tomb lies inside the Basel Munster, located in Switzerland, an architectural monument which
having survived medieval earthquakes, and
B) Erasmus's tomb lies inside Switzerland's Basel Munster, an architectural monument that survived medieval
earthquakes and
C) Switzerland's Basel Munster, an architectural monument that survived medieval earthquakes, houses
Erasmus's tomb,
D) The Basel Munster, in Switzerland, an architectural monument which, having survived medieval earthquakes,
is now home to the tomb of Erasmus and
E) The tomb of Erasmus, being housed inside Switzerland's Basel Munster, is an architectural monument that
survived medieval earthquakes and

15. His co-workers praised both his determination and the way he paid attention to detail.

A) and the way he paid attention to detail


B) and also praised his attention to detail
C) and his attention to detail
D) they praised the way he paid attention to detail
E) also they praised his attention to detail

16. The art studio is spacious, pleasantly cluttered, and has good lighting.

A) and has good lighting


B) and being well-lit
C) and is lit well
D) and well-lit
E) and the lighting is good

17. The school board requested that a waiver be obtained and that the residency requirements are reviewed.

A) that the residency requirements are reviewed


B) the residency requirements will be reviewed
C) the residency requirements reviewed
D) to review the residency requirements
E) a review of the residency requirements

18. Some of the many renovations set for Memorial Field in the coming years include building additional
seating, improving safety, and the construction of a new varsity athletics centre.

17
A) and the construction of a new varsity athletics center
B) and constructing a new varsity athletics center
C) and also the construction of a new varsity athletics center
D) and a new varsity athletics center
E) and a new varsity athletics center under construction

19. Richard is not only a terrific pianist, but also great at playing hockey.

A) Richard is not only a terrific pianist, but also great at playing hockey.
B) Richard not only is a terrific pianist, but is also great at playing hockey.
C) Not only great at playing hockey, Richard also is a terrific pianist.
D) Richard is not only a terrific pianist, but also a great hockey player.
E) Also great at playing hockey, Richard is a terrific pianist.

20. The philosophical doctrine of Incompatibility posits an inherent irreconcilability among the doctrine of
Determinism, which holds that each state of affairs is necessitated by the states of affairs that preceded it,
and the existence of free will.

A) among the doctrine of Determinism, which holds that each state of affairs is necessitated by the states of
affairs that preceded it, and the existence of free will
B) between the doctrine of Determinism, holding each state of affairs as necessitated by the states of affairs that
preceded it, and free will existing
C) in the doctrine of Determinism, which holds the idea that each state of affairs is necessitated by the states of
affairs preceding, and the existence of free will
D) between the doctrine of Determinism, which holds that each state of affairs is necessitated by the states of
affairs preceding it, and the existence of free will
E) among the doctrine of Determinism, which holds that each state of affairs may be necessitated by the states
of affairs preceding it, and free will existing

21. I dont remember whom has the most stock in the company.

A) whom has the most stock


B) who have the most stock
C) whom have the more stock
D) who has the most stock
E) whose the most stock

22. Anyone who chooses to be part of our coalition to defend homeless people are required to sign several
documents.
A) people are required to sign
B) people is required to sign
C) person is required to sign
D) people are required, signing
E) people is required, signing

23. Larry was entertained and enlightened by the scintillating presentation.


18
A) was entertained and enlightened
B) entertained and was enlightened
C) entertained and enlightened
D) was entertaining and enlightened
E) will entertain and enlighten

24. Valerie recalls her college years with such nostalgia that she often lost herself in reminiscence.

A) she often lost herself in reminiscence


B) she often had lost herself in reminiscence
C) she often loses herself in reminiscence
D) she often will be losing herself in reminiscence
E) she often will have lost herself in reminiscence

25. Even though he does not like crowds, John still likes New York City more than Sybil.

A) John still likes New York City more than Sybil


B) New York City is still liked more than Sybil by John
C) John is still liking New York City more than is Sybil
D) John still likes New York City more than Sybil does
E) New York City is still more liked by John than Sybil

Directions for Q26 to Q13: A part of each sentence is underlined. Choose the right option to replace the
underlined part. If no change is required, choose the option "The sentence is correct as given".

26. Unless I'm grossly mistaken, the store is usually closed on Sundays.
(a) Until I'm grossly mistaken (b) Until I'm greatly mistaken (c) Unless I'm grossly mistook
(d) Unless I'm grossly mistaked (e) The sentence is correct as given.

27. The first robot that can sense and show emotions is developed as the ultimate interactive toy.
(a) was developed (b) is now developed (c) is being developed
(d) were being developed (e) The sentence is correct as given.

28. A government employee has to work for some hours a day and can avail himself of only a fixed number of
holidays.
(a) a given number (b) a specified number (c) an unscheduled number
(d) some number (e) The sentence is correct as given.

29. Haridas Kumar Pal's battered Fiat car was alongside the garage.
(a) along side (b) alongside of (c) alongside with
(d) along side to (e) The sentence is correct as given.

30. The lion turned in Haridas Kumar Pal and knocked him senseless.
(a) turned down (b) turned off (c) turned about
(d) turned on (e) The sentence is correct as given.

31. Because of his broken hip, Haridas Kumar Pal has decided to not run for office this year.
(a) has decided to not run to office (b) has decided not to run for office
19
(c) has decided to never run for office (d) has decided to never run to office
(e) The sentence is correct as given.

32. Had I realized how close I was to failing, I would not have gone to the party.
(a) Had I been realizing how close (b) If I would have realized
(c) Had I had realized how close (d) When I realized how close
(e) The sentence is correct as given.

33. Humans' reliance on fire gives them the power to destroy or create.
(a) Humans reliance on fire (b) Humane reliance on fire (c) Human's reliance on fire
(d) Humans's reliance on fire (e) The sentence is correct as given.

34. The giant panda, resembling a bear, with his predominant white body, black ears, limbs, and eye patches,
weighs from 200300 lb.
(a) her predominant white body (b) their predominantly whitely body (c) its predominantly white body
(d) predominantly white body (e) The sentence is correct as given.

35. There are few buffaloes roaming the Great Plains today than during the last century due to the campaigns
of
white Americans against Native Americans.
(a) There are fewer buffaloes (b) There are little buffalo (c) There are less buffalo
(d) There are lesser buffaloes (e) The sentence is correct as given.

36. Both candidates oppose increased defence spending. Neither of the two candidates oppose the war in
Iraq.
(a) Neither of them oppose the war
(b) Neither of the two candidates are opposed to the war
(c) Neither of the two candidates is opposed to the war
(d) Either of the two candidates are not opposed to the war

37. Of the numerous decisions a student has to take, the question of which career path to choose is for certain
the more confusing.
(a) the question of which career path to choose is certainly the most confusing
(b) it seems certain that the more confusing is the question of which career path to choose
(c) certainly choosing their career path is the most confusing
(d) the question of which career path to choose is certainly the more confusing

38. Until 1994, multi-racial democratic elections was not held in South Africa.
(a) were not held at (b) had not been held in
(c) have not taken place (d) were not held in

Directions for Q39 and Q40: The sentences below have four words/phrases which are underlined. Identify the
incorrect usage. If there is no error, mark 'E'.

39. The union insisted on an increase in their members' starting pay, and threatened to call a strike if the
company
A B C
refused to meet their demand.

20
D
(a) A (b) B (c) C (d) D (e) E

40. Television viewers claim that the number of scenes depicting alcohol consumption have increased
dramatically
A B C
over the last decade.
D
(a) A (b) B (c) C (d) D (e) E

21
SENTENCE COMPLETION

What are Sentence Completion Questions?

They are nothing but the good old Fill in the Blanks type of questions weve been handling since
primary school!

The only difference is the test is of a slightly different kind when it comes to your

level today; and what companies/ recruiters test in you using these variety of questions.

So, WHAT DO THEY TEST?

o Vocabulary

o Not just meanings of words, but also their fine usage

o Your ability to mark logical consistency among given elements of a sentence.

o Your ability to grasp how words fit into various contexts

o Sometimes, your mere knowledge of how parts of sentence affect each other can help you determine
which option to pick;.You may not necessarily know the meanings of the options given!!

The Challenges:

Sometimes subordinate clauses, relative clauses, prepositional phrases, etc. can complex the sentences
and make comprehension and right selection difficult.

Some questions are set to judge whether you understood the tone of the writer as well as his subject.

Sometimes, technical, poetic, formal/ informal, academic, research-oriented, etc. styles of writing
would need a specific word that might be difficult to figure out of all close choices given.
22
Question types:

1. Single blank

The man is most _______, talking sweetly one day and blasting on the other.

a) Indifferent

b) Objective

c) Unpredictable

d) Ineffectual

e) Unobtrusive

Answer: c) unpredictable

2. Double blank

Although her attitude is usually ________; but she can be ________ at the name of her landlord.

a) Vicious, ignored

b) Serene, provoked

c) Energetic, disappointed

d) Meek, complimented

e) Perverse, betrayed

Answer: b) Serene, provoked

Double Blank questions are easier to solve rather than the Single Blank ones. The reason is you can eliminate
word pairs based on the inaptness of just one word in the pair!!

3. Close Text Completion Questions:

The girl was ______( solitude/ lonely/ solitary/ unique). Yet, she never felt lonely. It was her ______
(contempt/ likeness/ preference) for ________ (despondency/ dilemna) that made her a _________
(fighter/ fugitive).

Answers: solitary, contempt, despondency, fighter

So, how do we go about the questions?

1. Read through the sentence to understand.

2. You could predict a word in your own mind that could suit the blank. Do this without looking at the
options given so that you arent confused or misled by them.

23
3. Check out every option to find the closest match.

4. Remember, dont be hasty. You might sometimes miss out on the last option that might be more
suitable than another one that looks good!

Context Clues

Important words or phrases can help decipher which word would fit in the blank best. These are context clues.

The ________ is a masterpiece for it is a statement on truth as well as the manner in which step-by-step his real
life experiences helped him locate his true call. (autobiography/ prosody/ elegy)

The context clue in this case is real life experiences. Context Clues can be synonyms, antonyms or definitions.
Usually if an antonym context clue is present, the words, however, but, etc. will be also present in the
sentence.

Context Clues

Whenever a comma or colon comes after the blank, what comes after is usually the definition/ description of
the omitted word.____, the revival of old learning and culture, changed the pages of history.

Ans: Renaissance

Exercise I

Choose the correct option to fill the gaps

1.Ankit found himself _______ on the matter, as he could not decide which way to go.

a) Equanimous

b) Equivocating

c) Equilibrium

2.Although we feared it would be extreme cold at the hill station, it turned out to be __________ and
temperate that day!

a) Snowy

b) Burning

c) Hot

3.She is someone who adheres to good work culture. Her _________ has been applauded by her boss many
times.

a) Profanity

b) Professionalism

24
c) Professorship

4. The ___________ of her nature baffled me: I could not understand how and why she behaved so.

a) Serenity

b) Complexity

c) Greatness

5.The ___________ work of those craftsmen is unique. They pay such great attention to the details of the
embroidery.

a) Bland

b) Superficial

c) Intricate

6.He has a ________ personality: his moods and feelings keep him overpowered.

a) Dominating

b) Mercurial

c) Outrageous

7. _______________ is an interesting subject. Who wouldnt be curious about the development of the
human race with respect to so many respects?

a) Religion

b) History

c) Anthropology

8. She was the most ______ lady in the entire crowd. The others didnt even know how to greet like ladies.

a) Affluent

b) Sophisticated

c) Powerful

9. Some of the best players in the world _____ their skills playing street football.

a) honed

b) killed

c) Wasted

25
d) Accelerated

10. Scientists have probed into the various theories provided about __________. Even the Bible talks about
how life originated, for that matter.

a) Biology

b) Evolution

c) Palestine

d) human communication

11.For mathematics to be understood, you need to ________ some brain!

a) Imply

b) Apply

c) exploit

d) Deploy

Red herrings!

A red herring is a distracter, a misleading clue. (the expression originates from the fact that criminals rub
herring, a type of fish, on trails to distract the hunting dogs chasing them)

We are lagging behind in ____________ and everyone seems to be just copying trends set by the West.

a) Eastern

b) Spiritual

c) Nation

d) Innovation

The word, West can be a red herring here. The moment we see it, we may jump to concluding that Eastern or
Spiritual or even nation might be the answer. The idea, however, is about not copying. Hence, the answer is
its opposing innovation. None of the other options fits in grammatically even!

Exercise II

12. The path to ___________ is a path of hardship.

a) Workmanship

b) Successful

c) Illumination

26
d) Enlightenment

13.You seem to have ____________ done this. There is no way the vase could have otherwise fallen from
here.

a) Architecturally

b) Dcor-wise

c) Deliberately

14.The sign-post is so _________. If only the ones who erected it had some literary sense!

a) Shakespearian

b) Poetic

c) Ambiguous

d) Ambivalence

15. Jacksons performance was ________________. People even fainted.

a) Overcrowding

b) Overwhelming

c) Outstanding

16. Ive reprimanded him. ____________, he will not repeat his mistake.

a) Seamfully

b) Seamingly

c) Seemingly

d) Beamingly

17.I kept feeling guilty that I had disturbed him. But contrary to my fear, I found that he was __________ !

a) Unfazed

b) Innocent

c) Overwhelmed

d) Imprisoned

18. He is a silent social worker; and prefers to hide his _________ tendencies from shrewd colleagues at work.

a) Societal
27
b) Philatelist

c) Philanthropist

d) Partying

19.She likes to work in _____________ projects. We knew she was versatile.

a) Multi-utility

b) Vivacious

c) Multi-disciplinary

d) Stylish

20.Your _________ to the entire thing is wrong.

a) Method

b) Movement

c) Tacting

d) Approach

Amplification Indicators:

Words like not just,..but signify that the word pair you need to look for consists of those similar in
meaning: but the second one is supposed to be more intense in the same quality than the first one.

Madhu is not just __________, she is ____________.

a) Sweet, sour

b) Submissive, aggressive

c) Assertive, aggressive

Answer: c

Exercise III

21. Sarita has been talking to me since morning. Not only do I find her attitudinal change ______ , I even find
it somewhat _______.

a) Surprising, intriguing

b) Interesting, boring

c) Shocking, surprising

28
22.The Simla trip was fantastic. Not only did the tour bring me a ______, but it almost sent me into ________
world.

a) Freshening, mundane

b) Holiday, international

c) Enjoyment, another

d) Change, another

23.Your noise, children, is not just _______; it is an absolute ________.

a) Sweet, honey-like

b) Sweet, sugar-cube

c) Commotion, chaos

d) Commodity, chaotic

24 The teacher tried to ______ her class of their dependence on the number lines pasted to the tops of their
desks.
a. wane
b. wax
c. whet
d. wean
e. wield

25. The young girl was so full of enthusiasm and ______ that she infected the room with energy.
a. languidness
b. apathy
c. vivacity
d. blandness
e. tenacity

26.Ricky is a ______ of the local coffee shop; you can find him there just about every morning.
a. diva
b. relic
c. denizen
d. maverick
e. pariah

27. The child danced with ______ joy at hearing the news that her father had arrived home from his lengthy
business trip.
a. staid
b. unbridled
c. stealthy
d. beneficial
e. restrained

29
28. Having never left the landlocked Midwest his entire life, Albert found that swimming in the ocean was
quite a(n) ______.
a. familiarity
b. extrovert
c. instinct
d. novelty
e. tabernacle

29 The rowdy crowd at the music concert ______ Herve, and he spilled his soda on his pants.
a. jettisoned
b. harrowed
c. jostled
d. lauded
e. superceded

30. Known for his strong command of the courtroom, it was common knowledge that Judge Disantis
considered outbursts from defendants to be ______.
a. questionable
b. objectionable
c. antisocial
d. pliable
e. visionary

31. Blinded by ______, Nicholas accepted the job offer with the highest pay but the least possibility of making
him happy.
a. ennui
b. heresy
c. infamy
d. avarice
e. temperance

32.It was very unprofessional of you to ______ your assistant in front of everyone at the meeting; she
deserves more respect, and any criticism of her performance should be done in private.
a. placate
b. augment
c. usurp
d. preclude
e. deride
33.The postcard advertised a free cruise to anyone who bought a magazine subscription, but after reading the
fine print Sasha found the cruise was just a ______.
a. petition
b. gimmick
c. compromise
d. reference
e. motif

34. Many employers like to visit college campuses and ______ college seniors to work for their companies.

30
a. daunt
b. recruit
c. illuminate
d. dither
e. flout

35.As the pressures of her business became overwhelming, Charlotte chose to ______ her role as PTA
president.
a. expedite
b. transgress
c. propagate
d. relinquish
e. retaliate

36. Dont let Julies enthusiasm fool you; shes just a ______, not a professional dancer.
a. maverick
b. denizen
c. mercenary
d. maven
e. dilettante

37. Normally, Maya would not have made so many spelling mistakes in her essay; she is usually ______ about
her spelling.
a. sumptuous
b. scurrilous
c. ridiculous
d. scrupulous
e. fatuous
38. It took four men two hours to move the ______ sofa up three flights of stairs into our apartment.
a. suave
b. garrulous
c. unwieldy
d. pivotal
e. quixotic
39. In the Roman myth, Artemis made a pilgrimage to the ______, hoping to learn the answer to her dilemma.
a. denouement
b. decorum
c. oracle
d. vizier
e. pillar
40. Brian was an ______ child, he was sent to the principals office on numerous occasions for his rude
classroom behavior.
a. impeccable
b. impertinent
c. observant
d. obscure
e. adjuvant

31
VOICE & ACCENT
What is Accent?
Accent is a characteristic pronunciation especially determined by the regional of social background of the
speaker. There are different accents like American, British, and Australian etc But, our aim is to neutralize &
globalize your accent.

There exists no language the spoken form of which can be learnt directly from its written form. This is so
because no language is written exactly as it is spoken. In other words, in no language is the relationship between
the sound and the spelling perfect. And, in a language like English such relationship does not even begin to be
satisfactory. This makes the need for a special course in spoken English all the stronger. Basically, the way you
pronounce each sound of the language affects your overall accent.

For example, the sound of the letter R is pronounced differently by Spanish, German and English speakers. The
same is true for many other sounds. And so you get your accent The method to improve your English accent,
or your English word pronunciation, is to practice the proper ways to pronounce the sounds of English. Each
sound is produced in a certain way, and when you produce it right, it sounds natural.

Components of Accent
There are 3 main parts to an accent:
Intonation
Pronunciation
Rate of Speech
We would take these components separately one by one in the following units.

The Phonetic alphabet

The phonetic alphabet is an internationally recognized set of words used to clarify the letters of the
alphabet. Whenever you have to take down a message and check the spelling of the word, use the
phonetic alphabet to check that you have got it right.

32
Some important points to know about English pronunciation:

Written English consists of twenty-six letters.


Spoken English, however, consists of forty four sounds.
Because there are more sounds in spoken English than letters in written English, some English letters
represent more than one sound.
Each English vowel letter (a, e, i, o, u) can be pronounced as a short sound and one or more long sounds.
For example, the letter a can represent three common sounds.
The letter a is pronounced as a short sound in cat, pack, hand, man,
a is pronounced as a longer sound in cake, make, face (adding the letter e at the end makes alonger
a is pronounced as a longer sound in cart, park, hard, are, far.

What are VOWELS?


Speech sounds are broadly categorized as Vowels & Consonants. Vowels are those sounds during the utterance
of which the air escapes freely, without any obstruction. In written English there are 5 vowels i.e. a, e, i, o, u.

But, in spoken English there are 20 vowel sounds that are made either by vowels themselves or with the vowels
in combination called Diphthongs.

Definition: Vowel sounds are sounds that are produced by the un-interrupted flow of air.

Lip vowel sounds are produced by crafting of the lips.

Tongue vowel sounds are produced by movement of the tongue. These are complete vowel sounds.

Vowels are made in a different way than consonants. Vowels don't involve air blockage, but instead require a
more continual sound flow and sound shaping. Phoneticians describe vowel production in terms of HAR:

Height (whether the tongue is high, mid, or low in the mouth)


Advancement (how front or back the tongue is)
Rounding (whether the lips are protruded, for sounds like the "oo" of "boot.")

Another way is to consider place of articulation (where in the mouth the tongue is place) and manner of
articulation (how the sound is made) features. Here are some key manners of articulation terms for
consonants:

Cardinal vowels: Anchor points worked out by the phonetician Daniel Jones to help people classify
vowels. Cardinal vowels are special vowels, not really found in any world languages, that phoneticians
use for ear training to later detect small differences in sound quality between real-world examples.
Diphthong: Sounds that glides from one vowel to another, such as in "cow," "boy," and "fight".
These sounds are made with two tongue positions.
Monophthong: A vowel with a single sound quality, such as the middle sound in "rat" or "bit".
These sounds are made with one tongue position.
Rhotic: Also referred to as r-coloring, rhotic means there is an "r" like sound present, such as the vowel
sounds in "fear," "fare," "far," and "for".
Schwa: A mid-central, unrounded vowel [] that is poorly named because it rhymes with "duh".

33
CONSONANTS

Consonant sounds are produced by partial or full interruption of the breath flow. Consonants give clarity and
sharpness to the words . They do the work of making our speech crisp and clear, just like the function of treble
in a music system. When you whisper it is consonants that carry forward the sound and the meaning.
They also convey logic
A-Z is the Alphabet
B C D are called letters of the alphabet
The English Consonant sounds are divided into two broad buckets, which are
Voiced Consonants-Plosive
Unvoiced Consonants- Non Plosive
Consonant sounds can be produced either with or without a vibration of the vocal cords.
If the vocal cords are apart when the airstream is pushed from the lungs, the air is not
obstructed at the glottis and it passes freely into the supra-glottal cavities. The sounds
produced this way are characterized as voiceless, e.g. [p], [t] and [s].
On the other hand, if the vocal cords are in contact, the airstream forces its way through and
causes them to vibrate. Such sounds are called voiced sounds, e.g. [b], [d] and [z].

IPA (International Phonetic Association) Symbols


Consonants

VOICED UNVOICED

B P

D T

G C

V F

THE THIN

Z K

VISION S

J SH

L CH

34
W

TECHNIQUES
P, b, m Lips come together, gentle contact, release p, b with a spur of which is more of a nasal sound.
W Lips are rounded as if saying ooo.
Lips slowly open out into the sound of a, without the teeth ever biting the lower lip.
F, v- 1. Upper teeth land on lower lip. Bite your Vs
2. Release continuous flow of air.
3. V takes vibration instead of air.
S, - 1. Front teeth closed together
2. Tongue flat BEHIND both sets of teeth
3. Release continuous flow of air.
Z Same as s, but with vibration.
Th 1.Tongue between the front teeth.
2. Continuous flow of air. (Sounds like s, but with tongue between teeth. Remember, the tongue
should never touch the palate.)
the 1. Same as th, but the vibration instead of air flow.
(Like z with the tongue between the teeth)
T, d- 1. Tongue tip behind upper teeth.
2. Front teeth closed together.
3. Air released with a click as teeth separate. (Sound of a symbol, train, etc.)
Ch, j 1. Jaws shut. Release ch, j as in Hindi
Sh- 1. Same as ch, j with a continuous flow of air.
Zh 1. Same placement as sh, but with voice.
y - 1. Same as zh, but jaws slightly apart not touching
r- 1. Curl tip of tongue inward while saying aaa, producing arr as lions roar.
L 1. Keep tip of tongue behind front upper teeth, (as in Hindi I) and curl tongue up towards palate so
the front quarter of the tongue is in contact with palate, not just tip.
k- 1. Similar to Hindi k, with slight puff of air.
g- 1. Same placement as k, but add voice.
Ng- 1. Nasal sound, the g is not produced from the back of the throat separately, the sound stays in the
nasal cavity.

Consonant sound /f/ in four, laugh, phone

There are three ways of pronouncing the sound /f/.


Notice that in the words laugh and phone, the letter combinations ph and gh are pronounced as the sound
/f/.

Read the following sentence aloud. Underline the seven words that contain the consonant sound /f/.
I laughed when I found a funny pamphlet about fixing an elephants cough.

35
For example- ffather, fill ph- telephone, graph gh-enough

The consonant letter c can be pronounced two ways:


c can be pronounced as the sound /k/ as in cat, collect, act, come.
This is the most common way of pronouncing the letter c.
c can be pronounced as the sound /s/ as in city.
Consonant letter c pronounced as sound /s/ as in cents, circle, peace

Underline the nine words with the same consonant sound.(Note that in the following sentence, the letter c is
pronounced as /s/ in nine of the words; only one letter c is pronounced /k/.)
The city celebrated twice in December with an exciting circus, and excellent dancing and racing.

The letter 'c' is generally pronounced as /s/ when followed by the letters 'i', 'e' and 'y' and is pronounced
as /k/ when followed by any other letter.
Note that often, in words with double 'c', such as accident, accept, accent, the first 'c' is pronounced /k/,
the second is pronounced /s/. However, always check a dictionary if in doubt.
The consonant letter g can be pronounced two ways:
g can be pronounced as the sound /g/ as in game, go.
This is the most common way of pronouncing the letter g.
g can be pronounced as the sound /dz/ as in page, large, gem.

Read the following sentence aloud. All the words contain the letter g but are pronounced with
the sound / dz/. Underline six words with this consonant sound.

The page about the gym had a general range of stages for gentlemen.
.
Some guidelines for pronouncing g:
The letter g is always pronounced /g/ before another consonant sound (as in glass, green)or before the
letters a, o or u.
The letter g is sometimes pronounced /dz/ before e, y or i.

DIPHTHONG
(a combination of two vowel sounds)
Diphthongs begin with one vowel sound and change to another vowel sound in the same syllable.
Your mouth position changes slightly through the vowel sound.

Try saying these words: (the diphthongs are in red)


brown
fair
share
slow
guy
boy

/eI /(Long A Sound)


This diphthong is similar to the long A sound. It is pronounced as a long A sound sliding into
a long E sound.

36
Try saying these words with two vowel sounds: (the diphthong is in red)
day
say
may
away

/a/
This diphthong is pronounced as a long I sound sliding into a long E sound.

Try saying these words with two vowel sounds: (the diphthong is in red)
sky
try
fry
die

//
This diphthong is pronounced as a long O sound that quickly slides into a long E sound.

Try saying these words: (the diphthong is in red)


boy
toy
annoy
enjoy

//
This diphthong is pronounced as a long E sound sliding into an Ur sound.

Try saying these words: (the diphthong is in red)


pier
hear
steer
clear

/e/

This diphthong is pronounced as a long A sound sliding into an Ur sound.

Try saying these words: (the diphthong is in red)


bear
hair
fair
stairs

/a/
This diphthong is pronounced as a short A sound sliding into an "oo" sound.

Try saying these words: (the diphthong is in red)


brown

37
cow
how
frown

/o/
This diphthong is often just pronounced as a single long O sound. As a diphthong, it has a long O
sound sliding into an "oo" sound.

Try saying these words using the two vowel sounds: (the diphthong is in red)
yellow
coat
float
though

Tongue Twisters

(Consonant Blends)

Two consonants that form a blended sound can be hard for new English speakers. Build up their confidence with
these:

She sells seashells by the seashore


I saw a kitten eating chicken in the kitchen
I thought I thought of thinking of thanking you
Slim slam slap
A big black bug snoozed on a big black rug
He threw three free throws
Thin sticks, thick bricks
Fred fed Ted bread and Ted fed Fred bread

(L vs. R)

Some learners may have difficulty saying L and R. Asian language speakers often confuse the two letter sounds,
so these tongue twisters are perfect practice.

Red lorry, yellow lorry


Truly rural
I scream, you scream, we all scream for ice cream Rolling red wagons
Red blood, bad blood

(B vs. V)

Blue blurry vines blind


Betty loves the velvet vest best
Barber baby bubbles and a bumblebee
Burnt base, vicious vase
Vivacious Val vacuumed Violets very vivid vehicle
38
(Vowels)

The correct pronunciation of vowels is essential if you are going to be fluent in the new language. These tongue
twisters will give you plenty of practice with enunciating their vowels.

Eddie edited Earls easy music


Gooey gopher guts
Excited executioner exercising his excising powers excessively
Annie ate eight Arctic apples
An orange oval spooks the odd operative
An awful aardvark and an aching ape ate an antelope

Sounds Letters Examples Notes


[i:] e, ee be, eve, see, meet, been [i];
ea sleep, bread, deaf [e];
ie, ei meal, read, leave, sea, great, break [ei];
team, friend [e]
field, believe, receive
[i] i it, kiss, tip, pick, machine, ski,
y dinner, liter, pizza [i:]
system, busy, pity,
sunny
[e] e let, tell, press, send, meter [i:]
ea end, sea, mean [i:]
bread, dead, weather,
leather
[ei] a late, make, race, able, said, says [e];
ai, ay stable, height, eye [ai]
ei, ey aim, wait, play, say,
ea day,
eight, weight, they,
hey,
break, great, steak
[] A cat, apple, land, war, warm [o:]
travel, mad;
AmE: last, class,
dance, castle, half
[ai] i, ie ice, find, smile, tie,
y, uy lie, die,
my, style, apply, buy,
guy
[au] ou out, about, house, group, soup [u:]
ow mouse, know, own [ou]
now, brown, cow,
owl, powder
[o] O not, rock, model,

39
bottle, copy
[o:] or more, work, word [r]
o order, cord, port,
aw, au long, gone, cost,
ought coffee,
al, wa law, saw, pause,
because,
bought, thought,
caught,
hall, always, water,
war, want
[oi] oi, oy oil, voice, noise, boy,
toy
[ou] o go, note, open, old, do, move [u:]
oa, ow most, how, owl [au]
road, boat, low, own,
bowl

[yu:] u use, duty, music, cute,


ew huge, tune,
eu few, dew, mew, new,
ue, ui euphemism, feud,
neutral,
hue, cue, due, sue, suit
[u:] u rude, Lucy, June, guide, quite [ai];
o, oo do, move, room, tool, build [i]
ew crew, chew, flew,
ue, ui jewel,
ou blue, true, fruit, juice,
group, through, route;
AmE: duty, new, sue,
student
[u] oo look, book, foot,
u good,
ou put, push, pull, full,
sugar,
would, could, should
neutral sound [] u, o gun, cut, son, money, Also:
ou love, stressed, [];
a, e tough, enough, rough, unstressed, [].
o, i about, brutal, taken,
violent,
memory, reason,
family
[r] er, ur, ir serve, herb, burn, heart, hearth [a:]
or, ar hurt, girl, sir,
ear work, word, doctor,
dollar,
40
heard, earn, earnest,
earth

The Ow sound in words


out Rowdy Thousand about shout
ground Found Brown frown account
cloud Clout Trout shroud doubt

Example:
The Ow sound in Sentences
I saw the brown girl frowning.
I found my watch lying on the ground.
There is no doubt about the politicians clout.
Without the witness the case was put under the shroud.
I heard the girl shouting loudly for help.
The kids went to the town to see the fair.

A note on the pronunciation of the letter r


In some varieties of English (notably North American, Scottish and Irish), the letter 'r' is clearly pronounced
wherever it occurs in words, (eg. bird, her, purple) However, in other varieties of English, r is only pronounced
when it is followed by a vowel sound (as in around).This variation in the pronunciation of English does not
interfere with intelligibility between varieties of English, as each form of pronunciation (pronouncing or not
pronouncing r) is widely known.

The oi Sound in words


Boy Ointment Boil point
Avoid Annoyed voice royal
Coin Destroy employ foil
Toys Enjoy oyster groin

41
Examples:
The oi Sound in Sentences
Boiling the drinking water you can destroy the germs in it.
The royal family loves to eat oysters.
I told the boy to apply some ointment.
He lost his job for shouting at his employer.
The CBI foiled the terrorists ploy.
Learn to enjoy with your toys.

aye sound in words


Air Hair There care fare
Share Dare Tear where prayer

Examples:
Sentences with aye sound
She went there to get her chair.
Where did you get your hair cut from?
Dare you be unfair to your partner.
Claire barely got her share.
Dont tear the stuffed bear.

The eu sound
new Few Due illusion dilute cute
queue Confusion View diffusion circular distribution
contribute Secular Virtue future

Examples:
Sentences with eu sound
1. She bought a new dress from the market.
2. They distributed the sweets in the School.
3. The customers were standing in the queue.
4. The van used for taking the patients is called an ambulance.
5. Barbie is the cutest doll in the world.

The ea sound in words


Near Dear Earlobe fearless cheerful
Beer Clear Sheer peer interfere
Smear Tear Sneer

Examples:
Sentences with ea sound
We should not interfere in others work.
She was wearing lovely earrings in her ears.
When the team won the game everyone cheered.
We love our near and dear ones.
She had tears in her eyes when she got hurt.

42
The oa sound
casual Manual Sewer moor actual visual
poor Sure Tour cruel dual usual

Examples

Sentences with oa sound


1. The poor man did not get any shelter in the rain
2. We should always be kind to people and should not be cruel.
3. Sonia is going for a world tour with her family.
4. She was very sure of winning the tennis game.
5. Her usual routine included going for a morning walk.
6. The sewer got blocked due to the rain water.

Consonant Tongue Twisters Exercise

Twisters with "t" and "th" Twisters with "r" and "l"
Those toes aren't these toes. The rickety ladder rattled right and left before it
crashed through the glass.
These teas aren't those teas. Rotten lettuce really reeks.
This tike ties threads together twice. Loose, leafy lettuce reminds me of really pretty,
green trees.
Those threads the two tikes tied are tight. Real lemon, real lime, which would you pick every
time?
Thirty tee-shirts are tan, and thirteen tee-shirts are Ribbons rolled, ribbons loose, hair untied, what's
tie-dyed teal green. your excuse?
The teal tee-shirts total thirteen, the tan tee-shirts Tip and tap, rip and rap, lip and lap. Tip, rip, lip,
total thirty. tap, rap, lap.

Twisters with "s" and "sh" Tongue Twisters for S T C F L


She's so sick, and she's so sore, I wish her well Snickety snackety snuck
forevermore.
A shout from the south woke the sleeping sherriff. trickety trackety truck
Something sure is fishy in this city. clickety clackety cluck.
Silver slivers shimmer softly in the sunlight. Feely filly fay, freely frilly fray

Twisters with "s" and "sh"


She's so sick, and she's so sore, I wish her well forevermore.
A shout from the south woke the sleeping sherriff.
Something sure is fishy in this city.
Silver slivers shimmer softly in the sunlight.

Tongue Twisters for S T C F L


43
Snickety snackety snuck trickety trackety truck clickety clackety cluck.
Feely filly fay, freely frilly fray reapy rippy ray leapy lippy lay

Learn to articulate properly. It is simple and can BE INTERESTING. Tongue twisters are excellent for
sharpening enunciation. They make your lips, jaw, and tongue EXERCISE and increase your ability to
articulate.

Tongue Twisters For: B, P, M, and W

These consonants demand ACTIVE LIPS! Say "Boom". Explode that "b." Bring those lips down hard,
quick, and
sharply for B, P, and M. For the W, pucker the lips.

FOR B: A big black bug bit a big black bear, made a big black bear bleed.
FOR P: Peter Piper picked a peck of pickled peppers. If Peter Piper picked a peck of pickled peppers,
where is
the peck of pickled peppers Peter Piper picked.
FOR Military malarkey makes monstrous madmen into maligned martyrs.
M:
FOR If a woodchuck would chuck wood, how much wood would a woodchuck chuck, if a
W: woodchuck would?
But if a woodchuck would chuck wood, how much wood would a woodchuck chuck, if a
woodchuck could and
would chuck wood?

Th (thing) and TH(thou)


Touch the tip of the tongue to the rim of the upper teeth. The tongue tip should protrude ever so
slightly.

FOR Theophilus Thistle, the thistle sifter, sifted a sieve of unsifted thistles. If Theophilus the
TH: thistle
sifter sifted a sieve of unsifted thistles, where is the sieve of sifted thistles Theophilus the
thistle sifter
sifted?
What dost thou think of those that go thither?

Tongue Twister S,Z and WH


These sounds require extremely tenuous coordination. To pronounce "S" you raise your tongue, groove
it, and
arch it toward the hard palate. Force the breath through the narrow fissure. The same for the "Z"
except it is
vocalized. For "Sh" and "Zh" the fissure is broader. For "Wh" purse the lips as you blow the breath
through the
extended fissure.

44
FOR S: Suzy Schell sells sea shells on the seashore.
FOR Z: Moses supposes his toeses are roses, but Moses supposes amiss. For Moses knowses his
toeses
aren't roses as Moses supposes.
FOR What whim led Whitey White to whittle near a wharf where a whale might wheel and
WH: whirl?

T, D, N, L, and R
A lazy TONGUE will get you in trouble with these twisters. The first four of these consonants are made
alike.
Your tongue should snap as a whip. The tip of it should SHARPLY TOUCH the hard palatejust above the
upper teeth.On the R, the entire tongue arches itself along the roof of the mouthwithout touching it.

FOR T: Thomas Tattertoot took taut twine to tie ten twigs to two tall trees.
FOR D: Double bubble gum bubbles double. Non double bubble gum doesn't bubble double.
FOR N: A snifter of snuff is enough snuff of a sniff for the snuff-sniffer.
FOR L: Likeable Lillian loves lovely luminous aluminum linoleum.
FOR R: Around the rugged rock the ragged rascal ran.

Tongue Twister F V H K and NG

F and V
Both F and V are formed by biting lightly the lower inside lip. Say the word "fife." This is an example.
F is unvocalized and the breath is merely allowed to escape. But V is vocalized.

FOR F: I never felt felt feel Hat like that felt felt.
FOR V: Vern Verve is well versed in very wordy verb verse.

H, K, and NG

H is simply made by expiring through the mouth. K requires the back of the tongue to touch the soft
palate. The breath is then released VERY SHARPLY. G is merely the vocalized form of this sound.
When sounding Ng (sing), again arch the tongue in the same manner. But force the voice through the
nasal passage.

FOR H: Harry Hugh hid the heel behind the high hill. If Harry Hugh hid the heel behind the high hill,
where is the heel Harry Hugh hid?
FOR K: Cass Cash can catch a check cashier to cash his un-cashed check.
FOR The ringing, swinging, singing singers sang winning songs
NG:

Syllables
45
Syllables are sound units that build up the structure of every word. They are a very important part of
speech. They give a word its pronunciation. Without them speech would sound dull, boring and
meaningless.

Syllable Stress is very important to understand as using stress on the wrong syllable can effect our
pronunciation. Indians dont use the correct syllable stress.

Example: Develop Photographer Industry Technology


Pronunciation is the key to Global Comprehensibility.
A majority of the communication lapses are due to incorrect pronunciation.

Word Stress Rules


Word Primary Stress Example
One Syllable Word
Only One Primary Stress CAR, CARE, PEN

Two Syllable PRE-sent, EX-port, IN-dia,


HAP-
Noun/Adjective/Adverb Mostly on the First Syllable
py

Two Syllable Verb/Preposition


under-STAND, ex-PECT, a-
Mostly on the Second Syllable WAY

Words With More Than Two Syllables


Word Primary Stress Example
Compound Noun
Mostly on the First Part BLACK-bird, GREEN-house

Compound Adjective
bad-TEMPERED, old-
Mostly on the Second Part
FASHIONED

Compound Verb
Mostly on the Second Part get-OUT, shut-UP, come-OVER
46
Ending in -ic, -sion, -tion, - teleVIsion, eduCAtion,
ible, Mostly on the Second Last
muSIcian, GLOrious, ENGlish,
-ial, -able, -ian, -ious, -ia, -ish Syllable
DURable

Ending in -cy, -ty, -phy, -gy, -


Mostly on the Third Last biOlogy, phoTOgraphy,
al, -ise, -ate
Syllable CRItical, CALculate

Ending in -ade, -ee, -eer, -


ese,
lemoNADE, engiNEER,
-ette, -que, -oon casSETTE,

NOUN VERB SENTENCES


NOUN VERB SENTENCES
an accent to accent His Scottish accent, accented his
English.
a conflict to conflict The conflict was caused due to
the conflicting interests between
the two parties.
a discharge to discharge The discharge was green when it
was discharged from his body
during surgery
an envelope to envelope She took the envelope from his
hands and enveloped him in a
bear hug
an incline to incline I am inclined to believe that she
pushed him off the incline!
an object to object The object of our discussion
objects to being manhandled.
a present to present They presented Michael with his
birthday present at midnight
when he was present.
a recall to recall Do you recall the email that he
recalled?
a subject to subject The subject was subjected to a
harrowing experience while
travelling to work.
47
Tip- How do we stress?
3 simple rules to remember
We pause for a brief moment before saying the word
We raise the volume awee bit
We stretch the whole vowel sound

Examples

`Subject / sub`ject
I always found history a boring subject.
The prisoners were subjected to extreme torture by the army.
`
Object / Ob`ject
The crystal vase was an object of desire.
I believe the judge will object to the defendants plea.

`Record / Re`cord
He found his favorite record in the store
Jim wanted to record the conversation for reference.

First Sound Stress


`Accent `Absent
`Addict `Address
`Conduct `Contract
`Contrast `City
`Customer `Object
`Perfect `Family
`Interest `Industry
`Number `Relative

MID-Sound Stress
De`mocracy Po`litical mechanical hy`pocracy cor`rective me`dicinal a`merica in`structions

LAST Sound Stress


di`plomacy ph`tographer fa`miliar mo`notony ex`posure dep`artment (mnt) im`portant pro`cess

What is Sentence Stress?


Words in a sentence are not all given the same salience in oral English. Some words are picked out and
are stressed in contrast to others. The one that is the most stressed is said to receive the sentence
stress. This usually implies differences in meaning. In the following sentences, the sentence stress is
indicated in bold case.

Consider the difference in meaning for each of these scenarios.


Sentence Stress Illustrated:

48
Sentences Meaning
I dont think she would write it. I dont think that, but someone else does.

I DONT think she will listen to him. It is not true that I think that.

I dont think she will listen to HIM. I think that she will listen to someone else than
him.

I dont think she WILL listen to him. I think that she will not be willing or agreeable to
listening to him.

I dont think she will LISTEN to him. Instead of listening, she might talk to him.

Word Endings:
Lets hear the following words ending in an ed ending and see which sound do we hear.

Is it a t, d or id sound.
id D T
Dated Blamed Asked
Married Cared Revised
Wasted Mastered Reached
Carried Tuned Placed
Fainted Argued Watched
Painted Played Licked
Invited Assumed Tricked

Word endings with able sound as ible and ij


ible Ij
Reachable Storage
Measurable baggage
Fashionable Coverage
Comfortable Luggage
Accountable Courage
Adorable Advantage

What is Neutral Accent?


An accent is a way of pronouncing a language. It is therefore impossible to speak without an accent.
Your accent results from how, where, and when you learned the language you are speaking and it gives
impressions about you to other people. People do not have a single fixed accent which is determined by
their experiences. We can control the way we speak, and do, both consciously and unconsciously. Most
people vary their accent depending on who they are speaking with. We change our accents, often
without noticing, as we have new life experiences.

49
VOICE MODULATION (INTONATION)
Intonation
Intonation is the music of the language. In English, we use tone to signal emotion, questioning, and parts
of the sentence among many other things. It's important to recognize the meaning behind the tones
used in everyday speech, and to be able to use them so that there are no misunderstandings between
the speaker and the listener.
It is generally true that mistakes in pronunciation of sounds can be overlooked, but mistakes in
intonation make a lasting impression.

Try this short exercise:


Say this sentence aloud and count how many seconds it takes.

The beautiful Mountain appeared transfixed in the distance.


(Time required? Probably about 5 seconds. Now, try speaking this sentence aloud)

He can come on Sundays as long as he doesn't have to do any homework in the evening.
(Time required? Probably about 5 seconds)

Wait a minute the first sentence is much shorter than the second sentence!
The beautiful Mountain appeared transfixed in the distance
He can come on Sundays as long as he doesn't have to do any homework in the evening

You are only partially right!

This simple exercise makes a very important point about how we speak and use English. Namely, English
is considered a stressed language while many other languages are considered syllabic. What does that
mean? It means that, in English, we give stress to certain words while other words are quickly spoken
(some students say eaten!). In other languages, such as French or Italian, each syllable receives equal
importance (there is stress, but each syllable has its own length).

In other words we can say that, The intonation of a language refers to the patterns of pitch variation,
or the tones, its uses in its utterance.

Basically, stress words are considered CONTENT WORDS such as


Nouns e.g. kitchen, Peter
(most) principle verbs e.g. visit, construct
Adjectives e.g. beautiful, interesting
Adverbs e.g. often, carefully

Non-stressed words are considered FUNCTION WORDS such as


Determiners e.g. the, a, some, a few
Auxiliary verbs e.g. don't, am, can, were
Prepositions e.g. before, next to, opposite
Conjunctions e.g. but, while, as
Pronouns e.g. they, she, us

Intonation in Questions
50
Statements usually end with a pitch fall. Questions that can be answered with a yes or no are
exceptions to this rule and usually end with a rise in the pitch.
For example, see the following questions. Do you notice the rise at the end? (Underlined words are the
focus words or words which are stressed)

Are you married?


Do you want to go out?
Do you need a raise?

Information Questions

Now see these questions that cannot be answered with a yes or a no, we can call them information
questions.

How many children do you have?


How long have you worked at this company?
Where is the boss today?

These questions do not have a rise in the intonation at the end. Instead, the rise is on the more
important word, the focus word in the sentence. This is also the word that gets the most stress. So the
intonation reinforces the stress pattern. Then the intonation drops at the end of the thought group, as it
does in statements.

Intonation with Choices


When the speaker wants to offer you a choice, the pitch or intonation will go UP on the first (or all the
choices before the last one) and down for the final option to indicate the end of choices.

For example, see the following sentences:


Do you want chocolate or vanilla?
Do you want chocolate, vanilla or strawberry? Do you want red, brown or black?

Intonation/ Stress Shifts


Sometimes the intonation and stress change to show what the speaker feels is important. For Example,
if youve been leaving office earlier each day this week and your boss asks the second question below,
maybe she\he will stress TODAY and rise in intonation will also be on today. Listen as your instructor
reads these sentences:
Youve left early every day this week.
When are you leaving today?

Stress moves from one word to the other. Notice how the entire meaning of the question changes as
stress moves from one word to the other.
1. When are you leaving today?
2. When are you leaving today?
3. When are you leaving today?
4. When are you leaving today?

51
RATE OF SPEECH
The class is given the following passage to read in groups and the partner notes down the time taken to
read the 100-word passage.

Dwarfs
On dark nights, I sometimes dream of little dwarfs who like to ride through the woods and along country
roads on tiny donkeys. On cold stabbing nights they try to invert riddles to the sound of wind. They tend
to find clear skies disappointing and much prefers dark stormy nights. One dark dismal night in winter, I
observed a dozen of these little men hunting for dandelions in the woods. It was raining hard but in the
distance I could see a little figure riding away from me. I followed close behind until the dwarf reached a
door through which he disappeared. I knocked hard on the door, hoping to catch a glimpse of where he
lived. All of a sudden I heard a dreadful drumming sound behind me. A drunken dwarf dressed in a
diamond encrusted cloak was riding past me on a dappled donkey! I tried to address him, but with
downcast eyes, he trundled down the road into the distance. As the day dawned, I woke from my
dream.

Common Errors in Speech


Where are you speaking from?
Return back
Four days back
Double oo, 99 (Say it individually)
Have a seat, have a book
Home # not residence
Give me/ give you a ring (call)
Choose one (not pick)
How did you find the place/person?
Going to go
Repeat again
Why not
Give an exam (take an exam)
Speak loudly
Back-side
Cook / laugh myself
Sports not games
I was finding / searching him (looking for)
Tomorrow we would be doing this. (Will)
Today morning / Yesterday evening. (This / last)
In the night (at)
More better, more louder, more happier
Very much less

52
Exercise

1. Insert the correct word(s) in the sentences below.


1. If we go __________ the beach, would you like to come __________? (to/ too/ two)
2. I am going __________ you like it or not. (whether/ weather)
3. I shall put __________ parcels over __________. (there / their/ theyre)
4. I like to come __________ because I always __________ the latest gossip. (hear/here)
5. There is __________ paper left in the printer, or did you __________ that already?(no/ know)
6. __________ you like to come to the party with me? (would/ wood)
7. You need to go __________ the door at the end of the corridor to get to the exit.(through/ threw)
8. I must __________ a letter to the bank. (write / right)

9. I need to see if he has cashed the __________. (check/ cheque)

10. He __________ his motorbike along the __________. (road/ rode/ rowed)

2.Identify and underline the - Sound AE


1. He led the lad home.
2. Anna sat down after losing the first set.
3. The brave men manned the hostile border.
4. The camel went to the chemist and said he was sad.
5. Her head had hurt all night.
6. Then he asked if Kevin was taller than Keith.
7. Lets bet on Sachins bat.
8. The fat bo went to the fete.
9. He got a message from her in the massage parlor.
10. He is eating a red raddish.
11. His dad is unfortunately dead.
12. He went to bed with a bad cold.
13. He was merry cos he didnt marry.
14. When will the Van start working?

15. Please hand over the hen.

3. Fill in the blanks using aw sound.


Niagara ____ls is the highest falls in the world.
The children are playing with the basket ______l.
He got a ___________ship from his school.
_________ys walk to your left on the road.
Roses have _______ns.

4.Unscramble the jumbled words


Liob
Jyeon
Snioe
Oyj
Inoc

53
Sybo
Ayorl

5.Choose the Correct Response.


I went to the barber to get a (hair/ hare)cut.
Sheena is a (fare/ fair) girl.
On Ridhimas birthday her father gifted her a teddy(bare/ bear).
Please dont (stare/ stair) at the stranger.
I bought a (pear/ pair) of scissors for the craft classes.
I will (ware/ wear) a red dress for the party.

6.Match the columns

A B
The invisible gaseous substance surrounding the earth scare
Have the courage to do something player
Fearful or anxious air
The money a passenger on public transport has to pay fare
A person taking part in a sport or game. dare
A large, heavy mammal with thick fur and a very short tail. care
Feel concern or interest. bear

7.Identify the eu sound words

dew blue Moo


collision confusion combination
distribution distortion disorganize
social soothing Secular
fruit fusion Fiction

8.Match the columns.


A B
State of happiness fear
Laughing at someone Interfere
Being afraid of something Cheerful
Meddling with others affairs Sneer

9.Underline the ea sound words in the following passage.


I got up on Monday and saw a clear and blue sky out from my window. So I decided to go to the market
near
my house. I went to a jewelers shop to buy earrings for a friend. There I saw a fearful site. A man was
holding

54
a knife and asking for money. I didn't go inside and peered through the glass. Then I called the police
quickly.
When the police came they went in with guns and arrested the thief. Everyone was relieved and
cheered for the
police. Then I went inside and bought lovely earrings for my dear friend.

10. Match the column with aye/oi/ow/eu/ea and oa sound words


A B
The man with no money and house Ground
Something which is not far away Confusion
The children play with it Near
Giving your things to others to use Toys
A big field where cricket is played Share
A state of being puzzled Poor
Colour of a cocoa chocolate Employ
A big furry animal in the zoo Brown
Looking at something closely Bear
Something which is connected Illusion
Hiring someone in your organization Peer
Something which is not real Joint

11.Silent consonants
In English, many written words contain consonant letters that are not pronounced. These letters are
referred to as silent letters.

Match each word with the appropriate meaning below.


Rhyme often listen island calm column talk foreign hour
exhausted bark

For example frequently often


1.sixty minutes __________________________
2land surrounded by water __________________________
3 peaceful and quiet __________________________
4 words containing the same sounds __________________________
5 to hear and give attention when someone speaks __________________________
6 to speak __________________________
7 to be very tired and without energy __________________________
8 from another country or another place __________________________
9 a strong, tall piece of stone or wood __________________________
10 used to support a building

12.More silent consonants


Read aloud the words in the box below and match each word with the appropriate meaning below.
plumber doubt debts wrap knock limbs wrists knife

55
For example: to cover completely with paper or other material wrap
1a person who repairs water pipes __________________________
2 to be uncertain __________________________
3 where your hands join your arms __________________________
4 to hit or strike something on another thing __________________________
5 money owed to other people __________________________
6 a sharp tool used for cutting __________________________
7 arms, legs or branches of trees __________________________

13Complete the information about silent consonants and write some examples.
Write more examples of each pattern on the lines below.
1Silent b can follow m lamb _________ _________
2At the beginning of words, silent k can go before _____ _________ ________ _________
3At the beginning of words, silent w can go before _____ _________ _________ _________

14 Read the following articles following correct pronunciation of words with proper stress and
intonation pattern.

Article 1

Woman power to the fore

There is a paradigm shift visible in the manner in which women are being presented in some recent
advertisement campaigns

In the course of history, there comes a time when humanity is called to shift to a new level of
consciousness, to reach a higher moral ground- a time when we have to shed our fear and give hope to
each other. That time is now.

- Wangari Maathai

One of the hallmarks of modern commercial advertising is the effort to re-create traditional ethical
values and myths. Advertisement campaigns seek to create a feeling for the glorious days of yore and re-
create the nostalgic past in the present. Divine figures as well as traditional art forms and festive
cultures are often parodied by commercial advertisements. It creates in the mind of the people a feeling
of nostalgia and a sense of pride in tradition and gives them the illusion of a chance to re-create it with
the help of the product propagated by the advertisement.

Traditional values such as family, friendship, respect for elders and so on are promoted through ads to
create a positive setting for the commercial product. In addition to the feel-good sensation, commercial
ads in recent times have tended to promote new and sometimes revolutionary values. Especially when
considering the mobile network ads in the last two years, you cannot but notice novel concepts of an
ideal society propagated by them. Post-modern ideas such as New Feminism, gender equality, rights of
the marginalised people, ecological concerns and so on are artistically promoted, coupled with
traditional values. The most striking among these are undoubtedly the concerns in New Feminism. This
56
is an attempt to trace and appreciate the strikingly new female-centric social values pictured in the
popular mobile network ads in recent months.

Article 2

Chinese jets intercept U.S. aircraft

A U.S. surveillance plane was forced to take evasive action after two Chinese fighters intercepted it over
the East China Sea, the Pentagon said on Monday.

The incident occurred on Sunday when the two Chinese J-10 warplanes intercepted a U.S. Navy EP-3
reconnaissance plane in international air space west of the Korean Peninsula, Pentagon spokesman Navy
Captain Jeff Davis said.

One of the Chinese jets came underneath the U.S. plane at high speed, then slowed and pulled up in
front of it, he said. It was forced to take evasive action to prevent collision, he said, adding that that it
was uncharacteristic of the Chinese military. There are intercepts that occur in international air space
regularly, and the vast majority are conducted in a safe manner.

Article 3

The Reading Habits Of Highly Successful People Some of the world's highest achievers have one thing
in common: it isnt a high IQ, nor is it an incredible lucky streak, but their appreciation for reading.
Books were their most profitable investment.
Two teenage boys found employment at a grocery store in Omaha, Nebraska. The older boy, from a
poor family devastated by the Great Depression, bred and sold hamsters for spare change. The younger
boy, grandson of the store owner, had been delaying college and working odd jobs, like selling chewing
gum and coke bottles door to door. Back then, each boy made about $2 a day. Just a few decades later,
theyd be raking in $20 billion in profit per year with their conglomerate, Berkshire Hathaway. Who were
these boys? None other than Charlie Munger and Warren Buffett. How did they become the most
successful investors America has ever seen?

The billionaire book club Buffett and Munger are not the only ones who credit their success to reading.
Tech entrepreneur Elon Musk reportedly learned how to build rockets by reading books. Musk was
bullied a lot as a child in South Africa. He found comfort in fantasy and science-fiction books, which
inspired him to leave a legacy in the world.

Article 4

Network challenges: On Reliance JioPhone

As the telecom sector faces a price disruption, the regulator needs to bridge the data divide

Theres a fresh twist in the tale for Indias telecom sector, which is a success story with around a billion
connections issued so far and about 350 million subscribers estimated to have smartphones. About ten
months after beginning commercial operations and acquiring 125 million customers with attractive data

57
and voice service offerings,Reliance Jio has announced a planthat could disrupt the telecom landscape
by challenging existing price points. For a refundable security deposit of 1,500 and a tenth of that as
monthly charges, it plans to give away free feature phones that will support 4G services and can be
returned after three years. Incumbent service providers, now saddled with high debt that could turn
into non-performing assets for lenders, would naturally be wary of this move although analysts
expect overall industry revenue to rebound from its declining trajectory. Similarly, Jios proposition of
connecting the phone to the television has affected the stock prices of direct-to-home service providers,
though it may only offer three-four hours of such viewing per day. While rivals may need to rejig their
service offerings to keep up, the disruptive potential of this development could be far more profound,
especially in terms of bridging the countrys digital divide.

Indias Internet adoption rate remains among the lowest in the Asia-Pacific region with 422 million
subscribers. A large chunk of them access the Net through smartphones. Although wireless data usage
has shot up dramatically over the past year, 500-550 million Indians use feature phones that offer no
data services. A Kleiner Perkins Internet Trends Report for 2017 notes that even though smartphone and
data costs are declining in India, they are still too high for most. Cheaper phones as well as data are
essential to bring online the next 100-200 million people. While market forces will come into play here,
policymakers need to step up their game too. The government must rationalise the multiple statutory
levies on telecom service providers, which have been flagged as major stress points for the sector.

Article 5

First Made-in-India Jeep vehicle unveiled

The Jeep Compass will be in full production in July and vehicles are expected to hit the market during
the third quarter of the year

Automaker Fiat Chryslers India arm on Thursday rolled out the first locally manufactured Jeep Compass
SUV from its Ranjangaon facility in Pune. With this, India has joined China, Mexico and Brazil as a
manufacturing and export hub for the Jeep Compass. FCA has invested $280 million in the plant to
support local production of Jeep. The company said the export of the vehicle would commence in the
last quarter of calendar year 2017. The establishment of manufacturing operations and start of
production of the Jeep Compass is an important milestone for our journey in India, Alcala said. The
Compass will be the first Made-in-India Jeep vehicle, and we are appreciative of the governments
efforts to make India a desirable manufacturing destination for multinational organisations like ours, he
said. With the start of production and an investment of $280 million in Jeep Compass localisation, we
are reaffirming our commitment to Indian consumers and our long-term presence in the country, Alcala
added. The Jeep Compass will be in full production in July and vehicles are expected to hit the market
during the third quarter of the year in three variants Sport, Longitude and Limited, the company said.

By 2017-end, the Jeep brand portfolio in India will consist of three new models Jeep Wrangler and
Jeep Grand Cherooke, besides Jeep Compass, it said. FCA India Automobiles Pvt Ltd is a wholly-owned
subsidiary of Italian-American auto maker Fiat Chrysler Automobiles (FCA). The Compass is the first
model from the Jeep range to be produced in India. Fiat Chrysler had launched the Jeep brand in India in
August last year with its completely built units (CBUs). The Ranjangaom manufacturing facility, a 50:50

58
joint venture between FCA and Tata Motors, has a capacity to produce 1.6 lakh units and 3.5 lakh
engines.

15 Share your ideas on the following topics keeping in mind correct pronunciation of words
with proper stress and intonation pattern.

1.How do you pay for online shoppingUsing Online banking or Cash on Delivery and why?

2.What are the latest changes brought about by technology in our present lives? Do you think there may
be certain other changes or inventions as well?

3.If you want to join a company, which benefits do you think would be most attractive to you and why?

4.Talk about the role of taking celebrities in promotional activities? Is a big celebrity in the
advertisement a guarantee of good quality?

5.How GPS is useful for the society?

6.What are some of the common problems that may occur in a mobile phone?

7.Is man becoming a slave to technology?

8.Do you think the products that we buy reflect the sort of people we want to be? Give reasons.

9.What is the best thing about the Internet?

10.Tell something about your favourite websites.

59
DIRECT AND INDIRECT SPEECH
Introduction
There are two ways to convey a message of a person, or the words spoken by a person to other person.

1. Direct speech 2. Indirect speech

Suppose your friend whose name is John tells you in school, I will give you a pen. You come to home
and you want to tell your brother what your friend told you. There are two ways to tell him.
Direct speech: John said, I will give you a pen.
Indirect Speech: John said that he would give me a pen.

In direct speech the original words of person are narrated (no change is made) and are enclosed in
quotation mark.

While in indirect speech some changes are made in original words of the person because these words
have been uttered in past so the tense will change accordingly and pronoun may also be changed
accordingly.

In indirect speech the statement of the person is not enclosed in quotation marks, the word that may
be used before the statement to show that it is indirect speech.

Indirect speech is also called reported speech because reported speech refers to the second part of
indirect speech in which something has been told by a person.

Reporting verb: The verb first part of sentence (i.e. he said, she said, he says, they said, she says,) before
the statement of a person in sentence is called reporting verb.

Examples: In all of the following example the reporting verb is said.


He said, I work in a factory (Direct speech)
He said that he worked in a factory. (Indirect speech)
They said, We are going to cinema (Direct speech)
They said that they were going to cinema. (Indirect speech)

Reported Speech. The second part of indirect speech in which something has been told by a person
(which is enclosed in quotation marks in direct speech) is called reported speech. For example, a

60
sentence of indirect speech is, He said that he worked in a factory. In this sentence the second part he
worked in a factory is called reported speech and that is why the indirect speech as a whole can also be
called reported speech.

Fundamental rules for indirect speech

1. Reported speech is not enclosed in quotation marks.


2. Use of word that: The word that is used as a conjunction between the reporting verb and
reported speech.
3. Change in pronoun: The pronoun (subject) of the reported speech is changed according to the
pronoun of reporting verb or object (person) of reporting verb (first part of sentence). Sometimes the
pronoun may not change.

In following example the pronoun of reported speech is I which will be changed in indirect speech
into the pronoun (Subject) of reporting verb that is he.

Example

Direct speech Indirect Speech

He said, I am happy. He said that he was happy.

I said to him, You are intelligent I said him that he was intelligent.

(You changed to he the person of object of reporting verb)

1. Change in time: Time is changed according to certain rules like now to then, today to that day,
tomorrow to next day and yesterday to previous day. Examples:
Direct speech: He said, I am happy today
Indirect Speech: He said that he was happy that day.
2. Change in the tense of reported speech: If the first part of sentence (reporting verb part) belongs to
past tense the tense of reported speech will change. If the first part of sentence (reporting verb part)
belongs to present or future tense, the tense of reported speech will not change. Ex:
Direct speech Indirect Speech

He said, I am happy. He said that he was happy. (Tense of reported speech changed)

He says, I am happy He said that he is happy. (Tense of reported speech didnt change)

Table for change in tense of reported speech for all TENSES.

61
DIRECT SPEECH INDIRECT SPEECH
PRESENT TENSE
PRESENT SIMPLE changes into PAST SIMPLE
He said, I write a letter He said that he wrote a letter.
She said, He goes to school daily He said that she went to school daily.

PRESENT CONTINUOUS changes into PAST CONTINUOUS

He said, He is listening to the music He said that he was listening to the music.
She said, I am washing my clothes She said that she was washing her clothes.

PRESENT PERFECT changes into PAST PERFECT

She said, He has finished his work She said that he had finished his work.
He said, I have started a job He said that he had started a job.

PRESENT PERFECT CONTINUOUS changes into PAST PERFECT CONTINUOUS

He said, I have been studying since 3 Oclock He said that he had been studying since 3 Oclock.
She said, It has been raining for three days. She said that it had been raining for three days.

PAST TENSE
PAST SIMPLE changes into PAST PERFECT
He said to me, You answered correctly He said to me that I had answered correctly.
John said, They went to cinema John said that they had gone to cinema.
PAST CONTINUOUS changes into PAST PERFECT CONTINUOUS
They said, We were enjoying the weather They said that they had been enjoying.
He said to me, I was waiting for you He said to me that he had been waiting for me.

PAST PERFECT changes into PAST PERFECT (tense does not change)

She said, She had visited a doctor She said that she had visited a doctor.
He said, I had started a business He said that he had started a business.

FUTURE TENSE

FUTURE SIMPLE TENSE WILL changes into WOULD

He said, I will study the book He said that he would study the book.
She said, I will buy a computer She said that she would buy a computer.

62
FUTURE CONTINUOUS TENSE WILL BE changes into WOULD BE

I said to him, I will be waiting for him I said to him that I would be waiting for him.
She said, I will be shifting to new home She said that she would be shifting to a new home.

FUTURE PERFECT TENSE WILL HAVE changes into WOULD HAVE

He said, I will have finished the work He said that he would have finished the work.
She said, They will have passed the examination She said that they would have passed the examination.

*Note: The tense of reported speech may not change if reported speech is a universal truth though its reporting verb belongs to
past tense.

Examples:
Direct speech Indirect Speech
He said, Mathematics is a science He said that mathematics is a science.
He said, The sun rises in east He said that the Sun rises in east.

Indirect speech for Interrogative (question) sentence

For changing interrogative (question) sentence into indirect speech we have to observe the nature of
question and then change it into indirect speech according to its rules for indirect speech. A question
can be of two types. One type is which can be answered in only YES or NO and other type which needs a
little bit explanation for its answer and cannot be answered in only YES or NO.
Examples
Do you like music? (It can be answered in YES or NO)

How are you? (It cannot be answered in YES or NO but it needs a little bit explanation i.e., I am fine.)

Questions which can be answered in YES/NO.

To change questions (which can be answered in yes or no) into indirect speech, word if or whether is
used before the question in indirect speech.

Rules for change in tense of question sentences are same as for change in normal tenses in indirect
speech but sentence will not start with the auxiliary verb of the tense.

The word that is not used between reporting verb and reported speech as a conjunction in indirect
speech for question sentence. Question mark is not used in indirect speech.
Examples
Direct speech: He said to me, Do you like music?

63
Indirect Speech: He asked me if I liked music. (Not, did I like music)
Or Indirect Speech: He asked me whether I liked music.
Direct speech: She said, Will he participate in the quiz competition?
Indirect Speech: She asked me if he would participate in quiz competition.
Direct speech: I said to him, Are you feeling well?
Indirect Speech: I asked him if he was feeling well.

Question which cannot be answered in YES/NO.

To change such questions into indirect speech, the words if or whether is not used. The tense of the
question is changed according to the rules for change in normal tenses in indirect speech but sentence
will not start with the auxiliary verb of the tense. The word, that, is not used between reporting verb
and reported speech as a conjunction in indirect speech for question sentence. Question mark is not
used in indirect speech.
Examples:
Direct speech: He said to me, How are you?
Indirect speech: He asked me how I was. (Not, how was I)
Direct speech: Teacher said to him, What is your name?
Indirect speech: Teacher asked him what his name was.
Direct speech: She said to him, Why did you come late?
Indirect speech: She asked him why he had come late.

Indirect speech for exclamatory and imperative sentences

Indirect speech of imperative sentence:

A sentence which expresses command, request, advice or suggestion is called imperative sentence.
For example, Open the door. Please help me. Learn your lesson.

To change such sentences into indirect speech, the word ordered or requested or advised or
suggested or forbade or not to do is added to reporting verb depending upon nature of imperative
sentence in reported speech.

Examples

Direct speech Indirect Speech


He said to me, Please help me He requested me to help him.
She said to him, You should work hard for exam She suggested him to work hard for exam.
64
He said, Open the door He ordered to open the door.
They said to him, Do not tell a lie They said to him not to tell a lie.
The teacher said to student, Do not waste time The teacher advised the students not to
waste time.

Indirect speech of exclamatory sentences

Sentence which expresses state of joy or sorrow or wonder is called exclamatory sentence.
Ex: Hurrah! We won the match. Alas! I failed the test. Wow! What a nice shirt it is.

To change such sentences, the words exclaimed with joy or exclaimed with sorrow or exclaimed
with wonder is added in the reporting verb depending upon the nature of exclamatory sentence in
indirect speech.

Examples.

Direct speech Indirect Speech


He said, Hurrah! I won a prize He exclaimed with joy that he had won a prize.
She said, Alas! I failed in exam She exclaimed with sorrow that she had failed in the exam.
John said, Wow! What a nice shirt it is John exclaimed with wonder that it was a nice shirt.
She said, Hurrah! I am selected for the job She exclaimed with joy that she was selected for the job.

Indirect speech for sentence having MODALS, can, may, must,

Present modals are changed to past modals


Direct Speech Indirect Speech
CAN changes into COULD
He said, I can drive a car He said that he could drive a car.
She said, He can play a violin. She said that he could play a violin.

MAY changes into MIGHT


He said, I may buy a computer He said that he might buy a computer.
She said, He may visit a doctor. She said that he might visit a doctor.
MUST changes into HAD TO
He said, I must work hard He said that he had to work hard.
She said, They must carry on their work She said that they had to carry on their work.

Exercise 1 Change the following direct speech into indirect speech.

65
1. He said, I have got a toothache.

2. Manu said, I am very busy now.

3. Hurry up, she said to us.

4. Give me a cup of water, he told her.

5. She said, I am going to college.

6. She said to me, Thank you

7. Raju said, Gautam must go tomorrow.

8. Geetha says, My father is an Engineer.

9. He said, I have passed the physical test.

10. She said to me, You are my only friend.

11. I love reading books, he said to me.

12. Where are you going? James asked Mary.

Exercise 2 Change the following direct speech into indirect speech.

1.Why do you want to work here?

2. What kind of experience do you have managing a team?

3. How many years did you work as a freelance reporter?

4. What was the most challenging thing about your last position?

5. What kinds of editing software have you used?

6. How much experience do you have as a beat reporter?

7. Why do you want to leave your current position?

8. What were your responsibilities at CKRD News?

9. Are you willing to travel?

10. Do you have any questions for me?

66
Exercise 3 Choose the correct option

1. He said to me, "I am ready".

a.He told to me that he is ready.

b.He told me that he was ready.

c.He told me that I am ready.

d.He told me that I will ready.

2. Sonia said, "You help my sister".

a.Sonia said that I helps her sister.

b.Sonia said to me that I helped her sister.

c.Sonia said that I helped her sister.

d.Sonia says that I helped her sister.

3.They said, "We can not live without oxygen".

a.They said that we can not live without oxygen.

b.They said that they can not live without oxygen.

c.They said that they will not live without oxygen.

d.They said that they could not live without oxygen.

4. Rahul said to me, "We are mortal".

a.Rahul told me that we are mortal.

b.Rahul says to me that we are mortal.

c.Rahul said to me that we all are mortal.

d.Rahul said to me that he and I are mortal.

5. The Indian express says, "We shall issue a astrology section in our Thursday's paper".

67
a.The Indian express says that it will issue a astrology section in their Thursday's paper.

b.The Indian express says that they will issue a astrology section in their Thursday'spaper.

c.The Indian express said that it will issue a astrology section in its Thursday's paper.

d.The Indian express says that it will issue a astrology section in its Thursday's paper.

6. John says, "I shall go there".

a.John said that he went there.

b.John says that he will go there.

c.John says that he went there.

d.John said that I will go there.

7. Robert will say to me, "I am your classmate".

a.Robert will tell me that he is my classmate.

b.Robert will tell me that he was my classmate.

c.Robert will tell me that he will be my classmate.

d.Robert said me that he is my classmate.

8. He said to me, "I shall write an essay".

a.He said to me that he will be writing an essay.

b.He told me that he would write an essay.

c.He said to me that he will write an essay.

d.He told me that he would write an essay.

9. He said to me, "I shall be writing an essay".

a.He told me that he would have been writing an essay.

b.He told me that he would be writing an essay.

c.He told me that he will be writing an essay.

68
d.He told me that he shall be writing an essay.

10. Sita said to Geeta, "You can learn piano".

a.Sita told Geeta that she can be learn piano.

b.Sita told Geeta that you can learn piano.

c.Sita told Geeta that she could learn piano.

d.Sita told Geeta that she can learn piano.

69
VOCABULARY ENRICHMENT
Pair-Define-Explain
Learn vocabulary by sprinkling ideas with interesting words and phrases.
Pair the word or phrase with high-frequency synonym, definition, explanation or visual
depiction.
Repeat them frequently always pairing with the explanatory source.
Examples
He committed an egregious error a very bad mistake.
The liquid becomes effervescent bubbly, full of bubbles when we stir it.
The Pilgrims embarked on a long journey. They beganthey started on a long trip.
She was motivated by vengeance she wanted to punish him, sort of get back or get
even with him.

Vocabulary Bingo
Makes learning new vocabulary fun.
Lets learn through Bingo grids.
Lets fill out the weeks vocabulary words in any pattern on their papers.
Instead of learning and understanding the word only, lets learn synonyms, antonyms and
meaning of the word
Winner must not just say the winning word, but use them all in a sentence!
Lets increase your motivation and participation by using a approach to take interest in
Vocabulary Bingo.

WORD SYNONYM ANTONYM


adulterate contaminate purify
augment enlarge decrease
dour harsh cheery
fortitude steadfastness timidity
gibe ridicule compliment
opulent rich destitute
pliable supple rigid
stolid impassive emotional
tentative provisional definite
unkempt sloppy well-groomed
verbatim precisely paraphrased
warily carefully recklessly
indulge oblige deny
luster gloss tarnish
miscellaneous varied uniform
peevish cranky pleasant
cater pamper deny
dissuade discourage persuade

70
indifference apathy interest
indignant offended pleased
poised self-confident nervous
barren unproductive fertile
disrupt upset organize
insinuate imply barge in
animated vigorous lifeless
culminate conclude begin
goad Prod curb
literate educated Unlettered
peevish crabby greeable
Barren unproductive Fertile
Disrupt upset Organize
Insinuate imply barge in
Animated vigorous Lifeless
Culminate conclude Begin
Goad Prod Curb
Literate educated Unlettered
Peevish crabby Greeable
Singe scorch Incinerate
Unique unparalleled Ordinary
Upright perpendicular Horizontal
Verify prove Refute
Plague epidemic Blessing
Poised collected Nervous
Transparent clear Frosted
Unscathed unhurt Injured
Unscathed unhurt Injured
Adjacent alongside Faraway
Alight dismount Mount
Hurtle peed Crawl
Insinuate imply barge in
Interminable never-ending Fleeting
Uncertainty doubtfulness Sureness
Tactful skillful Clumsy
Substantial considerable Insignificant

EXERCISE 1
Choose the one word from the circle whose synonym and antonym are provided in the table:

Synonym Antonym Word


Contaminate purify
Enlarge decrease
Harsh cheery

71
Steadfastness timidity
Ridicule compliment
Rich destitute
Supple rigid
Impassive emotional

Provisional definite

Sloppy well-groomed
Precisely paraphrased
Carefully recklessly
Oblige deny
gloss tarnish
varied uniform

(Note- This picture is related to exercise 1)


72
Exercise2
Choose the one word from the circle whose synonym is provided in the table:

No. Synonym Word


1 stolid
2 uncertain
3 bloodbath
4 picky
5 predicament
6 dubious
7 obvious
8 deceptive
9 planned
10 predict
11 ridicule
12 cranky
13 discourage
14 poised
15 Tactful

73
(Note- This picture is related to exercise 2)

Exercise 3

Choose the one word from the circle whose antonym is provided in the table:

74
(Note- This picture is related to exercise 3)

No. Antonym Word


1 Clumsy
2 Guardian
3 Hinder
4 Merry
5 Gloomy
6 Lofty
7 Sober
8 Rigid
9 Exhibit

75
10 Nourish
11 Generous
12 Triumph
13 Haw
14 Reluctant
15 Authentic

Exercise4 BINGO GRID

In the grid provided below, there are 20 different meanings given alongwith the numbers. Match the
words with their meaning.
a. ambidextrous b. Bereft c.deploy d. Dour e. Fortitude f. Reiterate g. pliable h. Guise i. Maul j.
Nonchalant k. Austere l. Blaze m. splinter n. Deride o. erudite p. Fabricate q. Implacable r.
Inept s. fractious t. Despot

1. easily bent 2. learned or 3. feeling or 4. to make up 5. ill-humored


scholarly appearing casually
calm and relaxed

6. troublesome or 7. 8. to say again 9. able to use both 10. an external


irritable break or cause to hands equally well appearance
break into small
sharp fragments

11. incapable of 12. a very large or 13. deprived of 14. not suitable or 15. batter
being appeased or fiercely burning fire capable
mitigated

16. courage in 17. laugh at with 18. a dictator with 19. severe or strict 20. to position or
facing difficulties contempt absolute power in manner or arrange
attitude

76
Exercise5 BINGO GRID
In the grid provided below, there are 20 different words are given alongwith the numbers. Match the
words with their meaning.
a) weaken mentally or morally
b) full of trivial conversation
c) the lowest point of anything
d) seize and take control without authority
e) go against, as of rules and laws
f) having or showing knowledge or understanding or realization
g) the quality of being honest and straightforward
h) choose and follow a theory, idea, policy, etc.
i)unnecessary and unwarranted
j)demanding immediate attent
k)assign to a lower position
l)the dominance or leadership of one social group over others
m)demanding immediate attention
n)occurring by happy chance
o)plausible but false
p)characterized by dignity and propriety
q)tending to vanish like vapor
r)concise and full of meaning
s)someone who exhibits independence in thought and action
t)peculiar to the individual

1. arrogate 2. candor 3. cognizant 4. contravene 5. enervate

77
6. espouse 7. evanescent 8. evince 9. exigent 10. fortuitous

11. garrulous 12. gratuitous 13. hegemony 14. idiosyncratic 15. maverick

16. nadir 17. pithy 18. relegate 19. spurious 20. staid

Exercise6 BINGO GRID


In the grid provided below, there are 20 words given alongwith the numbers. Match the words with
their respective antonym.
a. unscathed b. Tactful c. Interminable d. Culminate e. Peevish f. Poised g.
Unique h. opulent
i. carefully j. Unlettered k. Peed l. Vigorous m. Indignant n. Definite o.
Peevish p. scorch
q. verify r. Contaminate s. Substantial t. unkempt

1. crawl 2. incinerate 3. ordinary 4. refute 5. greeable

6. insignificant 7. provisional 8. destitute 9. injured 10. fleeting

78
11. pleased 12. educated 13. nervous 14. lifeless 15. begin

16. well-groomed 17. purify 18. recklessly 19. greeable 20. clumsy

79
ESSAY WRITING
What is an essay?

Though the word essay has come to be understood as a type of writing in Modern English, its
origins provide us with some useful insights. The word comes into the English language through
the French influence on Middle English; tracing it back further, we find that the French form of
the word comes from the Latin verb exigere, which means "to examine, test, or (literally) to drive
out." Through the excavation of this ancient word, we are able to unearth the essence of the
academic essay: to encourage students to test or examine their ideas concerning a particular
topic.

Essays are shorter pieces of writing that often require the student to hone a number of skills such
as close reading, analysis, comparison and contrast, persuasion, conciseness, clarity, and
exposition. As is evidenced by this list of attributes, there is much to be gained by the student
who strives to succeed at essay writing.

The purpose of an essay is to encourage students to develop ideas and concepts in their writing
with the direction of little more than their own thoughts (it may be helpful to view the essay as
the converse of a research paper). Therefore, essays are (by nature) concise and require clarity in
purpose and direction. This means that there is no room for the students thoughts to wander or
stray from his or her purpose; the writing must be deliberate and interesting.

The major types of essays

1) Narrative Essays: Telling a Story

2) Descriptive Essays: Painting a Picture

3) Expository Essays: Just the Facts

4) Persuasive Essays: Convince Me

5) Argumentative Essays: Logic/ Argument

6) Narrative Essays: Telling a Story

In a narrative essay, the writer tells a story about a real-life experience. While telling a story
may sound easy to do, the narrative essay challenges students to think and write about
themselves. When writing a narrative essay, writers should try to involve the reader by making
the story as vivid as possible. The fact that narrative essays are usually written in the first
person helps engage the reader. I sentences give readers a feeling of being part of the story.
A well-crafted narrative essay will also build towards drawing a conclusion or making a personal

80
statement.
Descriptive Essays: Painting a Picture

A descriptive essay paints a picture with words. A writer might describe a person, place, object,
or even memory of special significance. However, this type of essay is not description for
descriptions sake. The descriptive essay strives to communicate a deeper meaning through the
description. In a descriptive essay, the writer should show, not tell, through the use of colorful
words and sensory details. The best descriptive essays appeal to the readers emotions, with a
result that is highly evocative.

Expository Essays: Just the Facts

The expository essay is an informative piece of writing that presents a balanced analysis of a
topic. In an expository essay, the writer explains or defines a topic, using facts, statistics, and
examples. Expository writing encompasses a wide range of essay variations, such as the
comparison and contrast essay, the cause and effect essay, and the how to or process essay.
Because expository essays are based on facts and not personal feelings, writers dont reveal
their emotions or write in the first person.

Persuasive Essays: Convince Me

While like an expository essay in its presentation of facts, the goal of the persuasive essay is to
convince the reader to accept the writers point of view or recommendation. The writer must
build a case using facts and logic, as well as examples, expert opinion, and sound reasoning. The
writer should present all sides of the argument, but must be able to communicate clearly and
without equivocation why a certain position is correct.

Argumentative Essays: Logic/ Argument

This is the type of essay where you prove that your opinion, theory or hypothesis about an
issue is correct or more truthful than those of others. In short, it is very similar to the
persuasive essay (see above), but the difference is that you are arguing for your opinion as
opposed to others, rather than directly trying to persuade someone to adopt your point of
view.

Descriptive Elements

Description should be present in any type of writing, from narration to the others
covered here.

81
Think about the dominant impression you want to give; dont attempt to describe every
single thing.

Use imagery to put your reader right there with you.

Order events in time and space in such a way that the reader can imagine the location
and time easily.

USE ADJECTIVES AND ADVERBS! Instead of: It was hot. In the afternoon, we rested on a
log before hiking further into the woods.

Try: Once the afternoon sun began to blaze down upon us, exhaustion set in, and we collapsed
on a nearby moss-covered log, groaning under the weight of its ancient branches. Our hike into
the heart of the forest would soon resume.

USE A THESAURUS! Instead of: old Try: elderly, aged, long in the tooth clean try:
spotless, hygienic, sterile good try: marvelous, wonderful, incomparable, unrivaled bad
try: dreadful, horrific, appalling, ghastly

Argumentative & Persuasive

Background: historical & social context of a controversial issue

Proposition: your thesis statement; what you will argue, including a roadmap of how
you will argue it

Qualifications: any limitations to your argument; for example, you may be opposed to
abortion, except in cases of rape or the health of the mother

Refutation: identification of the opposing position and its flaws (provide evidence!)

Support: your reasoning, facts, examples, statistics, opinions of authorities, and other
evidence to make your case

Conclusion: come full circle, with a conclusion that restates your thesis and shows how
you have made your case, summarizing the most important points you made

Dos of Essay Writing

Do Make your Essay Easy to Read

Because of the large number of essays that have to be graded, many instructors admit that they
skim read essays to get an idea if a student is competent in the topic and how well they have

82
adhered to the style guidelines. Therefore, it is a good idea to structure your essay so the key
points are visible when skim reading and are clear enough to get the message across.

Do include a Thesis Statement in the Introduction


A thesis statement is an important part of introduction and the essay in general, so it should
never be neglected. The thesis statement should reveal the main idea of the essay in a concise
format.

Do use Transitions Between Paragraphs

Sometimes paragraphs sound like separate pieces of text put together. This is the wrong
approach to writing.

Your essay should be smooth and coherent, leading the reader from one point to another. This
is why you should use transitions the phrases that help to connect each idea with the
previous one, serving like bridges between paragraphs.

Examples of phrases you can use for transitions include:

Despite the previous arguments

Speaking about this

Regarding this

With regards to this

As has been noted

To put it briefly

Do cite examples

Any example you use from literature, scientific work, etc. - should be cited.

Do Use Advanced Vocabulary

The aim of an essay is to not only to reveal your knowledge of the topic, but to show your
ability to choose appropriate vocabulary and show your language expertise.

You should show that your vocabulary has progressed since high school. That means using
advanced vocabulary and replacing good and nice with more appropriate synonyms to reflect
the shades of meaning.

83
Do Revise your Writing Thoroughly

Before you hit Save and print the final version, check your essay thoroughly to avoid spelling
mistakes, typos and incorrect sentence constructions. Apart from language mistakes, check if
you followed all the requirements: number of words/pages, text formatting, essay structure,
etc.

Donts of Essay Writing

Don't Overwhelm your Essay with Information and Facts

Though essays should be meaningful and detailed, learn to filter the information and choose
only important points.

There is a temptation to include every single detail of your research to make the essay holistic
and complete. However, your aim is to narrow the topic, show that you are able to analyze and
structure information, and choose only the most relevant facts to prove your points

Don't Use Too Many Clichs

While using set phrases, avoid overwhelming your essay with clichs.

Remember that not all clichs are good for every type of essay. What's more, professors expect
your work to be original and truly value students with fresh ideas and views. Also, beware of
using informal language. This doesn't mean that your writing should be reminiscent of a
scientific thesaurus rather than real speech, but academic writing requires a certain level of
formality.

Do Not Address the Reader


No matter what type of essay you are writing, academic writing rarely reveals the author nor
engages in the conversation with the reader. Addressing the reader is more a mark of fiction
than an academic essay. While writing a college essay you should be detached, objective and
analytical rather than appeal to the readers emotions and personality.

Dont Start an Essay with in this/my essay Phrase


The introduction has to present the main idea of the essay and reveal what you are going to
talk about.

Writing an effective introduction and including a thesis statement is enough to lead the reader
into the context of your essay topic without using this meaningless high school phrase In my
essay Im going to focus on

84
Dont Use Negative Language

Negative language doesnt mean vulgarisms. It means words with negative suffixes, phrases
with negation, etc.

For example, painless is not a negative word in its meaning. However, using it makes the reader
focus on pain instead of its absence. So it is better to replace so called negative language with
more positive, synonymous expressions, like using economical instead of inexpensive, or
comfortable/pleasant instead of painless.

The essay has the following standardized format:

1) Introduction

2) Elaboration

3) Conclusion

Introduction Paragraph

An attention-grabbing "hook"

A thesis statement

A preview of the three subtopics you will discuss in the body paragraphs.

First Body Paragraph

Topic sentence which states the first subtopic and opens with a transition

Supporting details or examples

An explanation of how this example proves your thesis

Second Body Paragraph

Topic sentence which states the second subtopic and opens with a transition

Supporting details or examples

An explanation of how this example proves your thesis

Third Body Paragraph

Topic sentence which states the third subtopic and opens with a transition

85
Supporting details or examples

An explanation of how this example proves your thesis

Concluding Paragraph

Concluding Transition, Reverse "hook," and restatement of thesis.

Rephrasing main topic and subtopics.

Global statement or call to action.

More tips to make your essay shine

Planning Pays

it is always better to brainstorm a bit before beginning your essay.

This enables you to find the best supporting ideas rather than simply the first ones
that come to mind and position them in your essay accordingly.

Your best supporting idea the one that most strongly makes your case and,
simultaneously, about which you have the most knowledge should go first.

Even the best-written essays can fail because of ineffectively placed arguments.

Aim for Variety

Sentences and vocabulary of varying complexity are one of the hallmarks of effective
writing.

Avoid using the same words and phrases over and over again.

Avoid beginning sentences with the dull pattern of "subject + verb + direct object." But
use different kinds of sentences like compound, complex etc.

Practice! Practice! Practice!

Remember that good writing does not happen by accident.

Practice writing sample essays on various topics. Even if they are not masterpieces at
first, a bit of regular practice will soon change that and make you better prepared
when it comes to the real thing.

Effective essay Argument


86
A strong argument tries to persuade the reader to accept a point of view using:

1. A declarative statement of an idea or opinion.

2. Support for the statement: including relevant facts, opinions based on facts and/or
careful reasoning.

Analysis of Argument questions present a short argument on an issue where you discuss how
well it is reasoned.

Exercise: Write essays on the following topics in 300-350 words

1.According to some people, elderly drivers should be required to reapply for their driving licenses
because with age comes diminished vision, hearing, and reaction time. How do you feel about this
issue? Explain what you think should be done and why.

2.Have you ever traveled to a place that you found very meaningful and rewarding? Write an essay that
persuades others to visit this important place.

3.If it were up to you to choose one item from the twenty-first century to place in a time capsule for
future generations, what would you choose? Use specific reasons and examples to support your choice,
explaining both the items significance and the reasons why it embodies the culture of the early twenty-
first century.

4.Thomas Edison, the renowned inventor, is famous for having said, Genius is one percent inspiration,
ninety-nine percent perspiration. Do you agree with this definition of genius? Why or why not?

5.Explain the causes and effects of not voting in elections

6.Many people spend a great deal of time with animals. Write about the relationships that people have
with animals.

7.People often say Dont judge a book by its cover. Describe a time when you misjudged someone
based on his or her appearance or when someone misjudged you.

8. Carpooling, recycling, and planting trees are all activities that are good for the environment. Write an
essay convincing readers to actively participate in one of these activities.

87
READING COMPREHENSION
What is it?

Reading comprehension is the ability to read text, process it, and understand its meaning.

Comprehension varies with capability

Recognition, decoding, and fluency are building blocks of effective reading

Comprehension is a prerequisite

Requisites for readers to comprehend

Interpret and evaluate events, dialogue, ideas, and information

Connect information to what they already know

Adjust current knowledge to include new ideas or look at those ideas in a different way

Determine and remember the most important points in the reading

Read between the lines to understand underlying meanings

Types of questions

Discovering main idea

Identifying detail / fact

Drawing conclusions / predicting outcomes

Distinguishing between fact and opinion

Understanding cause and effect

Comparing and contrasting ideas

Determining Authors Purpose

Key factors

Motivation and Purpose

Vocabulary and Previous Knowledge

Reading fluency and decoding

88
Nature of Passage (Interest, Difficulty)

Genre of text (Fiction, non-fiction, poetry, science etc.)

Time limit (Time consuming, GATE)

Comprehension Strategies

Time management

Skimming - Zig zag movement of the eye

Accompanied by grasping of major ideas of the passage

89
Scanning (Eye reading)

Reading using ones eyes and getting details, as in, facts, figures, examples (Specific info.)

SQW3R - SURVEY, QUESTION,WRITE, READ, RECALL REVIEW


This is an active reading method that, although will seem time consuming in the beginning, will enhance
your reading comprehension and learning efficiency

Survey the text


If it is a new book make sure you take a few minutes to look over the table of contents. Bring your
syllabus out and look to see how your professor broke the book chapters up - is the professor following
the format of the book or is he/she changing how the order of the chapters will be read.

Survey the entire chapter before you start reading so that you become familiar with what will be
presented and how much time you need.

Look at the title and all subtitles to know what the chapter is about and how it is broken up into
parts. Label major headings with Roman numerals and subheadings with capital letters to show
the relationship between topics.
Look at the chapter outline - it is the blueprint to the chapter
Look at the end of the chapter aids - questions, summaries... this will give you an idea of what
the author feels is important and help you select the main ideas as you read
Read the introduction and/or first paragraph first - this will tell you the purpose of the chapter
Read the last paragraph - this will summarize the key information
Look to see how important terminology is presented (in bold, italic...). If there is a lot of
terminology you should bring out index cards and create a terminology index as you read

Question What am I looking for?


Before you begin reading, turn the subheadings into questions so that your mind is actively looking for
answers rather than passively reading along.

Turn subheadings into questions


Create questions from information printed in the margins
Create questions for each graph presented
Try writing out the questions so that as you read you can periodically look at the questions and
stop to see if you can recall the information.

WRITE Take notes in your own words

Write your own sub heading, summarize the things. Always write key points.

READ
Concentration and attention to detail are essential to active reading and comprehension.

90
You must locate the main idea of each paragraph
Read the paragraph and identify the topic - usually the idea that is repeated once or twice
Find the main idea - the sentence that summarizes the details
Look for the supporting details: key words, small groups of words - make sure the details refer
to the same main point. Now underline the main idea
Write the answers to the questions you developed from the subheadings. You have now
developed a practice test for the chapter!

RECALL how much you remember from the text, keep away the text, recall the details to prepare new
notes, then compare your notes with recall knowledge

Assures that you comprehend what you are reading


Helps avoid plagiarizing if writing a research paper
Helps you remember the main idea and details or examples
As you read try to associate read material with life experience for better recall

REVIEW- Revise your notes, answer your questions

Reviewing is a daily process. It is essential that this process take place in order for true learning and
comprehension of material to happen. The more you review and recite the better the recall. It may
seem like a tedious task but when this process becomes a consistent study habit it actually makes your
study time more efficient. You will no longer cram for exams because you are starting the exam process
on the first day of class.

Review starts with organization. Make sure your binder starts with the syllabus and all notes and
handouts are inserted in date order.

Review your notes and questions created for the chapter daily
Review the highlighted areas of your text
Review your comments you made in the text as you read
Recall the main points of each section of the chapter
Review your index card file
Create an outline from your texts subheadings, your class notes, reading notes, margin
comments, and the questions you developed for each chapter. This will act as a study guide for
the exam. Recite the information daily to ensure automatic recall and true learning. Make sure
you recite the information in your own words for better comprehension. Check and recheck the
information where you hesitate or aren't able to recall details.

Vocabulary

The better your vocabulary the better understanding and recall of information read. To improve your
vocabulary:

Read often and vary your reading material to give yourself a varied vocabulary background.
Read with a dictionary available. Do not assume you know the meaning of a word! If you
hesitate then look it up. Write the word on an index card and create a file of new words learned

91
each week. Write the definition in your own words and create a sentence using the word that
relates to your life. The more you associate the word to your life the better the recall.
Get a vocabulary calendar and learn a word a day! File the calendar pages in your word file.
Use the words each day in conversation.

Comprehension

The best way to know if you are comprehending material is to monitor your recall as you read.

Learn an active reading method to ensure you are staying engaged in the reading process. The
SQ3R method is described in later in this section
Try to recite details and main ideas after each section. The more you can recall the better the
comprehension. If you find that you cannot recall much, break your reading up into smaller
sections using the subheadings and again recite. Increase the section size as you see
improvement. Remember, concentration plays a large part in your ability to comprehend so
assess your focus.
Do not slow down your reading speed to try and compensate for lack of comprehension.
Research tells us that reading at a slower speed often interferes with comprehension because it
forces us to resort to word for word reading. Use and index card or ruler as a guide to keep
focused and your eyes moving.
You should see improvement in comprehension as you learn to monitor your recall, practice and
develop the habit of active reading, and increase your reading speed (next section).

Reading Speed

Increasing your reading speed can help you maximize your study time and develop additional skills for
better learning efficiency. In order for reading speed to increase with effective results you must have the
motivation to practice the necessary techniques and the desire to improve. Once you understand your
present reading rate you can learn to increase your speed by practicing the hints described below. These
hints are not to be confused with speed reading techniques that are best learned by taking a speed
reading course for maximum benefit.

Understand that each type of reading demands a different reading rate. An adventure novel
may allow for a quick rate whereas a science text may demand a steadier rate.
Most individuals fixate on each word. To increase speed you should proactively focus on 2-4
words at a time. For example: Increasing my/ reading rate will/ help me maximize/ my study
time.
The better your vocabulary, the better your recall, the faster you read, the better your
comprehension.
Mouthing the words while you read slows down your reading speed because it forces your to
focus on each word rather than groups of words.
Practice makes perfect therefore the more you read the more proficient you become. Practice
will increase speed, vocabulary, comprehension, knowledge base...
The type of reading you need to do dictates the reading rate. If you are reading for detail you
must read at a rate for comprehension. If you are reviewing, reading for general main ideas then
skimming and using a faster rate is appropriate.

92
To increase your speed you need to force yourself to read at a faster rate for short periods of
time. Use a card or ruler to guide your speed and focus on the page. Go at a speed that is
uncomfortable but you are still comprehending the material. This must be practiced daily. After
practicing the above for a few days, retime yourself reading at an average rate and you should
see a speed rate increase. The rate increase will only continue if you practice this technique. If
you do not practice then your speed rate will revert back to your previous "comfortable" rate.
Remember to check for comprehension as your reading rate increases to ensure that you are
increasing at appropriate increments to enhance learning.
Try to avoid rereading. Pay attention to your concentration so that rereading becomes
unnecessary. Rereading is usually a habit formed from lack of confidence in comprehension
ability. If you practice the SQ3R method and consistently recall and review while reading, the
rereading habit will become unnecessary.
You should never read at a rate that is slower than your average rate. Research shows a
correlation between reading rate and comprehension. Slower speed does not guarantee a
better understanding of the material, in fact, research implies that increasing the rate of speed
often results in higher comprehension. While you are increasing your reading rate, it is
important to constantly monitor your comprehension by periodically recalling details and main
ideas to ensure that you are not forcing a speed that might hinder rather than enhance your
reading efficiency.

EXERCISE

1. But I did not want to shoot the elephant. I watched him beating his bunch of grass against his knees,
with the preoccupied grandmotherly air that elephants have. It seemed to me that it would be murder
to shoot him. I had never shot an elephant and never wanted to. (Somehow it always seems worse to kill
large animal.) Besides, there was the beast's owner to be considered. But I had got to act quickly. I
turned to some experienced-looking Burmans who had been there when we arrived, and asked them
how the elephants had been behaving. They all said the same thing; he took no notice of you if you left
him alone, but he might charge if you went too close to him.
1.) The phrase preoccupied grandmotherly air signifies:
a) being totally unconcerned b) pretending to be very busy
c) very superior attitude d) calm, dignified and affectionate disposition
2.) From the passage it appears that the author was
a) an inexperienced hunter b) kind and considerate c) possessed with fear d)a worried man

2. As the mid-century approached, the women of America were far from being acclimated to their
assigned dependent role. In fact, leaders of the growing suffrage movement were seeking equality
under the law. Incredible as it seems now, in early nineteenth-century America a wife, like a black slave,
could not lawfully retain title to property after marriage. She could not vote, and she could legally be
beaten by her master.
1.) One of the goals of the suffrage movement was
a) dependence on a master b) equality with men c) recognition of divorce. d) abolition of slavery.
2.) Which sentence describes American women of the early 19th century?
a) They were against marriage. b) They were satisfied with their role in society.

93
c) They were victims of a male-dominated society. d) They had many slaves to do their work.

3. Jagir Singh has sold red onions at the market in South Delhi every day for the past half century.
Perched on an upturned create, wrapped tight against the chilly air, he offers pyaz, a staple for much of
Indian cooking, for 60 rupees a kilo, the most he can remember. Business is brisk but most customers
pick up only a small handful of onions. That is just as well wholesale supplies are tight, he says, and the
quality is poor. As the Indian economy grows by some 9% a year, food prices are soaring. In late
December the commerce ministry judged that food inflation had reached 18.36%, with pricey
vegetables mostly to blame. Officials have made some attempts to temper the rise in the past month
scrapping import taxes for onions, banning their export and ordering low priced sales at government
run shops. But, there is no quick fix.
Heavy rain in the west of India brought a rotten harvest. Vegetables from farther afield, including a
politically sensitive delivery from a neighbouring country, are costly to move on Indias crowded,
potholed roads. Few refrigerated lorries and poor logistics mean that much of each harvest is wasted.
Newspapers alleged that collectors are cashing in. The biggest problems are structural. Food producers,
hampered by land restrictions, archaic retail networks and bad infrastructure, fail to meet extra demand
from consumers. It was estimated in October that a 39% rise in income per person in the previous five
years might have created an extra 220 million regular consumers of milk, egg, meat and fish. Supplies
have not kept up with this potential demand.
The broader inflation rate may be a less eye - watering problem than the onions suggest. The central
bank has lifted interest rates steadily in the past year and is expected to do so again later this month.
Headline inflation fell to 7.5% in November, down by just over a percentage point from October, though
it is still above the central banks forecast of 5.5% for March.
1) The usage of the phrase cashing in in the passage can possibly mean
a) Profiting b) Running Away c) Bailing Out d) Buffering
2) Which of the following is most opposite to the word Archaic as used in the passage?
a) Simple b) Straightforward c) Modern d) Ancient
3) Which of the following is/are the reason/s for increase in food/vegetable prices?
(1) Bad weather
(2) Land restriction
(3) Poor infrastructure for storage and transportation
a) Only 2 b) Only 1 & 3 c) Only 1 & 2 d) All 1, 2 & 3
4) Which of the following is most similar in meaning to the word Temper as used in the passage?
a) Displeasure b) Rage c) Harness d) Control
5) Which of the following is possibly the most appropriate title for the passage?
a) Food Inflation in India b) Food Deficit Worldwide
c) Food Imports in India d) Benefits To Indian Consumers

4. The first step is for us to realise that a city need not be a frustrate of life; it can be among other
things, a mechanism for enhancing life, for producing possibilities of living which are not to be realized
except through cities. But, for that to happen, deliberate and drastic planning is needed. Towns as much
as animals, must have their systems of organs-those for transport and circulation are an obvious
example. What we need now are organ systems for recreation, leisure, culture, community expression.
This means abundance of open space, easy access to unspoilt Nature, beauty in parks and in fine
buildings, gymnasia and swimming baths and recreation grounds in planty, central spaces for

94
celebrations and demonstrations, halls for citizens' meetings, concert halls and theatres and cinemas
that belong to the city. And the buildings must not be built anyhow or dumped down anywhere; both
they and their groupings should mean something important to the people of the place.

1) What does "A city need not be a frustrater of life" in the passage means?

2) What does the "The building must not be built anyhow or dumped down anywhere" implies in the
passage?

3) The author talks about 'Unspoilt Nature'. In what way can Nature remain unspoilt?

4) What is the function of a city according to the author?

5) What does the word 'drastic' in the passage means ? Frame it in a sentence of your own.

5. Lets say every religion has procedure for the marriage contract or bonding two people in such a
relationship that could frame out respect for generations. Traditions and cultures are different. But a
bonding aspect, the need of living or the instinctive desires are the same. They could not be negotiated
or could not be denied. Now can Muslims say only Nikkah is legitimate factor for the bonding. No. But
yes as it is Sunnah of Holy Prophet so it is the right way but repeating Sunnah without the spirit of
Sunnah is wrong just to fulfill the need for sake not the respectful worthy values of life. Nowadays this
legitimate aspect is equal to lust fulfillment. So when the generation is being raised in such
circumstances they neglect the original version of the topic and practical aspect. Whatever is the easier
way to fulfill the need they may adopt that. So the young generations though they are in Muslim
community or in Muslim countries they are equally doing the same stuff which is legally, morally and
religiously not legitimate. They support this ideology because they themselves lack the practice and they
didnt watch their times to follow this religiously.

1)What is the tone of the author?


2)What cannot be negotiated?
3). When can Sunnah be termed as incorrect?
4). What does the author mean by ideology?

5). Use the phrase to follow religiously in a sentence of your own.

6. The grass so little has to do,


A sphere of simple green,
With only butterflies to brood,
And bees to entertain,
And stir all day to pretty tunes
The breezes fetch along,
And hold the sunshine in its lap

95
And bow to everything;
And thread the dews all night, like pearls,
And make itself so fine,
A duchess were too common
For such a noticing.
And even when it dies, to pass
In odors so divine,
As lowly spices gone to sleep,
Or amulets of pine.
And then to dwell in sovereign barns,
And dream the days away,
The grass so little has to do,
I wish I were the hay! By Emily Dickinson
1) The speaker writes that the grasses stir in the breezes. What is the meaning of stir here?
a) to mix b) to move c) to make a lot of noise d) to notice
2) What do the breezes bring to the grasses?
a) thread b) pearls c) pretty tunes d) bees to entertain
3) Why does the speaker say she wants to be the hay?
a) It has so little to do. b) It is a simple green colour.
c) It can dream. d) It smells divine.
4) What does the grass do at night?
a) entertain the bees b) hold the sunshine in its lap
b) thread the dews like pearls d) bow to everything
5) What is the main reason that Dickinson wrote this poem?
a) to teach readers facts about grass
b) to persuade readers to think about grass
c) to explain what happens to grass when it dies
d) to entertain readers with her thoughts about grass

7. "Two offenses of a very different nature, and by no means of equal magnitude, you last night laid to
my charge. The first mentioned was, that, regardless of the sentiments of either, I had detached Mr.
Bingley from your sister, and the other, that I had, in defiance of various claims, in defiance of honour
and humanity, ruined the immediate prosperity and blasted the prospects of Mr. Wickham. Wilfully and
wantonly to have thrown off the companion of my youth, the acknowledged favourite of my father, a
young man who had scarcely any other dependence than on our patronage, and who had been brought
up to expect its exertion, would be a depravity, to which the separation of two young persons, whose
affection could be the growth of only a few weeks, could bear no comparison. But from the severity of
that blame which was last night so liberally bestowed, respecting each circumstance, I shall hope to be
in the future secured, when the following account of my actions and their motives has been read. If, in
the explanation of them, which is due to myself, I am under the necessity of relating feelings which may
be offensive to yours, I can only say that I am sorry. The necessity must be obeyed, and further apology
would be absurd.

96
Pride and
Prejudice, by Jane Austen
1) According to the writer of the letter, which offense would be considered the worst?
a) ruining Mr. Wickhams prospects wantonly
b) separating Mr. Bingley from the letter readers sister
c) continuing dependency through patronage
d) relating feelings that may be offensive to the letter reader
2) Which of the following characters are the young persons mentioned?
a) Mr. Wickham and the letter readers sister b) the letter writer and the letter reader
c) the letter readers sister and Mr. Bingley d) the fathers favourite and the letter
reader
3) Near the end of the passage, the phrase secured is used to mean
a) tightly fastened b) safe from being charged c) captured or confined d) accomplished
4) The letter writers purpose in presenting his comments is
I- to defend his actions against the letter readers accusations of wrongdoing
II- to apologize for offending the letter readers feelings with his explanation
III- to satisfy himself in retrospect that his actions were appropriate
a) II only b) III only c) I and II only d) I, II, and III only

8. Of course, in one sense, the first essential for a mans being a good citizen is his possession of the
home virtues of which we think when we call a man by the emphatic adjective of manly. No man can be
a good citizen who is not a good husband and a good father, who is not honest in his dealings with other
men and women, faithful to his friends and fearless in the presence of his foes, who has not got a sound
heart, a sound mind, and a sound body; exactly as no amount of attention to civil duties will save a
nation if the domestic life is undermined, or there is lack of the rude military virtues which alone can
assure a countrys position in the world. In a free republic the ideal citizen must be one willing and able
to take arms for the defence of the flag, exactly as the ideal citizen must be the father of many healthy
children. A race must be strong and vigorous; it must be a race of good fighters and good breeders, else
its wisdom will come to naught and its virtue be ineffective; and no sweetness and delicacy, no love for
and appreciation of beauty in art or literature, no capacity for building up material prosperity can
possibly atone for the lack of the great virile virtues. By Theodore
Roosevelt
1) According to Roosevelt, being manly is
a) Being a good human b) Being a good citizen
c) Being a good husband and a father d) Being a good father
2) To be a good citizen man has to be
a) Faithful b) Fearless c) Honest d) All of the above

3) Antonym of Naught
a) Death b) Prosperity c) Misery d) Entirety
4) Synonym of Atone
a) Make up b) Build up c) Avail d) Uphold
5) By virtue Roosevelt summarize
97
a) Manly virtues b) Humanly virtues c) Citizenship d) All of the above

9. John has been arrested? Im afraid he is headed downhill. This is not the first time. Each time it gets
worse. Im worried about him. The tone of this is
a) angry. b) fearful. c) sarcastic. d) surprised.

10. Go on, John! Soon youll be able to steal a cookie without your mother knowing it. The tone of this
is
a) praising. b) happy. c) sentimental. d) sarcastic.

11. We sat around the fire in the old house. My brothers and I and our father hadnt all been together
for a year. It was a cold November day. Only a year had passed. Mothers shawl still hung over the back
of her rocker. It had hung there that night a year ago. It hung there quietly just as her memory hung in
our minds. We tried to talk, but silence kept breaking in - the silence of the rocker that creaked no more.
We four shared the silence of the rocker.
1. The tone of this passage is
a) humorous. b) hopeless. c) satiric. d) sentimental.

12.Renee: Do you have few minutes? This is regarding the booking of a meeting room for our discussion.
Michelle: Fire away!
Renee: We need to book a meeting room. But the meeting rooms are almost booked out. I'm still
working on that.
Michelle: Please try your best to get us a room ASAP. You know we cannot reschedule the rehearsal.
Time is ticking away.
Renee: I'll do my best. And I'll make sure all the presentation slides are ready on the computer.
Michelle: Good. And don't forget to give me the rundown on the rehearsal.
Sam: No problem. Renee and I will see to it. I have already filled in Samira too regarding this.
Michelle: Then that concludes our meeting today. I hope our meeting works wonders.
1) Choose the correct option
a) By Fire away, Michelle wants Renee to not disturb him that moment
b) Renee is not bothered about Michelles mood
c) Michelle is accommodating.
d) Renee is impolite
2) Michelle says that Time is ticking away because
a) The meeting is very concise
b) There is ample time for meeting to be held
c) The meeting is a prolonged meeting
d) None of the above
3) By give me the rundown Michelle is
a) Overconfident about the meeting
b) Wants Renee to submit the summary points in the meeting
c) Wants Renee to submit the detailed report in the meeting
d) Want all the participants to rehearse before the meeting
4) Choose the correct option :
a) The paragraph clearly says Samira is unaware of the issue

98
b) Samira will be handling another meeting at the same time
c) Samira is aware of the problem
d) Sam has no problem because Samira is in the team
5) Which of the following mean the same as it appears in the context above
a) Tajmahal is a wonder of the world
b) My friend works for a wonderful project
c) Scolding works wonders with toddlers
d) None of the above

13. What is immediately needed today is the establishement of a World Government or an International
Federation of mankind. It is the utmost necessity of the world today, and all those persons who wish to
see all human beings happy and prosperous naturally feel it keenly. Of course, at times we feel that
many of the problems of our political, social, linguistic and cultural life would come to an end if there
were one Government all over the world. Travellers, businessmen, seekers of knowledge and teachers
of righteousness know very well that great impediments and obstructions are faced by them when they
pass from one country to another, exchange goods, get information, and make an efforts to spread their
good gospel among their fellow-men. In the past, religious sects divided one set of people against
another, colour of skin or shape of the body set one against the other.
But today when philosophical light has exploded the darkness that was created by religious differences,
and when scientific knowledge has flasified the superstitions, they have enabled human beings of all
religious views and of all races and colours to come in frequent contact with one another . It is the
governments of various countries that keep people of one country apart from, those of another. They
create artificial barriers, unnatural distinctions, unhealthy isolation, unnecessary fears and dangers in
the minds of common men who by their nature want to live in friendship with their fellow-men. But all
these evils would cease to exist if there were one Government all over the world.
1) In the passage, what divides people of a country against another?
2) What is the world Government be expected to do?
3) What is the urgent heed of the world today in the passage?
4) What problems are mentioned in the passage to be solved with the establishment of world
Government?
5) What factor, according to the passage, sets one man against another?

14. Certain scraps of evidence bear out those who hold a very high opinion of the average level of
culture among the Athenians of the great age. Pericles's funeral speech is undoubtedly the most famous
evidence from Athenian literature, that its level was indeed high. However, Pericles was a politician, and
it is possible that he was flattering his audience. We know that thousands of Athenians sat hour after
hour in the theatre listening to the plays of the great Greek dramatists. The Greek plays, particularly the
tragedies, maintained an extremely high intellectual level throughout, with no letdowns, no concessions
to the lowbrows or to the demands of ''realism'', like the gravediggers scene in Shakespeare's Hamlet.
The music and dancing seen in these plays were also of an equally high level. The best modern parallel
can be seen in the restrained, difficult opera of the 18th century. The comparison is no doubt
dangerous, but can you imagine almost the entire population of an American city (in suitable
installments, of course) sitting through performances of Mozart's Don Giovanni or Gluck's Orpheus?
Perhaps the Athenian masses went to these plays because of a lack of other amusements. They could at

99
least understand something of what went on, since the subjects were part of their folklore. Undoubtedly
the theme of grand opera is not part of the folklore of the American people.

1) In the given passage whose sincerity is being questioned by the author ?

2) According to the passage, what is the average thinking of the Americans regarding operas?

3) What is the authors attitude towards Greek plays?

4) What is the suggestion of the author regarding Greek plays?

5) What do you mean by folklore? Use it in a sentence of your own.

15. With the voices of more animal rights activists being heard and propagated via the social media,
animal abuse has gained worldwide recognition as a serious crime punishable by law. Forgetting to
periodically feed your pets or even kicking them in anger could land you in jail or see you slapped with a
heavy fine. What then about animals who abuse humans? Should domesticated creatures be penalized
for displaying their true nature? Or is it a mistake of the owners that warrants criminal prosecution?
Despite their life-threatening encounter with a mad hound, both ten-year-old Samuel Ang and thirty-
nine-year-old Mr. Tan Kim Wei could not bear to hear of the monster being put down and its young
owner, thrown into a cell for ten months!

Perched comfortably, Samuel had been savouring some juicy rambutans. A fierce snarl violently jolted
him forward. Slipping, he hung on for dear life. The pit bull terrier propped itself up against the trunk
and clawed its fore limbs upward. Samuel's trousers soon dangled in ribbons, blood oozing down his
legs. "Somebody help me! It's ... It's going to kill me! Help! Help!" he yelled, legs flaring aimlessly in a
futile attempt to save his feet from the ferocious beast.

Then, without the slightest warning, Samuel came crashing down. That was all the canine needed. It
lunged at the boy, sank its teeth into his leg and hung on tight. Samuel hit it in vulnerable areas and one
of his punches met its chest. The hound howled with pain. Samuel tried to scramble away but the
agitated beast pounced on him within a split second. Though panic-stricken, Samuel doubled his efforts,
punching and kicking the vicious creature wherever he could.

Seeing the ugly tussle between man and beast, Mr. Tan sprinted over, grabbed the thick branch that had
brought Samuel down and rammed it at the savage creature. It leapt and yelped as it landed a few
meters away. A good portion of the heavy baton had ripped off. Before he knew it, the animal charged
at him with renewed vigour. A second swing and more wood and splinters flew but the pit bull fought
back relentlessly. Not long after, it managed to knock Mr. tan over. The sound of snapping and sight of
the teeth coming at him terrified Mr. Tan. "Isn't anyone going to help me? his heart cried out as he tried
his best to fend off the animal.
"Rover!" screeched a feminine voice, ending the senseless attack on a helpless victim and his innocent
saviour. What became of Rover and its young owner was a heart-wrenching tale indeed.
1) According to paragraph 1, ______ would land pet owners in jail
a) displaying their pets b) pampering their pets
100
c) encountering abused pets d) forgetting to feed their pets
2) Why had Samuel originally climbed the tree?
a) He had wanted to agitate the dog.
b) He had wanted to escape from the dog.
c) He had wanted to pick some rambutans for the dog.
d) He had wanted to eat the rambutans growing from it.
3) The phrase doubled his efforts suggests that Samuel ______
a) was in more pain than before b) was feeling better than before
c) had less energy to fend the dog off d) put in more energy to fend the dog off
4) Why had Mr. Tan sprinted over the scene?
a) He had wanted to attack the dog. b) He had wanted to rescue Samuel.
c) He had wanted to threaten the dog. d) He had wanted to pick up the tree branch.
5) What happened to the dog in the end?
a) Mr. Tan managed to kill the dog. b) The dog was given up for adoption.
c) The dog was ordered to be put down. d) The dog was ordered to be imprisoned.

16. India is rushing headlong towards economic success and modernization, counting on high-tech
industries such as information technology and biotechnology to propel the nation to prosperity. Indias
announcement that it would no longer produce unlicensed inexpensive generic pharmaceuticals bowed
to the realities of the World Trade Organisation while at the same time challenging the domestic drug
industry to compete with the multinational firms. Unfortunately, its weak higher education sector
constitutes the Achilles heel of this strategy. Its systematic disinvestment in higher education in recent
years has yielded neither worldclass research nor very many highly trained scholars, scientists or
managers to sustain hightech development.

There was a time when countries could achieve economic success with cheap labour and lowtech
manufacturing. Low wages still help, but contemporary largescale development requires a
sophisticated and at least partly knowledge based economy. India has chosen that path but will find a
major stumbling block in its university system. India has significant advantage in the 21st century
knowledge race. Its education sector is the third largest in the world in student numbers after China and
the United States. It uses English as the primary language of higher education and research. It has a long
academic tradition. Academic freedom is respected. There are a small number of high quality
institutions, departments and centres that can form the basis of quality in higher education. The fact
that the States, rather than the Central Government, exercise major responsibility for higher education
creates a rather cumbersome structure, but the system allows for a variety of policies and approaches.
Yet the weaknesses far outweigh the strengths. India educates approximately 10 per cent of its young
people in higher education compared with more than half in the major industrialised countries and 15
per cent in China. Almost all of the worlds academic systems resemble a pyramid, with a small high
quality tier at the top and a massive sector at the bottom. India has a tiny top tier. None of its
universities occupy a solid position at the top. A few of the best universities have some excellent
departments and centres and there are a small number of outstanding undergraduate colleges. The
University Grants Commissions recent major support of five universities to build on their recognised
strength is a step forward, recognising a differentiated academic system and fostering excellence. These
universities, combined, enroll well under 1 per cent of the student population.

101
1) What does the phrase Achilles Heel mean as used in this passage?
(a) Weakness (b) Advantage (c) Low quality (d) Quickness
2) Choose the word/ group of words which is most similar in meaning to STUMBLING BLOCK
(a) argument (b) frustration (c) fallout (d) hurdle
3) What did India agree to do at the behest of the World Trade Organisation?
(a) It would ask its domestic pharmaceuticals companies to compete with the international ones.
(b) It would buy only license drugs from USA
(c) It would not manufacture cheap common medicines without a license.
(d) None of these
4) The worlds academic system is compared to a pyramid with a _______ at the top and _________
at the bottom
(a) massive substandard sector/ small high quality
(b) tiny quality par excellence/huge substandard sector
(c) poor substandard sector/ small high quality
(d) None of the above
5) Choose the word/ group of words which is most opposite in meaning to CUMBERSOME
(a) handful (b) breathtaking (c) manageable (d) difficult

17. I wandered lonely as a cloud


That floats on high o'er vales and hills,
When all at once I saw a crowd,
A host, of golden daffodils;
Beside the lake, beneath the trees,
Fluttering and dancing in the breeze.

Continuous as the stars that shine


And twinkle on the milky way,
They stretched in never-ending line
Along the margin of a bay:
Ten thousand saw I at a glance,
Tossing their heads in sprightly dance.

The waves beside them danced; but they


Out-did the sparkling waves in glee:
A poet could not but be gay,
In such a jocund company:
I gazedand gazedbut little thought
What wealth the show to me had brought:

For oft, when on my couch I lie


In vacant or in pensive mood,
They flash upon that inward eye
Which is the bliss of solitude;
And then my heart with pleasure fills,
And dances with the daffodils. By -William Wordsworth

102
1) The poet is describing the beauty of
a) A bright sunny morning
b) A particular flower stretched across a bay
c) A type of penguins filled along the shore of a beautiful island
d) None of the above
2) In the line but they out-did the sparkling waves in glee the author implies
a) The daffodils are happily flying in a rhythmic wave
b) The beauty of the daffodils outweigh that of the waves that touch the shore
c) The daffodils dance along with the rhythmic wave of the shore
d) The daffodils are dancing due to the cool breeze flowing out of the bay
3) Jocund means
a) Joking b) High spirited c) Intelligent d)Grand
4) Pensive mood means
a) Happy b) Thoughtful c) Peaceful d)Worried
5) What wealth the show to me had brought the poet refers to wealth as
a) Power of watching such a scenery b) Pride of travelling
c) Joy of loneliness d) None of the above

18. True, it is the function of the army to maintain law and order in abnormal times. But in normal times
there is another force that compels citizens to obey the laws and to act with due regard to the rights of
others. The force also protects the lives and the properties of law abiding men. Laws are made to secure
the personal safety of its subjects and to prevent murder and crimes of violence. They are made to
secure the property of the citizens against theft and damage to protect the rights of communities and
castes to carry out their customs and ceremonies, so long as they do not conflict with the rights of
others. Now the good citizen, of his own free will obey these laws and he takes care that everything he
does is done with due regard to the rights and well-being of others. But the bad citizen is only restrained
from breaking these laws by fear of the consequence of his actions. And the necessary steps to compel
the bad citizen to act as a good citizen are taken by this force. The supreme control of law and order in a
State is in the hands of a Minister who is responsible to the State Assembly and acts through the
Inspector General of Police.

1. What does the expression 'customs and ceremonies' means ?


2. According to the writer, what is the responsibility of the police?
3. What reflects the main thrust of the passage?
4. What is the meaning of the following sentence "They are made to secure die property of
citizens against theft and damage"?
5. What expresses accurately the idea contained in the first sentence?

19. One of the main reasons of corruption in elections today is the lure of power which haunts the
politicians so much that they feel no qualms of conscience in adopting any underhand method to come
out successful. The Watergate Scandal in the U.S.A. is an eloquent example to testify to the fact how
even the top level politicians can stoop to the lowest level in order to maintain themselves in power.
Who does not remember how Adolf Hitler rode roughshod overall canons of electoral pro-priety to
103
capture power ? In India also the record of the various political parties is not clean. Corruption thrives in
elections because those in the field play on the psychology of the electorate. The voters are swayed by
the tall promises of the candidates to whose machinations they fall an easy prey. They are also
susceptible to fall an easy prey to the adulations of the politicians due to their illiteracy. Besides, in the
representative democracies today and particularly in big countries the constituencies are quite
extensive obviating the possibility of corrupt practices being discovered. Anti-corruption laws are
honored more in their breach than in their observance. Even the code of conduct to be observed by the
parties fighting the elections becomes a dead letter in as much as it is jettisoned out of existence and
thrown unscrupulously over board by the unfair politicians whose only aim is to maintain themselves in
the saddle.

1. Which example of the U.S.A. testifies to the fact that even the top level people can stoop very
low in order to maintain themselves in power ?
2. How does Adolf Hitler came to power?
3. Why according to the writer do the voters fall an easy prey to the machinations of the
politicians ?
4. Why according to the writer, do the corrupt practices indulged in elections go unnoticed ?
5. Give the most appropriate title to the above passage and state the reason for the title.

20. The announcement that Donna had reached middle age came stealthily and without warning.
Donna, at 42 years old, was in perfect health. She was fit, trim, and ate only the healthiest of foods,
eschewing soda and candy. She was still asked for her identification when buying wine at the grocery
store. She ran several miles every day; in local road races, she was known for beating people half her
age. Forty is the new twenty! her friends would say of Donnas seeming agelessness.
Donna heard the remark so often that a part of her believed that this aging thing was surely only for
other people. That all changed during what should have been a fairly innocuous event: a trip to the
Department of Motor Vehicles to renew her drivers license. Name? the clerk asked, without looking
up. Donna replied. Address? Donna gave it to her. Look through there, please, The clerk gestured
toward an eye-testing machine perched at the edge of the desk. Read the first four lines, she sighed.
Donna pushed her head against the contraption. Go ahead. Donna was perplexed. Everything was
fuzzy. I think the glass is dirty, Donna said, searching the desk for a tissue to wipe off the lens. Its not
dirty, the clerk said, her voice dripping with condescension and sarcasm. There must be some
problem, Donna said, pressing her head against the machine once more. Maam, do you wear
glasses? No, no, I dont, Donna responded, a little embarrassed now, as other people in the room
began turning to see what was going on. The clerk glanced down at Donnas old license and back up at
her. Welcome to middle age, sweetheart. Come back when youve had an eye exam and gotten
glasses.
1) What proverb best fits this story?
a) Time marches on. b) Time heals all wounds.
c) A stitch in time saves nine. d) There is no time like the present.
2) Which is the best antonym for stealthily?
a) secretly b) patiently c) blatantly (d) sneakily
3) Donna is eschewing candy and soda, she is _____________
a) overindulging in them b) avoiding them c) having them in moderation d) having them occasionally
4) Which of the following statements suggest condescension?
104
I. Peter is a bright student, he simply does not apply himself.
II. Ingrid is not invited to the party; she has no sense of style.
III. Cindys not too dumb, for a sixth grader.
a) I only b) I and II c) II and III d) I, II, and III
5) Which is the best synonym for innocuous?
a) inoffensive b) awkward c) stupendous d) dry

21. It was not until sometime after having passed through the course of training in two of our chief
schools of art that the author got any idea of what drawing really meant. What was taught was the
faithful copying of a series of objects, beginning with the simplest forms, such as cubes, cones, cylinders,
&c. (an excellent system to begin with at present in danger of some neglect), after which more
complicated objects in plaster of Paris were attempted, and finally copies of the human head and figure
posed in suspended animation and supported by blocks, &c. In so far as this was accurately done, all this
mechanical training of eye and hand was excellent; but it was not enough. And when with an eye trained
to the closest mechanical accuracy the author visited the galleries of the Continent and studied the
drawings of the old masters, it soon became apparent that either his or their ideas of drawing were all
wrong. Very few drawings could be found sufficiently "like the model" to obtain the prize at either of the
great schools he had attended. Luckily there was just enough modesty left for him to realise that
possibly they were in some mysterious way right and his own training in some way lacking. And so he set
to work to try and climb the long uphill road that separates mechanically accurate drawing from
artistically accurate drawing.
The Practice and Science of Drawing
by Harold Speed
1)It can be inferred from the passage that the great schools of art the author had attended
a) held drawing contests or other assessments to judge drawings accuracy
b) included trips to see the works of the old masters
c) neglected to teach drawing the human form
d) did not emphasize sufficient mechanical accuracy in their curricula
2) The author would likely agree with the following statements EXCEPT:
I It is important to learn to draw simple forms such as cones and cylinders.
II The painters represented in the galleries of the Continent produced works of great mechanical
accuracy.
III Artistically accurate drawing can only be learned from the schools of art such as the ones attended by
the author.
a) I only b) II only c) III only d) II and III only
3)The author would be most likely to advise a beginning art student to
a) avoid copying cubes, cones, and cylinders to begin with
b) develop both mechanically accurate and artistically accurate drawing skills
c) attend at least two great schools of art
d) focus on copying the human head and figure in various poses

22. Come to me in my dreams, and then


By day I shall be well again!
For so the night will more than pay
The hopeless longing of the day.

Come, as thou cam'st a thousand times,


105
A messenger from radiant climes,
And smile on thy new world, and be
As kind to others as to me!

Or, as thou never cam'st in sooth,


Come now, and let me dream it truth,
And part my hair, and kiss my brow,
And say, My love why sufferest thou?

Come to me in my dreams, and then


By day I shall be well again!
For so the night will more than pay
The hopeless longing of the day. By- Matthew Arnold

1) The mood of the song is


a) Whimsical b) Wistful c) Joyful d)Detached
2) Choose the correct statement
a) Thou means Your and Thy means You
b) Thou means Their and Thy means You
c) Thou means You and Thy means Your
d) Thou and Thy mean the same however are used for poetic emphasis
3) Choose the correct option
a) The poet is pining for the presence of someone b) The poet wants to have a fortune
c) The poet is cajoling his daughter d) None of the above
4) Hopeless longing of the day means
a) Bad times during the day b) The day is sultry and hot longing for a cool breeze
c) Longing without hope d) Miserably longing with hope
5) Come to me expresses the need to be
a) Physically present b) Feeling the presence c) Both a & b d)None of the above

23. It is a strange that, according to his position in life, an extravagant man is admired or despised. A
successful businessman does nothing to increase his popularity by being careful with his money. He is
expected to display his success, to have smart car, an expensive life, and to be lavish with his hospitality.
If he is not so, he is considered mean and his reputation in business may even suffer in consequence.
The paradox remains that if he had not been careful with his money in the first place, he would never
have achieved his present wealth. Among the two income groups, a different set of values exists. The
young clerk who makes his wife a present of a new dress when he hadnt paid his house rent, is
condemned as extravagant. Carefulness with money to the point of meanness is applauded as a virtue.
Nothing in his life is considered more worthy than paying his bills. The ideal wife for such a man
separates her housekeeping money into joyless little piles- so much for rent, for food, for the childrens
shoes; she is able to face the milkman with equanimity and never knows the guilt of buying something
she cant really afford. As for myself, I fall into neither of these categories. If I have money to spare, I can
be extravagant, but when, as is usually the case, I am hard up, then I am the meanest man imaginable.
1. In the opinion of the writer, what a successful businessman is expected to have?

2. What does the phrase lavish with his hospitality signifies?

106
3. What does low paid people should do, according to the author?

4. What is the meaning of the word paradox? Frame it in a sentence of your own.

5. How does the housewife, described by the writer, feel when she saves money?

24. The next few decades will see great changes in the way energy is supplied and used. In some major
oil producing nations, 'peak oil' has already been reached, and there are increasing fears of global
warming. Consequently, many countries are focusing on the switch to a low carbon economy. This
transition will lead to major changes in the supply and use of electricity. Firstly, there will be an increase
in overall demand, as consumers switch from oil and gas to electricity to power their homes and
vehicles. Secondly, there will be an increase in power generation, not only in terms of how much is
generated, but also how it is generated, as there is growing electricity generation from renewable
sources. <A>To meet these challenges, countries are investing in Smart Grid technology. Although Smart
Technology is still in its infancy, pilot schemes to promote and test it are already underway. Consumers
are currently testing the new smart meters which can be used in their homes to manage electricity use.
Cities are prime candidates for investment into smart energy, due to the high population density and
high energy use. It is here where Smart Technology is likely to be promoted first, utilizing a range of
sustainable power sources, transport solutions and an infrastructure for charging electrically powered
vehicles. The infrastructure is already changing fast. By the year 2050, changes in the energy supply will
have transformed our homes, our roads and our behaviour.

1) According to paragraph, what has happened in some oil producing countries?

2) What is the main idea of the final paragraph?

3) What can be inferred about the introduction of Smart Grid Technology?

4) i) Give the closest meaning of the word underway.

ii) Give an appropriate title to the passage.

25. Loren had been surreptitiously moving money from the accounts of his banks wealthiest clients to
one he created for himself in the Cayman Islands for over 8 years. He had every reason to believe that
no one suspected a thing, and he knew the money would be safe in an offshore account. He had been
prudent with all the money he had misappropriated from Signet Bank. He wore second hand suits to his
job as a bank teller there. He rode his bicycle from Anacostia the dirty blighted neighborhood he
called home to his Dupont Circle branch each day.

He rarely traveled outside of the Washington, DC area. He did nothing at all to raise a red flag. He
appeared as nothing more than an 18-dollar-an-hour bank employee in a town where many made
millions. No one, he was sure, had ever scrutinized the myriad of transactions that had taken place
dozens, sometimes hundreds of small transfers that amounted to $9,000 every other week. Otherwise,
the truth would have surely come to light a long time ago.

107
Nine thousand dollars every other week is just a drop in the bucket to these rich folks, Loren
reasoned. This relieved his guilt. In Lorens mind, it was the greedy people who got caught doing these
things. It was the ostentatious ones, too. It was the guys who built multi-million dollar homes with 24
rooms on a paupers salary.

To him, the bank was fraught with idiots; no one was capable of unraveling his master plan.
No it was too late for them. Today was the day Loren would begin reaping the benefits of all his
patience and cunning and begin living in blissful anonymity. After an ordinary day at work, Loren got on
his bike and road over the river to National Airport. As the plane lifted off the runway for the Caribbean,
Loren watched intently out the window. He had close to $2 million collecting interest in the bank and
had managed to pull it off, unnoticed. As the plane landed, Loren finally allowed himself to fully dream
of the carefree life he was about to begin. He moved down the airplane steps, across the tarmac and
headed toward the airport. The pleasant air felt comforting to his skin. He breathed in slowly and deeply
with anticipation, and then he was surprised to hear a familiar voice.Hey, Loren. Not so fast.

1) Which of the options is the best way to rewrite the following sentence from paragraph 2 while
keeping its original meaning as used in the story?
"He had been prudent with all the money he had embezzled from Signet Bank."
a) He had been showy with the funds he had taken from Signet Bank.
b) He had saved all the money he had deposited into Signet Bank.
c) He had felt guilty about all the money he had taken from Signet Bank.
d) He had been discreet with all the money he had stolen from Signet Bank.
2) The expression "to raise a red flag" means to
a) be guilty b) appear impoverished c) seem very smart d) give a warning signal
3) Which is the best synonym for myriad?
a) overflow b) number c) numerous d) shortage
4) What were some of the things Loren did to not raise suspicion?
I. He wore secondhand suits.
II. He rode a bicycle to work.
III. He only made a certain number of transactions per week.
a) I only b) I and II c) I and III d) I, II, and III
5) Which is the best antonym for unraveling?
a) Unrevealing b) Disentangle c) Sorting out d) Disengaging

26. The option in India will soon apply not only to the bags that hold the groceries, but also to the cash
used to purchase them. The Reserve Bank of India (RBI) is preparing to circulate 1 billion plastic notes of
10 rupees (6 fils) in five cities to test their practicability. The purpose of the new notes was to increase
the lifespan of the currency and combat counterfeiting. The five cities - Kochi, Mysore, Jaipur,
Bhubhaneshwar and Shimla - have been chosen for their geographic disparity and to test the effect of
their varying climates on the notes. No date has been announced for the start of the trials. Plastic
currency notes - or polymer banknotes, as they are also called - were first issued in Australia in 1988 and
have since been adopted in Singapore, Brazil, Mexico and Nigeria, among other countries. Only a
handful of nations have switched over entirely to polymer currency. They include Canada, New Zealand,
Brunei and Vietnam.
There have been no estimates in India of the cost of printing banknotes on paper versus plastic. But
central banks in Canada and New Zealand have said that plastic notes cost twice as much to produce.

108
However, polymer notes have an average lifespan of five years, compared with one year for paper
notes. You can tear paper with your fingers. You cant do that with polymer notes, Mr. Jhunjhunwalla
said. It isnt easy to write on polymer notes or crease them. Paper is affected in climate that is too cold
or too warm or too rainy. For the RBI, the durability of plastic cuts the expense of printing
replacements for soiled paper notes and disposing of those taken out of circulation. According to the
RBIs annual report for the year from 2009 to 2010, 13 billion banknotes - nearly a quarter of all the
notes in circulation - had to be destroyed. Until the mid-1990s, retracted banknotes were burnt. Today,
as in many other countries, soiled paper notes are shredded. The RBI has tried to recycle shredded notes
into novelty paperweights, bricks or cardboard. But Mr. Gandhi said they discovered the paper was so
finely shredded that they could not even give it away. The shredded notes now make their way to
landfills and land reclamations.

1.) What's the main reason to introduce plastic currency by the Reserve Bank of India ?
a) To improve security features to defeat the efforts of counterfeiters.
b) For improving the life of bank notes.
c) Plastic notes are stain proof and don't tear easily.
d) Other countries i.e. Australia and Singapore have also launched plastic notes.
2. Why only five selected cities are chosen for trial of plastic currency ?
a) Due to varied geographical locations and climatic conditions.
b) These cities have major complain of counterfeiting.
c) Security and intelligence agencies are working only in these cities to thwart the illegal activities
related to fake Indian currency notes.
d) All of the above
3. According to the passage, which of the following can be said about the plastic currency ?
a) Such notes incorporate many security features not available to paper banknotes.
b) Plastic currency is made from a polymer.
c) It is last significantly longer than paper notes.
d) None of these
4) Choose the word which is MOST OPPOSITE in meaning of the word printed in bold as used in the
passage-
i) Counterfeiting
a) Duplication b) Original c) Reproduction d) Facsimile
ii). Combat
a) Harmony b) Engagement c) Struggle d) Skirmish
5) Choose the word which is MOST SIMILAR in meaning of the word printed in bold as used in the
passage-
i) Retracted
a ) Cancelled b) Mended c) Forged d) Emphasized
ii) Novelty
a) Standard b) Weird c) Horrific d) Newness

27. "Did you see that?" Joe said to his friend Bill. "You're a great shooter!"
Bill caught the basketball and bounced it before throwing it again. The ball flew into the net.
"Bill, you never miss!" Joe said admiringly. "Unless I'm in a real game," Bill complained. "Then I miss all
the time."

109
Joe knew that Bill was right. Bill performed much better when he was having fun with Joe in the school
yard than he did when he was playing for the school team in front of a large crowd.
"Maybe you just need to practice more," Joe suggested. "But I practice all the time with you!" Bill
objected. He shook his head. "I just can't play well when people are watching me." "You play well when
I'm watching," Joe pointed out. "That's because I've known you since we were five years old," Bill said
with a smile. "I'm just not comfortable playing when other people are around." Joe nodded and
understood, but he also had an idea.
The next day Joe and Bill met in the school yard again to practice. After a few minutes, Joe excused
himself. "Practice without me," Joe said to his friend. "I'll be back in a minute."
Joe hurried through the school building, gathering together whomever he could findtwo students, a
math teacher, two secretaries, and a janitor. When Joe explained why he needed them, everyone was
happy to help. Joe reminded the group to stay quiet as they all went toward the school's basketball
court. As Joe had hoped, Bill was still practicing basketball. He made five baskets in a row without
noticing the silent people standing behind him.
"Hey, Bill!" Joe called out finally. Bill turned. A look of surprise came over his face. "I just wanted to
show you that you could play well with people watching you," Joe said. "Now you'll have nothing to
worry about for the next game!"

1) What would be the best title for the story?


a) Joe Joins the Team b) Practice Makes Perfect c) Bill Wins the Big Game d) Bill's Basketball Problem
2) The word performed is closest in meaning to _______.
a) acted b) played c) moved d) changed
3) Why does Bill play well when Joe is watching him?
a) He is comfortable with Joe. b) Joe tells him how to play better.
c) He does not know that Joe is there. d) He wants to prove to Joe that he is a good player.
4) Why does Joe decide to gather a group of people?
a) Because he wants more players for his team
b) Because he wants to help Bill feels less nervous
c) Because he wants to show them his talent
d) Because he wants more people to see the next game
5) At the end of the story, all of the following people watch Bill practice EXCEPT _______.
a) Joe b) a janitor c) a math teacher d) the basketball coach

28. Fans, for the past two weeks you have been reading about a bad break I got. Yet today I consider
myself the luckiest man on the face of the earth. I have been in ballparks for seventeen years and have
never received anything but kindness and encouragement from you fans.
Look at these grand men. Which of you wouldnt consider it the highlight of his career to associate with
them for even one day? Sure, Im lucky. Who wouldnt consider it an honour to have known Jacob
Ruppert also the builder of baseballs greatest empire, Ed Barrow to have spent the next nine years
with that wonderful little fellow Miller Huggins then to have spent the next nine years with that
outstanding leader, that smart student of psychology the best manager in baseball today, Joe
McCarthy!
Sure, Im lucky. When the New York Giants, a team you would give your right arm to beat, and vice
versa, sends you a gift, thats something! When everybody, down to the groundskeepers and those boys
in white coats remember you with trophies, thats something.

110
When you have a wonderful mother-in-law who takes sides with you in squabbles against her own
daughter, thats something. When you have a father and mother who work all their lives so that you can
have an education and build your body, its a blessing! When you have a wife who has been a tower of
strength and shown more courage than you dreamed existed, thats the finest I know.
So I close in saying that I might have had a tough break but I have an awful lot to live for!
By Lou Gehrig, Farewell to Baseball Address
1) Lou has been in _____ for seventeen years
a) Limelight b)Grounds c)Fame d)All of the above
2) Why does Lou consider him lucky?
a) He had been in the field for a long time and gained experience
b) He had got the privilege to meet renowned players
c) He had been loved by his fans
d) He had been blessed with everything best in life from career, sport to family
3) The phrase give your right arm means
a) Encourage b) Entertain c) Thank d)All of the above
4) The family members of Lou who were supportive were
a) Wife and in-law b) Parents and father-in-law c)Parents, Wife and in-law d) Parents-in-law
5) The mood of the speech is
a) Mirthful b) Casual c) Blithe d) Thankful

29.All the worlds a stage,


And all the men and women merely players;
They have their exits and their entrances;
And one man in his time plays many parts,
His acts being seven ages. At first the infant,
Mewling and puking in the nurses arms;
And then the whining school-boy, with his satchel
And shining morning face, creeping like snail
Unwillingly to school. And then the lover,
Sighing like furnace, with a woeful ballad
Made to his mistress eyebrow. Then a soldier,
Full of strange oaths, and bearded like the pard,
Jealous in honour, sudden and quick in quarrel,
Seeking the bubble reputation
Even in the cannons mouth. And then the justice,
In fair round belly with good capon lind,
With eyes severe and beard of formal cut,
Full of wise saws and modern instances;
And so he plays his part. The sixth age shifts
Into the lean and slipperdpantaloon,
With spectacles on nose and pouch on side;
His youthful hose, well savd, a world too wide
For his shrunk shank; and his big manly voice,
111
Turning again toward childish treble, pipes
And whistles in his sound. Last scene of all,
That ends this strange eventful history,
Is second childishness and mere oblivion;
Sans teeth, sans eyes, sans taste, sans everything. By William Shakespeare

1) The seventh stage sans everything and oblivion mean


a) Empowered and thoughtful b) Enriched and cheerful
c) Emptied and forgetful d) Enticed and painful
2) What does woeful ballad mean
a) Painful ode b) Happy poem c) Loving note d) Secret note
3) Choose the right sentence
a) In the sixth stage of life man tends to grow plump
b) A soldier has a face like a lion and is very diplomatic
c) The judge will be quick-witted, have abundant experience and lives in tandem with reality
d) The lover will burn with the desire to meet his lady love
4) Seeking the bubble reputation means
a) Getting a magnificent status in society b) Being recognized by all
c) Gaining a short lived recognition d) Gaining a long standing recognition
5) What is the tone of the bard?
a) Earnest b)Blithe c) Placid d) Nostalgic

30. Elizabeth was brooding in her room. She had sought asylum there since spurious gossip about her
began circulating at Seagrove Academy last week. Not that Elizabeth had ever been considered a social
butterfly. She preferred to live vicariously through the stories of her more brazen friends: late night
partying, fraternizing with boys, childish pranks. Still, she had taken to being more by herself than usual
since the allegations surfaced. She was up for consideration for the highly coveted Blauvelt Award, a
scholarship recognizing academic integrity and promise. A student had given headmaster Billings an
anonymous tip that Elizabeth had cheated on several tests this year.

The accusations were laughable. Elizabeth had long been an ace student at Seagrove. She lacked a
natural intelligence this was true. However, she compensated for this deficit through diligence and
perseverance; she was very thorough in her studies and exhibited an almost relentless determination.

Still, the accusations had given the recommendation committee pause. On Friday Elizabeth had been
called to Mr. Billings grand office, where she was asked copious questions about her recent exams. The
experience was quite traumatic. Seagrove was an elite school. Most of its students came from privileged
backgrounds. This was not the case for Elizabeth. Her family had little money. She attended Seagrove on
a full scholarship. The Blauvelt Award would help her family pay for college. So, it was with the same
diligence which she applied to her studies that Elizabeth planned to unmask her accuser. She opened
the school directory on her bed and began combing through the names. Seagrove was such a small and
insular community. Twenty-one kids would be in her graduating class. Elizabeth knew it was inevitable
that the person spreading rumors about her would come to light. It was just a matter of time.

1) Which is the best synonym for spurious?


112
a) ashamed b) horrible c) general d) untrue
2) Which is the best antonym for inevitable?
a) Unforeseeable b) Unavoidable c) Certain d) Anticipated
3) What does the reader learn about the student who accused Elizabeth of cheating?
I. The student is a former friend of Elizabeth.
II. The student does not want anyone to know who he or she is.
III. The student is either lying or mistaken.
a) I only b) I and II c) II and III d) I, II, and III
4) Elizabeth _______________________
a) was naturally clever and hardworking b) had innate talent
c) was determined and meticulous d) was creepy and sneaky
5) Seagrove Academy was a
a) school for gifted students b) school with many wealthy students
c) school where many kids lied and cheated d) large school where it was hard to get to know people

113
COVER LETTER
The purpose of a cover letter, which should always accompany your resume, is to introduce yourself and
compel an employer to learn more about you through your resume.

PERSONALIZE

The statement To Whom it May Concern is concerning! Always address your letter to a specific
person. If you do not know who should receive your letter, research the organizations Web site or call
the main number and ask for the appropriate persons name and title.

RESEARCH

In addition to determining the recipients name and title, research the organization so that you can
convey an appreciation for its mission and an understanding of the duties and qualifications of the
position for which you are applying.

CONVINCE

Articulate how your skills and experiences uniquely qualify you for the position and demonstrate
why you are a good fit for the organization. Be clear and concise and limit your cover letter to
one page. Additionally, maintain a professional tone while providing insight into your
personality so that your letter leaves an employer with a sense of wanting to learn more about
you.

PROOFREAD

A poorly written or error-laden letter is a surefire way to end your candidacy. If you want an employer
to spend additional time reviewing your resume, you must make time to proofread your letter and
ensure that your grammar and spelling are perfect.

114
COVER LETTER FORMAT

Your street address


City, state, zip code
Your cell phone number
Your e-mail address

Date

Ms./Mr./Dr. recipients first and last name


Title
Organization name
Street address
City, state, zip code

Dear Ms./Mr./Dr. Last Name:

Paragraph 1: State why you are writing, how you learned of the organization or position, and basic
information about yourself. If you are writing at the suggestion of someone who knows the recipient,
say so.

Paragraph 2: Demonstrate your knowledge and interest in the organization, and use specific examples
to show how your background and skills qualify you for the position.

Paragraph 3: This paragraph is optional and is recommended for elaborating on particularly relevant or
impressive details included in your resume.

Paragraph 4: Indicate that your resume and other supporting materials are enclosed. Reiterate your
interest in the position and your desire to meet for an interview. State your plans to follow up via e-mail
or phone within a certain period of time and invite the employer to contact you to request additional
information. Lastly, thank the employer for his/her consideration.

Sincerely,

Your signature (on a hard copy letter)

Your name (typed)

Enclosure(s)

115
A cover letter is job specific: This letter is for an Analyst position and the Job Requirements include:
research, quantitative and communication skills.

This letter uses a thesis statement (underlined) to connect skills to the position. The thesis statement
notes three skills listed in the posting and each following paragraph uses examples to describe the skill.

Mason Hall
515 E. Jefferson
Ann Arbor, MI 48109

September 15, 20xx

Seymour Careers
ABC Finance
515 XYZ Drive
New York, New York 60601

Dear Mr. Careers


I am applying for the analyst position listed in the University Career Centers Handshake platform. As
someone with a unique academic background, research experience and strong communication skills I
am eager to apply these experiences in the financial industry.

As a dual major in economics and physics I have developed an interdisciplinary approach to research.
Economics has taught me to examine systems and the problems that arise within these systems while at
the same time considering the people and their roles as decision makers. Through the study of physics I
have gained an appreciation for researching topics by working with their smaller components.

My academic training was applied during internships with JPMorgan Chase and Chrysler. My hands- on
projects included creating a long-term economic model, researching financial statements to develop
exposure profiles and writing weekly interest rate updates. Each project was an opportunity to use my
research and quantitative skills and gain experience in the financial industry.

Working with clients from various backgrounds and needs I have learned the importance of effective
communication. In my roles as peer advisor and camp counselor I assess clients needs and determine
the appropriate resources. As an actor/dancer I have effectively worked as part of a team, adapting to
different work styles and skill sets, to create an effective message for the audience. I hope to bring these
successful communication skills to ABC Finance.

I look forward to talking with you about the analyst position and will contact you within the next two
weeks to discuss appointment options. In the meantime, please feel free to contact me by phone or
email.

Sincerely
Mason Hall

116
EXERCISE

Use this worksheet to gather the necessary information to create a cover letter. Once you
have answered the questions, put the information in letter format.

Preparation

1. Based on the job announcement key words and what you know about the job, what is the
company looking for in a candidate? (What are they buying? What problem are they trying to
solve? What experience, results, skills, approach, and abilities do they need?)

2. To whom are you sending the letter?

Name:

Title:

Company, Address:

Opening Paragraph

1. What is the exact title of the job?

2. How did you discover the job was available? (e.g., job board, internet, placement center,
professional meeting, referral, person in hiring firm, career fair, recruiter, etc.)

117
3. Why are you interested in this particular job and company? (e.g., type of work, location,
reputation, opportunities, etc.)

Middle Paragraph

1. What do you have that matches those needs? (Experience, education, results, skills, approach,
abilities)

Matching cont.

2. Determine the three to five areas that are the highest priority to the company.

Write Cover letters for the following:

1. The Reliance Group is hiring people to work in their office as System engineers, Team leaders,
Project Managers,. Send letters of application to Reliance Group, Human Relations Director,
Subhash Agarwal, 1245 South Street, Level 14, Bangalore UB City, 560025.This help wanted ad
was found in The Times of India.

2. True Blue Insurance Company is hiring people to work in their office. Customer service, telephone
skills, filing, and computer skills are needed. In addition, employees will be handling confidential
information and must be trustworthy. This is an entry-level position.Send letters of application to
True Blue Insurance Company, Human Relations Director, Rob Showers, 456 Main Street, Coral
Springs, FL 33065. This help wanted ad was found online at Career Builder.com.

3. Amazon India is hiring people to work in their office, vacancies various positions are open currently.
Customer service, telephone skills, filing, and computer skills are needed. Send letters of application
to Amazon India Company, Human Relations Director, Ajay Mathews, Brigade Gateway, 8th Floor,
26/1, Dr.Rajkumar Road, Malleshwaram West, Bengaluru, Karnataka 560055

118
PARA JUMBLES
What is a para-jumble?

A set of jumbled sentences that you need to unscramble

Paragraph+ Jumbled= Para Jumble

Thus, Para-jumbles are nothing else but another method to test your Comprehension skills.

Example

1. He was looking forward to opening up the presents in the solitude of his room.

2. Sanjays birthday was celebrated with a big bash.

3. But the guests insisted he open them up right there.

4. Several people arrived at his home bearing gifts, both big and small.

2-> 4-> 1-> 3

Types

Fixed

Either one or both Opening and closing lines given

Movable

All the lines are jumbled need to identify theme and opening line

Example Fixed

1. Opening statement

A. .

B. .

C. .

D. .

5. Closing statement

. Opening statement

A. .
119
B. .

C. .

D. .

Clues

Generic to specific

1) Noun -> Pronoun

2) Classification -> Definition

3) List of Items -> Addressing each item on the list

4) Expansion -> Abbreviation

Cause and effect

1) Cause precedes effect

2) Idea precedes elaboration

Chronology to be preserved

Tips to solve

Identify subject

Identify topic sentence

Form pairs within the given sets based on CLUES

Eliminate options based on the pairs Proactive solving

Understand the central theme

1. The best way to solve Para jumbles is by developing high reading speed and by scanning all the
options available.

2. Try getting the feel of what the passage is talking about.

3. This will keep you in the right direction.

Track the chronology of events

1. In a few questions, the events mentioned in the paragraph can be arranged in a timeline of
occurrence
120
2. It becomes easier to logically construct a paragraph using the sequence.

3. The events are usually mentioned in ascending or descending order of their occurrence.

Review the options

1. Now, suppose you have 4 sentences A B C D. Based on chronological order, you know that A and
B will appear in the order BA. Now scan the available options and match:

(a) DABC (b) ACDB (c)CBAD (d) DBAC.

2. Now you know that the correct answer is either (c) or (d). Choose the best option out of these
two.

Look for articles

1. Definite article - the

2. Indefinite article - a and an

3. The is generally used when some person/place or event is being talked about for the second or
3rd time.

4. First time introductions are usually made with the help of a/an. So, the sentence containing
'the' is most likely to come after the sentence containing a/an.

Look for pronouns

1. Pronouns like he, she, they, it, them, their, him, her, can be easily spotted in para-jumbled
sentences.

2. Such pronouns are always used when the person being talked about has already been
introduced.

So sentence with pronoun is most likely NOT the first sentence.

Look for transition words

1. When you see a transition word in a sentence, be 90% sure that this sentence is not the first
sentence.

2. Transition words are used in connecting a particular idea to the next.

3. Example: Also, again, as well as, after all, for example, for instance, in short, likewise otherwise,
subsequently, hence, simultaneously

121
Look for abbreviations

1. The sentence containing the full form will come before the sentence containing the abbreviation

2. General information precedes specific information

3. Introduction of idea/ entity precedes its description

4. The conclusion comes in the end

How to link or form pairs

1. Pronouns

2. Acronyms/ Abbreviations/ Use of last names

3. Time sequence

4. Hypothesis and Example (Similar to idea and elaboration)

5. Articles

6. Topic sentence and conclusion

7. Indicators of cause and effect / Conjunctions

8. Support words and contrast words

Pronouns and Acronym

1. In terms of the gap between worth and rewards, translators come somewhere near nurses and street-
cleaners.

A. Hofman feels passionately about his work, and this is clear from his writings.

B. But thanklessness and impossibility do not daunt him.

C. He acknowledges too in fact he returns to the point often that best translators of poetry
always fail at some level.

D. Michael Hofman, a poet and translator, accepts this sorry fact without approval or complaint.

Answer deduced based on

Full name preceding last name/ first name

Proper noun preceding pronoun He in this case

Conjunction for connecting statements her But is used for contradicting statement C
122
DCBA is the correct order

Time sequence

A. Then two astronomersthe German, Johannes Kepler, and the Italian, Galileo Galilei-started
publicly to support the Copernican theory, despite the fact that the orbits it predicted did not
quite match the ones observed.

B. His idea was that the sun was stationary at the centre and that the earth and the planets move
in circular orbits around the sun.

C. A simple model was proposed in 1514 by a Polish priest, Nicholas Copernicus.

D. Nearly a century passed before this idea was taken seriously.

CBDA is the correct order

Hypothesis and Example

A. The potential exchanges between the officials of IBBF and the Maharashtra Body-Building
Association has all the trappings of a drama we are accustomed to.

B. In the case of sports persons, there is room for some sympathy, but the apathy of the
administrators, which has even led to sanctions from international bodies, is unpardonable.

C. A case in the point is the hefty penalty of US $10,000 slapped on the Indian Body-Building
Federation for not fulfilling its commitment for holding the Asian Championships in Mumbai in
October.

D. It is a matter of deep regret and concern that the sports administrators often cause more harm
to the image of the country than sportsmen and sportswomen do through their dismal
performances.

CBDA is the correct order

Articles

Articles can be divided into two categories -

1. Definite (the) 2. Indefinite (a and an).

When the author uses 'a / an' he/she wants to make a general statement - wants to introduce the
noun followed by a/an for the first time but when he/she uses 'the' to refer back to some previously
discussed noun.

123
It means having 'the' is very unlikely in the opening sentence.

If 'a/an' and 'the' both are used for the same noun then the sentence containing 'the' will come after the
sentence containing a/an.

Transition words / Indicators

Cause and Support Contrast


effect words words

Accordingly Furthermore Nevertheless

in order to Additionally Although

because Also Nonetheless

so...that And But

consequently Too Surprisingly

therefore as well Despite

given besides on the


contrary
thus indeed
even though
hence likewise
Ironically
when...then moreover
rather than
if...then
In contrast

A. Alexander Bain, Scottish clockmaker, patented the electric clock.

B. The next development in accuracy occurred after 1656 with the invention of the pendulum
clock.

C. Clocks have played an important role in man's history.

D. Spring-driven clocks appeared during the 15th century, although they are often erroneously
credited to Nuremberg watchmaker Peter Henlen around 1511.

CDBA is the correct order


124
EXERCISE

Directions for questions 1 to 10 : Sentences given in each question, when properly sequenced form a
coherent paragraph. Each sentence is labeled with a letter. Choose the most logical order of sentences
from amongst the four choices given to construct a paragraph.
1.
A. Many centre around practical needs getting meat out of fire, speed, using whatever is around.

B. There are also many superstitions attached to them: dropping chopsticks is bad luck, sticking them
upright in your rice is taboo because of the imagery of incense sticks at funeral altars.

C. Among the favourite customs are using them to fish cooked bits of meat and vegetables from boiling
broth while eating "hot pot" with friends, and serving choice pieces to show affection or respect.

D. Then there is the long list of chopsticks don'ts: don't point with them, don't spear food with them,
don't use them to tap your bowl; only beggars do that.

E. There are many stories of the provenance of chopsticks, which in some form have been entrenched in
Chinese history for thousands of years.

1) ECDBA 2) AEBCD 3) EABDC 4) CDBAD

2.
A. However, Owen Paterson, the environment secretary, has signaled he is opposed to a ban and
appears to support the position of the insecticide manufacturers and farming lobby who argue that
banning such products would harm food production.

B. A recent poll found that 71% of Britons would support such a ban.

C. The move would be warmly welcomed by environmentalists who have long argued that "neonics"
should be banned.

D. European officials are set to vote on a proposal that would see a group of insecticides known as
neonicotinoids, which have been implicated in the decline of bees, largely outlawed across the
continent.

E. The debate raises the wider question of how valuable bees, and other pollinators, are to our
agricultural economy.

1) DCBAE 2) BCADE 3) DECBA 4) EDBCA

3.
A. Cleaner nations will become richer and their economies grow faster than dirty nations.

B. If Africa were to burn its own coal reserves, the resultant carbon emissions would cause trillions of
dollars of damage to the rest of the world.

125
C. A global carbon market will create a new global system of economic values

D. But if the developed world can't clean up the globe on its own, it can create market conditions that
make reduction in carbon emissions an economic priority for every nation

E. Of course, the US and all developed nations for that matter can't solve the emissions problem
alone.

1) BEBCA 2) CAEBD 3) CBDEA 4) EBDCA

4.
A. These children may look normal but their brain development and immune systems most certainly are
not.

B. The media focus on children who are desperately thin and obviously wasting away means that chronic
under nutrition just as deadly can be overlooked.

C. In the same regions, about 7%-15% of children suffer from wasting.

D. Their stunted height is a grisly marker of multiple deprivations regarding food intake, care and play,
clean water, good sanitation and health care.

E. Approximately 40% of all children under five in south Asia and sub-Saharan Africa are short for their
age.

1) E CBAD 2) BAEDC 3) EDBCA 4) ECADB

5.
A. Instead, the new machines test for viral load, to verify if the virus is in the blood.

B. So, it is not possible to confirm whether the baby is positive.

C. Adults are normally diagnosed on the basis of antibodies, but when babies are born they still have
those of their mother.

D. But accurately testing children remains a challenge in much of the country.

E. About 400 centres across Mozambique now have printers that can quickly receive test results by GPRS
.

1) EDC BA 2) AEBCD 3) EABDC 4) CDBAD

6.
A. The number of development studies courses offered by colleges and universities has grown over the
last 20 years.

B. What are you hoping to get out of your course?


126
C. Has your interest been sparked by other studies, travel, or family connections?

D. As the new academic year begins for some people this month, we would like to hear what is
motivating you to study development.

E. The content of those courses has also changed to reflect new interests and trends in the sector, with
topics covering a range of subjects, from economics and politics, to the environment, gender and
anthropology.

1) BDAEC 2) DCBAE 3)DAEBC 4) AEDCB

7.
A. We have constructed it from unfortunate habits of thought about how to handle spiraling public debt.

B. With much of the global economy apparently trapped in a long and painful austerity-induced slump, it
is time to admit that the trap is entirely of our own making.

C. People developed these habits on the basis of the experiences of their families and friends: when in
debt trouble, one must cut spending and pass through a period of austerity until the burden (debt
relative to income) is reduced.

D. It seems like common sense even moral virtue to respond this way.

E. That means no meals out for a while, no new cars and no new clothes.

1) DABCE 2) BACDE 3) BACED 4) DEBAC

8.
A. From what we eat to how much energy we consume: everything is trackable, not least because our
gadgets come equipped with clever sensors.

B. Take the recent obsession with self-tracking.

C. Smart technologies are not just disruptive; they can also preserve the status quo. Revolutionary in
theory, they are often reactionary in practice.

D. But it wont take long for governments to start exploring self-tracking as a solution to problems
that could, and probably should, be tackled differently.

E. Right now, most of such self-tracking efforts come from the grass-roots enthusiasts.

1) ACBED 2)CBAED 3) BACED 4) EABCD

9.
A. The newspaper man, the flower seller, the milkman, the sweeper, they are all privy to the chatting
and sharing of news that the elderly find comforting.
127
B. When we remove them from their homes, it is not only the family that they are being removed from,
but all those other people that they see every day and whose joys and woes become their own.

C. In our system of living, the elderly at home have a circle of acquaintances, and friends.

D. The loneliness of the discarded elderly is manifold.

E. When we forcibly remove people from surroundings they have been part of for decades we put an
un-mendable tear in the fabric of their lives.

1) DECA B 2) ABDCE 3)ECADB 4) DBAED

10.

A. Some of the worst cancers arent detected by screening.

B. The only way to be sure is to look at the results of randomized trials comparing cancer deaths in
screened and unscreened people.

C. So how can we be confident that getting a screening test regularly is a good idea?

D. Even when screening works in such trials, the size of the benefit observed is surprisingly low:
Generally, regular screening reduces fatalities from various cancers between 15 percent and 25 percent.

E. They appear suddenly, between regular screenings, and are difficult to treat because they are so
aggressive.

1) DBAEC 2) AECBD 3) ADBEC 4)ACBED

Directions for questions 11-15 : Arrange sentences A, B ,C and D to form a logical sequence between 1
and 6 and choose the right answer option:

11.
1. The ears are fragile instruments.

A. These impulses travel via the auditory nerve to the brain, where they are interpreted as, say, words,
music or an approaching vehicle.

B. When sound waves enter the ear, they cause the eardrum to vibrate.

C. These in turn stimulate auditory nerve fibers, each attuned to a different frequency.

D. The vibrations are transmitted to the cochlea, in the inner ear, where fluid carries them to neatly
organized rows of hair cells.

6. Damage to this delicate apparatus results from both volume and length of exposure to sound.
128
1) BCAD 2)DABC 3) BDCA 4) BDAC

12.

1. Scientists used to think vagal tone was largely stable, like your height in adulthood.

A. To appreciate why this matters, heres a quick anatomy lesson.

B. Subtle variations in your heart rate reveal the strength of this brain-heart connection, and as such,
heart-rate variability provides an index of your vagal tone.

C. Your brain is tied to your heart by your vagus nerve.

D. Our data show that this part of you is plastic, too, and altered by your social habits.

6. By and large, the higher your vagal tone the better.

1) CDAB 2) BCAD 3) ACBD 4) DACB

13)

1. In the long run, national recognition of same-sex marriage is inevitable.

A. It is only a matter of time before all state laws reflect that view.

B . Prudence counsels that marriage equality should be allowed to continue gaining support in the
states, and that a federal resolution should be left for another day.

C . Same-sex marriage rights, at first imposed by courts, have now been recognized by state legislatures
and prevailed in all four states where they were on the ballot in last years election.

D . Young people overwhelmingly support it, and public opinion has shifted on this issue faster than on
almost any other social issue in history.

6. What is more, the courts doctrine dictates just this deferral.

1) CDAB 2) ACBD 3) BACD 4) DCBA

14)

1. A charge on private vehicles in selected areas and at particular times of day would be a reasonable
response to externalities they create.

A. Choosing to drive ones own car or other vehicle into a city centre puts pressure on limited road
space, contributes to pollution and global warming, and results in reduced mobility for all.

129
B. The cumulative time spent in traffic is a major economic loss.

C. Congestion charging schemes therefore levy a premium for the privilege of using a personal car, and
the funds thus collected should be ploughed back exclusively into public transport options.

D. This makes them robust, affordable and sustainable.

6. A number of technologies are available to implement a congestion charging system. The challenge is
to pick one that reduces transaction costs, and is sealed against revenue leakage.

1) ACBD 2) CABD 3) BCAD 4) ABCD

15)

1. Water is a renewable natural resource and public good.

A. However, most rivers, ponds, lakes and aquifers are common property.

B. Hence, excluding others from using water is not possible and the results are competition, over
extraction and conflict.

C. But the ownership right on land bestows a private character on water.

D. Therefore, water rights are not clearly defined and the right to using the resources is not protected.

6. However, cooperation has a greater role in achieving social harmony in water allocation and
increasing human welfare.

1) DACB 2) CADB 3)ACDB 4) BCAD

130
CRITICAL REASONING
Critical reasoning questions test your ability to analyze logical arguments.

Tests skills involved in making arguments, evaluating arguments, and formulating or evaluating a plan of
action.

The 8 Types of CR Questions

1) Weaken the Argument

2) Strengthen the Argument

3) Find the Assumption

4) Draw a Conclusion/What Can Be Inferred?

5) What is the Structure of the Argument?

6) Complete the Argument

7) Paradox Questions

8) Evaluate the Conclusion

Important concepts

Argument

unit of reasoning that attempts to prove that a certain idea is true by citing other ideas as
evidence. It has 2 parts Premise and Conclusion

Premise

what the author is trying to convince you of

Conclusion

the evidence used to support the conclusion

Analyze the given statement

All men are mortal. Socrates is a man. Therefore, Socrates is mortal.

Premise: All men are mortal

131
Premise: Socrates is a man

Conclusion: Socrates is mortal

Deductive -> Definite conclusion

Analyze the given statement

For the past 3 days, Gary has arrived late for work. This morning, while Gary was driving to work, his car
got a flat tire. Therefore, Gary will be late for work today.

Premise: Gary late for work last 3 days

Premise: Flat tire today

Conclusion: Gary will be late today

Inductive -> Indefinite conclusion

Isolating the Nerve Center of an Argument

Premise: the starting point of deductions; often, agreement to this is assumed.

Conclusion: what the author wants you to believe by the end of the argument

Assumption: the unstated link between premise and conclusion.

Although unstated, the assumption is the nerve center of the argument, the linchpin holding the whole
thing together.

Strengthening and Weakening Arguments

Which would most strengthen the argument?

The most powerful way to strengthen an argument is to validate its assumption.

Which of the following would most weaken the argument?

The most devastating attack on an argument is the denial of its assumption without the assumption,
the link between the premise and the conclusion is severed.

Argument Strength

132
Argument Strength
For the past 3 days, Gary has arrived late for
work. This morning, while Gary was driving to
STRENGTHEN work, his car got a flat tire. Therefore, Gary will
be late for work today.
For the past 53 days, Gary has arrived late for
work. This morning, while Gary was driving to
work, his car got a flat tire. Therefore, Gary will
be late for work today.
STRENGTHEN For the past 53 days, Gary has arrived late for
work. This morning, while Gary was driving to
work, his car got 4 flat tires, and his engine
exploded. Therefore, Gary will be late for work
today.
SCHOOL OF PROFESSIONAL ENHANCEMENT

Argument Strength
For the past 3 days, Gary has arrived late for
work. This morning, while Gary was driving to
WEAKEN work, his car got a flat tire. Therefore, Gary will
be late for work today.

For the past 3 days, Gary has arrived late for


work. This morning, while Gary was driving to
work, his car got a flat tire. Garys house is
one block from work. Therefore, Gary will be
late for work today.

SCHOOL OF PROFESSIONAL ENHANCEMENT

133
Assumption

Unstated premise(s) necessary to reach conclusion.

Personal experience not to be taken into account.

Assumptions:

Garys history of lateness will be repeated today

The time needed to change tire will exceed the time before work starts

Gary has no other means by which to arrive at work on time

Calling Gary lazy is not an assumption

Think Broadly

Assumptions are most often general statements, not specific statements.

If my premise is Fred has quality A, and my conclusion is Therefore, Fred has quality
B, then the assumption is not going to involve Fred at all.

The assumption would be something like most/all folks who have quality A also have
quality B.

In trying to identify the assumption, it can helpful to remember that you can omit any
specific people/places/items mentioned.

Precisely

Premise (s)+Assumption (s) = Conclusion

Analyze

Every hockey fan I know is nice. I do not know Judy, but since she is wearing a hockey jersey,
she must be nice.

Analysis

Premise

Every hockey fan I know is nice.

I dont know Judy.

Judy is wearing a hockey jersey.

134
Assumption

Since Judy is wearing hockey jersey, she is a fan

Hockey fans I know are representatives

Conclusion

Judy is nice

Tips for Identifying Conclusions and Premises

Watch for trigger words that indicate a conclusion

therefore, thus, hence, so, implies, indicates, consequently, as a result, clearly, accordingly, infer

Watch for trigger words that indicate a premise

since, because, for, due to, evidence, on the basis of, given that

Beware of common argument structures

1. Premise, premise, . . . , conclusion

2. Conclusion, premise, premise . . .

3. Conclusion in the question stem

Argument types

Cause and Effect

Premise: Event X occurs

Premise: Event Y occurs

Assumption: X is the only possible cause of Y

Conclusion: X causes Y

Example:

A recent study reveals that the rate of obesity is higher among senior citizens who watch more than 8
hours of television per day than among senior citizens who watch fewer than 8 hours of television per

135
day. Therefore, obesity among senior citizens is caused by watching more than 8 hours of television per
day.

Statistical

Premise: Information from sample

Assumption: Sample represents entire population

Conclusion: Something about entire population

Example:

In a recent survey, participants at a Republicans-only dance competitions were given a questionnaire.


Most of the respondents indicated that they enjoyed singing. Therefore, it can be concluded that most
Republicans are outgoing people.

Analogy

Premise: X is similar to Y in aspect 1

Premise: X is similar to Y in aspect 2

Premise: X is similar to Y in aspect 3

Assumption: X is similar to Y in all aspects

Conclusion: X is similar to Y in aspect n

Example:

Country X is a democratic, tropical country with a population of 5 million, and Country Y is a democratic,
tropical country with a population of 5 million. Since Country X is experiencing widespread crop failures,
Country Y must be experiencing widespread crop failures as well.

Lets analyze

Today is Tuesday & yesterday was Monday. Therefore, tomorrow will be Wednesday.

This speakers conclusion depends on which of the following assumption?

a. Wednesday is the day that precedes Thursday

b. Tuesday always follows Monday

c. If, in any given week Tuesday follows Monday, then Wednesday will follow Tuesday.
136
d. Every week consist of 7 day arranged in a particular order.

Approach to solve

Read the argument carefully. Wherever possible, identify premises, assumptions and conclusion.
(Take approx. 30 seconds to understand.)

Take a few extra seconds to make sure you understand the conclusion of the argument.

Read the question prompt to find what you are asked to do (strengthen / weaken / find
assumption etc.) and then think what a correct answer might involve. (Take approx. 30 seconds
to think.)

Now read the answer choices. Select the best answer, this should be easy if you have not rushed
the previous steps. (Take approx. 30 seconds to read and select.)

Assume and infer carefully

Starters

1.Mike says, I am planning to go for a movie this weekend. The assumption from the given statement
is

Mike is always spending his weekend watching movies.

Mike has planned to take a day off and relax this weekend.

Mike is going to enjoy with his friends this weekend.

2.Herald said that he had had a great time at the party. An observation from the given statement is

The party is over

The party is ongoing

The party is going to be held

3.Rita says that she is very sick. The inference from the given statement is

Rita is suffering from diarrhea

Rita is unconscious

Rita is not the speaker

4.Linda is very happy and excited about her French classes. The inference from the given statement is
137
Linda has enrolled herself for French classes

Linda is learning French very easily

Linda has a very good French tutor.

5.Ted has bought a new laptop. It means

Teds new laptop has become old.

Ted is a gadget freak.

Ted owns a laptop.

Try to analyze

If Nilesh graduated from JRD high school after 1998, then he performed community service

The preceding statement is a logical consequence of which of the following statements?

No student who graduated from JRD high school before 1998 performed community service

Only if the student graduated from JRD high school after 1998 could he or she have performed
community service

Some students who graduated from JRD highs school after 1998 performed community service

All students who graduated from JRD highs school after 1998 performed community service

Read

Increases in funding for police patrols often lower the rate of crimes of opportunity such as petty theft
and vandalism by providing visual deterrence in high crime neighborhoods. Levels of funding for police
patrols in some communities are increased when federal matching grants are made available.

Identify the inference

Which of the following can be correctly inferred from the statements above?

(a) Areas with little vandalism can never benefit from visual deterrence.

(b) Communities that do not increase their police patrols are at higher risk for crimes of opportunity late
at night.

(c) Federal matching grants for police patrols lower the rate of crimes of opportunity in some
communities.

(d) Only federal matching grants are necessary to reduce crime in most neighborhoods.
138
Reason

(c) is a summary of the information provided; it is the

logical end of a chain of reasoning started in the stimulus argument.

The sequence of events goes like this :

Increased funding -> Increased visual deterrence -> Lower crime

The last statement could be mapped as follows:

Federal grants -> Increased patrol funds

(c) makes the chain complete by correctly stating that federal grants can lead to lower crime in some
communities. Now the logical chain becomes:

Federal grants -> Increased funding -> Increased visual deterrence -> Lower crime

Read

Traditionally, decision making by doctors that is carefully, deductively reasoned has been considered
preferable to intuitive decision making. However, a recent study found that senior surgeons used
intuition significantly more than did most residents or mid-level doctors. This confirms the alternative
view that intuition is actually more effective than careful, methodical reasoning.

Identify the assumption

The conclusion above is based on which of the following assumptions?

(a) Senior surgeons are more effective at decision making than are mid-level doctors.

(b) Senior surgeons have the ability to use either intuitive reasoning or deductive, methodical reasoning
in making decisions.

(c) The decisions that are made by mid-level and entry-level doctors can be made as easily by using
methodical reasoning as by using intuitive reasoning.

(d) Senior surgeons use intuitive reasoning in making the majority of their decisions.

Reason

The correct answer is (a), which provides a missing link in the authors reasoning by making a connection
from the evidence: that intuition is used more by senior surgeons than other, less-experienced doctors,
and the conclusion: that, therefore, intuition is more effective.

139
None of the other choices helps bridge this gap in the chain of reasoning. Although some of the other
statements may be true, they are not responsive to the question. In fact, they mostly focus on irrelevant
factors such as appropriateness, ease of application, ability, etc.

Read

Three years after the Bhakra Nangal Dam was built, none of the six fish species native to the area was
still reproducing adequately in the river below the dam. Because the dam reduced the average
temperature range of the water from approximately 40 to approximately 10, biologists have
hypothesized that sharp increases in water temperature must be involved in signaling the affected
species to begin their reproduction activities.

Identify the strengthening point

Which of the following statements, if true, would most strengthen the scientists hypothesis?

(a) The native fish species were still able to reproduce in nearby streams where the annual temperature
range remains approximately 40.

(b) Before the dam was built, the river annually overflowed its banks, creating temporary backwaters
that were used as breeding areas for the local fish population.

(c) The lowest temperature ever recorded in the river prior to dam construction was 30; whereas the
lowest recorded river temperature after construction was completed has been 40.

(d) Non-native fish species, introduced after the dam was completed, have begun competing with the
native species for food.

Reason

(a) most strengthens the conclusion that the scientists reached. It does so by showing that there is a
control group. In other words, a similar population, not subjected to the same change as the population
near the dam, did not experience the same type of result.

Here the basic assumption about the conclusion that scientists reached is that because of the reduction
of average temperature range of the water, the reproduction of the native fish species has reduced
drastically.

Read

A drug that is very effective in treating some forms of cancer can, at present, be obtained only from the
bark of the Raynhu, a tree that is quite rare in the wild. It takes the bark of approximately 5,000 trees to
make one pound of the drug. It follows, then, that continued production of the drug must inevitably lead
to the Raynhus extinction.

Identify the weakening point


140
Which of the following, if true, most seriously weakens the above conclusion?

(a) The drug made from Raynhu bark is dispensed to doctors from a central authority.

(b) The drug made from the Raynhu bark is expensive to produce.

(c) The Raynhu generally grows in largely inaccessible places.

(d) The Raynhu can be propagated from cuttings and cultivated by farmers.

Reason

(d) provides an alternate source of the Raynhu bark.

Even though the tree is rare in the wild, the argument is silent on the availability of cultivated trees. The
author of the argument must be assuming that there are no Raynhu trees other than those in the wild,
in order to make the leap from the stated evidence to the conclusion that the Raynhu is headed for
extinction.

The option (d) weakens the assupmtion - there are limited raynhu trees - by saying that there are other
ways as well for the propogation of Raynhu.

Note that the correct choice does not make the conclusion of the argument impossible. In fact, it is
possible that there may be domesticated Raynhu trees and the species could still become extinct.
Answer choice (d) is correct because it makes the conclusion about extinction less likely to be true.

One more type

The management of school M has decided to give free breakfast from next academic year to all the
students in its primary section through its canteen even though they will not get any government grant.

(A) The school will have to admit many poor students who will seek admission for the next academic
year.

(B) The canteen facilities and utensils have to be checked and new purchases to be made to equip it
properly.

(C) Funds will have to be raised to support the scheme for years to come.

(D) All students will get the more nutritious food at free of cost.

(E) This decision will attract many students to get admission at school M

(F) Breakfast will not be nutritious and safe and it can be harmful for health.

Q1Which of the following A, B and C can be an immediate course of action for the management?

141
(a) Only (A)

(b) Only (B)

(c) Only (C)

(d) Both (B) and (C)

(e) None of these

Q2 Which of the following among A, B, E and D may be the reason behind the management taking
such decision?

(a) Only (A)

(b) Only (B)

(c) Both (A) and (E)

(d) Only (E)

(e) Only (D)

Q3 Which of the following A, B, E and F may be an immediate effect if there will be shortcomings in
the proper arrangement of breakfast?

(a) Only (A)

(b) Only (B)

(c) Only (E)

(d) Only (F)

(e) Both (E) and (F)

Exercise

1) Find the statement that must be true according to the given information.

Vincent has a paper route. Each morning, he delivers 37 newspapers to customers in his neighborhood.
It takes Vincent 50 minutes to deliver all the papers. If Vincent is sick or has other plans, his friend

142
Thomas, who lives on the same street, will sometimes deliver the papers for him.
A. Vincent and Thomas live in the same neighborhood.

B. It takes Thomas more than 50 minutes to deliver the papers.

C. It is dark outside when Vincent begins his deliveries.

D. Thomas would like to have his own paper route.

2) Find the statement that must be true according to the given information.
The Pacific yew is an evergreen tree that grows in the Pacific Northwest. The Pacific yew has a fleshy,
poisonous fruit. Recently, taxol, a substance found in the bark of the Pacific yew, was discovered to be a
promising new anticancer drug.
A. Taxol is poisonous when taken by healthy people.

B. Taxol has cured people from various diseases.

C. People should not eat the fruit of the Pacific yew.

D. The Pacific yew was considered worthless until taxol was discovered.

3) Find the statement that must be true according to the given information.
Erin is twelve years old. For three years, she has been asking her parents for a dog. Her parents have
told her that they believe a dog would not be happy in an apartment, but they have given her
permission to have a bird. Erin has not yet decided what kind of bird she would like to have.
A. Erin's parents like birds better than they like dogs.

B. Erin does not like birds.

C. Erin and her parents live in an apartment.

D. Erin and her parents would like to move.

4) Find the statement that must be true according to the given information.
Tim's commute never bothered him because there were always seats available on the train and he was
able to spend his 40 minutes comfortably reading the newspaper or catching up on paperwork. Ever
since the train schedule changed, the train has been extremely crowded, and by the time the doors
143
open at his station, there isn't a seat to be found.
A. Tim would be better off taking the bus to work.

B. Tim's commute is less comfortable since the train schedule changed.

C. Many commuters will complain about the new train schedule.

D. Tim will likely look for a new job closer to home.

5) Find the statement that must be true according to the given information.
When they heard news of the hurricane, Maya and Julian decided to change their vacation plans.
Instead of traveling to the island beach resort, they booked a room at a fancy new spa in the mountains.
Their plans were a bit more expensive, but they'd heard wonderful things about the spa and they were
relieved to find availability on such short notice.
A. Maya and Julian take beach vacations every year.

B. The spa is overpriced.

C. It is usually necessary to book at least six months in advance at the spa.

D. Maya and Julian decided to change their vacation plans because of the hurricane.

6) Find the statement that must be true according to the given information.
Ten new television shows appeared during the month of September. Five of the shows were sitcoms,
three were hour-long dramas, and two were news-magazine shows. By January, only seven of these new
shows were still on the air. Five of the shows that remained were sitcoms.
A. Only one of the news-magazine shows remained on the air.

B. Only one of the hour-long dramas remained on the air.

C. At least one of the shows that was cancelled was an hour-long drama.

D. Television viewers prefer sitcoms over hour-long dramas.

7) Find the statement that must be true according to the given information.
On weekends, Mr. Sanchez spends many hours working in his vegetable and flower gardens. Mrs.
144
Sanchez spends her free time reading and listening to classical music. Both Mr. Sanchez and Mrs.
Sanchez like to cook.
A. Mr. Sanchez enjoys planting and growing vegetables.

B. Mr. Sanchez does not like classical music.

C. Mrs. Sanchez cooks the vegetables that Mr. Sanchez grows.

D. Mrs. Sanchez enjoys reading nineteenth century novels.

8) Find the statement that must be true according to the given information.
Georgia is older than her cousin Marsha. Marsha's brother Bart is older than Georgia. When Marsha and
Bart are visiting with Georgia, all three like to play a game of Monopoly. Marsha wins more often than
Georgia does.
A. When he plays Monopoly with Marsha and Georgia, Bart often loses.

B. Of the three, Georgia is the oldest.

C. Georgia hates to lose at Monopoly.

D. Of the three, Marsha is the youngest.

9)During the past 20 years, computer scientists focused increasingly on starting and running successful
businesses. However, since businesses must be profitable, computer scientists must focus on developing
products that generate profit. Consequently, computer science has lost its creative aspect.

Which of the following assumptions is most necessary in order for the conclusion above to be drawn
from the argument above?

A All computer programs must lack creativity in order to be well received.

B Some computer scientists entirely disregarded creativity and chose instead to pursue profit.

C A program cannot be both creative and profitable.

D Computer scientists are obsessed with the profitability of their work.

E Non-profit institutions use large amounts of software

145
10)The cost of producing radios in Country Q is ten percent less than the cost of producing radios in
Country Y. Even after transportation fees and tariff charges are added, it is still cheaper for a company to
import radios from Country Q to Country Y than to produce radios in Country Y.

The statements above, if true, best support which of the following assertions?

(A) Labor costs in Country Q are ten percent below those in Country Y.

(B) Importing radios from Country Q to Country Y will eliminate ten percent of the manufacturing jobs in
Country Y.

(C) The tariff on a radio imported from Country Q to Country Y is less than ten percent of the cost of
manufacturing the radio in Country Y.

(D) The fee for transporting a radio from Country Q to Country Y is more than ten percent of the cost of
manufacturing the radio in Country Q.

(E) It takes ten percent less time to manufacture a radio in Country Q than it does in Country Y.

In each question below is given a statement followed by two conclusions numbered I and II. You have to
assume everything in the statement to be true, then consider the two conclusions together and decide
which of them logically follows beyond a reasonable doubt from the information given in the statement.

11 Statements: In a one day cricket match, the total runs made by a team were 200. Out of these 160
runs were made by spinners.
Conclusions:
I. I) 80% of the team consists of spinners.
II. Ii) The opening batsmen were spinners.
A. Only conclusion I follows

B. Only conclusion II follows

C. Either I or II follows

D. Neither I nor II follows

E. Both I and II follow


12 In each question below is given a statement followed by two conclusions numbered I and II. You have
to assume everything in the statement to be true, then consider the two conclusions together and
decide which of them logically follows beyond a reasonable doubt from the information given in the
statement.

Statements: The old order changed yielding place to new.


Conclusions:
146
I. I)Change is the law of nature.
II. Ii)Discard old ideas because they are old.
A. Only conclusion I follows

B. Only conclusion II follows

C. Either I or II follows

D. Neither I nor II follows

E. Both I and II follow


13 In each question below is given a statement followed by two conclusions numbered I and II. You have
to assume everything in the statement to be true, then consider the two conclusions together and
decide which of them logically follows beyond a reasonable doubt from the information given in the
statement.

Statements: Government has spoiled many top ranking financial institutions by appointing bureaucrats
as Directors of these institutions.
Conclusions:
I. I)Government should appoint Directors of the financial institutes taking into consideration the
expertise of the person in the area of finance.
II. Ii)The Director of the financial institute should have expertise commensurate with the financial
work carried out by the institute.
A. Only conclusion I follows

B. Only conclusion II follows

C. Either I or II follows

D. Neither I nor II follows

E. Both I and II follow

14 In each question below is given a statement followed by two conclusions numbered I and II. You have
to assume everything in the statement to be true, then consider the two conclusions together and
decide which of them logically follows beyond a reasonable doubt from the information given in the
statement.

Statements: Population increase coupled with depleting resources is going to be the scenario of many
developing countries in days to come.
Conclusions:
I. 1)The population of developing countries will not continue to increase in future.
II. Ii)It will be very difficult for the governments of developing countries to provide its people
147
decent quality of life.
A. Only conclusion I follows

B. Only conclusion II follows

C. Either I or II follows

D. Neither I nor II follows

E. Both I and II follow


15 In each question below is given a statement followed by two conclusions numbered I and II. You have
to assume everything in the statement to be true, then consider the two conclusions together and
decide which of them logically follows beyond a reasonable doubt from the information given in the
statement.

Statements: Prime age school-going children in urban India have now become avid as well as more
regular viewers of television, even in households without a TV. As a result there has been an alarming
decline in the extent of readership of newspapers.
Conclusions:
I) Method of increasing the readership of newspapers should be devised.
ii)A team of experts should be sent to other countries to study the impact of TV. on the readership of
newspapers.
A. Only conclusion I follows

B. Only conclusion II follows

C. Either I or II follows

D. Neither I nor II follows

E. Both I and II follow


16 In each question below is given a statement followed by two conclusions numbered I and II. You have
to assume everything in the statement to be true, then consider the two conclusions together and
decide which of them logically follows beyond a reasonable doubt from the information given in the
statement.

Statements: In Japan, the incidence of stomach cancer is very high, while that of bowel cancer is very
low. But Japanese immigrate to Hawaii, this is reversed - the rate of bowel cancer increases but the rate
of stomach cancer is reduced in the next generation. All this is related to nutrition - the diets of Japanese
in Hawaii are different than those in Japan.
Conclusions:
I. I)The same diet as in Hawaii should be propagated in Japan also.
II. Ii) Bowel cancer is less severe than stomach cancer.

148
A. Only conclusion I follows

B. Only conclusion II follows

C. Either I or II follows

D. Neither I nor II follows

E. Both I and II follow


17 In each question below is given a statement followed by two conclusions numbered I and II. You have
to assume everything in the statement to be true, then consider the two conclusions together and
decide which of them logically follows beyond a reasonable doubt from the information given in the
statement.

Statements: The Government run company had asked its employees to declare their income and assets
but it has been strongly resisted by employees union and no employee is going to declare his income.
Conclusions:
I. I)The employees of this company do not seem to have any additional undisclosed income
besides their salary.
II. Ii)The employees union wants all senior officers to declare their income first.
A. Only conclusion I follows

B. Only conclusion II follows

C. Either I or II follows

D. Neither I nor II follows

E. Both I and II follow


18. The average life expectancy for the United States population as a whole is 73.9 years, but children
born in Hawaii will live an average of 77 years, and those born in Louisiana, 71.7 years. If a newlywed
couple from Louisiana were to begin their family in Hawaii, therefore, their children would be expected
to live longer than would be the case if the family remained in Louisiana.
Which of the following, if true, would most seriously weaken the conclusion drawn in the passage?
A. Insurance company statisticians do not believe that moving to Hawaii will significantly lengthen the
average Louisianan's life.
B. The governor of Louisiana has falsely alleged that statistics for his state are inaccurate.
C. The longevity ascribed to Hawaii's current population is attributable mostly to genetically determined
factors.
D. Thirty percent of all Louisianans can expect to live longer than 77 years.
E. Most of the Hawaiian Islands have levels of air pollution well below the national average for the
United States.

149
19. The average life expectancy for the United States population as a whole is 73.9 years, but children
born in Hawaii will live an average of 77 years, and those born in Louisiana, 71.7 years. If a newlywed
couple from Louisiana were to begin their family in Hawaii, therefore, their children would be expected
to live longer than would be the case if the family remained in Louisiana.
Which of the following statements, if true, would most significantly strengthen the conclusion drawn in
the passage?
A. As population density increases in Hawaii, life expectancy figures for that state are likely to be revised
downward.
B. Environmental factors tending to favor longevity are abundant in Hawaii and less numerous in
Louisiana.
C. Twenty-five percent of all Louisianans who move to Hawaii live longer than 77 years.
D. Over the last decade, average life expectancy has risen at a higher rate for Louisianans than for
Hawaiians.
E. Studies show that the average life expectancy for Hawaiians who move permanently to Louisiana is
roughly equal to that of Hawaiians who remain in Hawaii.

20. Insurance Company X is considering issuing a new policy to cover services required by elderly people
who suffer from diseases that afflict the elderly. Premiums for the policy must be low enough to attract
customers. Therefore, Company X is concerned that the income from the policies would not be
sufficient to pay for the claims that would be made.
Which of the following strategies would be most likely to minimize Company X's losses on the policies?
A. Attracting middle-aged customers unlikely to submit claims for benefits for many years.
B. Insuring only those individuals who did not suffer any serious diseases as children
C. Including a greater number of services in the policy than are included in other policies of lower cost
D. Insuring only those individuals who were rejected by other companies for similar policies
E. Insuring only those individuals who are wealthy enough to pay for the medical services

21. A program instituted in a particular state allows parents to prepay their children's future college
tuition at current rates. The program then pays the tuition annually for the child at any of the state's
public colleges in which the child enrolls. Parents should participate in the program as a means of
decreasing the cost for their children's college education.
Which of the following, if true, is the most appropriate reason for parents NOT to participate in the
program?
A. the parents are unsure about which public college in the state the child will attend.
B. The amount of money accumulated by putting the prepayment funds in an interest-bearing account
today will be greater than the total cost of tuition for any of the public colleges when the child enrolls.
C. The annual cost of tuition at the state's public colleges is expected to increase at a faster rate than the
annual increase in the cost of living.
D. Some of the state's public colleges are contemplating large increases in tuition next year.
E. The prepayment plan would not cover the cost of room and board at any of the state's public colleges.

22. Company Alpha buys free-travel coupons from people who are awarded the coupons by Bravo
Airlines for flying frequently on Bravo airplanes. The coupons are sold to people who pay less for the
coupons than they would pay by purchasing tickets from Bravo. This making of coupons results in lost
revenue for Bravo. To discourage the buying and selling of free-travel coupons, it would be best for
Bravo Airlines to restrict the
A. number of coupons that a person can be awarded in a particular year
150
B. use of the coupons to those who were awarded the coupons and members of their immediate
families
C. days that the coupons can be used to Monday through Friday
D. amount of time that the coupons can be used after they are issued
E. number of routes on which travelers can use the coupons

23. The ice on the front windshield of the car had formed when moisture condensed during the night.
The ice melted quickly after the car was warmed up the next morning because the defrosting vent,
which blows on the front windshield, was turned on full force.
Which of the following, if true, most seriously jeopardizes the validity of the explanation for the speed
with which the ice melted?
A. The side windows had no ice condensation on them
B. Even though no attempt was made to defrost the back window, the ice there melted at the same rate
as did the ice on the front windshield.
C. The speed at which ice on a window melts increases as the temperature of the air blown on the
window increases
D. The warm air from the defrosting vent for the front windshield cools rapidly as it dissipates
throughout the rest of the car.
E. The defrosting vent operates efficiently even when the heater, which blows warm air toward the feet
or faces of the driver and passengers, is on.

24. To prevent some conflicts of interest, Congress could prohibit high-level government officials from
accepting positions as lobbyists for three years after such officials leave government service. One such
official concluded, however, that such a prohibition would be unfortunate because it would prevent
high-level government officials from earning a livelihood for three years. The official's conclusion
logically depends on which of the following assumptions?
A. Laws should not restrict the behavior of former government officials.
B. Lobbyists are typically people who have previously been high-level government officials.
C. Low-level government officials do not often become lobbyists when they leave government service.
D. High-level government officials who leave government service are capable of earning a livelihood only
as lobbyists.
E. High-level government officials who leave government service are currently permitted to act as
lobbyists for only three years.

25. A conservation group in the United States is trying to change the long-standing image of bats as
frightening creatures. The group contends that bats are feared and persecuted solely because they are
shy animals that are active only at night. Which of the following, if true, would cast the most serious
doubt on the accuracy of the group's contention?
A. Bats are steadily losing natural roosting places such as caves and hollow trees and are thus turning to
more developed areas for roosting.
B. Bats are the chief consumers of nocturnal insects and thus can help make their hunting territory more
pleasant for humans.
C. Bats are regarded as frightening creatures not only in the United States but also in Europe, Africa, and
South America.
D. Raccoons and owls are shy and active only at night; yet they are not generally feared and persecuted.

151
E. People know more about the behavior of other greatly feared animal species, such as lions, alligators,
and greatly feared animal species, such as lions, alligators, and snakes, than they do about the behavior
of bats.

26. Meteorite explosions in the Earth's atmosphere as large as the one that destroyed forests in Siberia,
with approximately the force of a twelve-megaton nuclear blast, occur about once a century. The
response of highly automated systems controlled by complex computer programs to unexpected
circumstances is unpredictable.
Which of the following conclusions can most properly be drawn, if the statements above are true, about
a highly automated nuclear-missile defense system controlled by a complex computer program?
A. Within a century after its construction, the system would react inappropriately and might accidentally
start a nuclear war.
B. The system would be destroyed if an explosion of a large meteorite occurred in the Earth's
atmosphere.
C. It would be impossible for the system to distinguish the explosion of a large meteorite from the
explosion of a nuclear weapon.
D. Whether the system would respond inappropriately to the explosion of a large meteorite would
depend on the location of the blast.
E. It is not certain what the system's response to the explosion of a large meteorite would be, if its
designers did not plan for such a contingency.

27. The fewer restrictions there are on the advertising of legal services, the more lawyers there are who
advertise their services, and the lawyers who advertise a specific service usually charge less for that
service than lawyers who do not advertise. Therefore, if the state removes any of its current restrictions,
such as the one against advertisements that do not specify fee arrangements, overall consumer legal
costs will be lower than if the state retains its current restrictions. If the statements in the passage are
true, which of the following must be true?
A. Some lawyers who now advertise will charge more for specific services if they do not have to specify
fee arrangements in the advertisements.
B. More consumers will use legal services if there are fewer restrictions on the advertising of legal
service.
C. If the restriction against advertisements that do not specify fee arrangements is removed, more
lawyers will advertise their services.
D. If more lawyers advertise lower prices for specific services, some lawyers who do not advertise will
also charge less than they currently charge for those services.
E. If the only restrictions on the advertising of legal services were those that apply to every type of
advertising, most lawyers would advertise their services.

28. The fewer restrictions there are on the advertising of legal services, the more lawyers there are who
advertise their services, and the lawyers who advertise a specific service usually charge less for that
service than lawyers who do not advertise. Therefore, if the state removes any of its current restrictions,
such as the one against advertisements that do not specify fee arrangements, overall consumer legal
costs will be lower than if the state retains its current restrictions. Which of the following, if true, would
most seriously weaken the argument concerning overall consumer legal costs?
A. The state has recently removed some other restrictions that had limited the advertising of legal
services.

152
B. The state is unlikely to remove all of the restrictions that apply solely to the advertising of legal
services.
C. Lawyers who do not advertise generally provide legal services of the same quality as those provided
by lawyers who do advertise.
D. Most lawyers who now specify fee arrangements in their advertisements would continue to do so
even if the specification were not required.
E. Most lawyers who advertise specific services do not lower their fees for those services when they
begin to advertise.

29. Defense Department analysts worry that the ability of the United States to wage a prolonged war
would be seriously endangered if the machine-tool manufacturing base shrinks further. Before the
Defense Department publicly connected this security issue with the import quota issue, however, the
machine-tool industry raised the national security issue in its petition for import quotas. Which of the
following, if true, contributes most to an explanation of the machine-tool industry's raising the issue
above regarding national security?
A. When the aircraft industries retooled, they provided a large amount of work for too builders.
B. The Defense Department is only marginally concerned with the effects of foreign competition on the
machine-tool industry.
C. The machine-tool industry encountered difficulty in obtaining governmental protection against
imports on grounds other than defense.
D. A few weapons important for defense consist of parts that do not require extensive machining.
E. Several federal government programs have been designed which will enable domestic machine-tool
manufacturing firms to compete successfully with foreign toolmakers.

30. Opponents of laws that require automobile drivers and passengers to wear seat belts argue that in a
free society people have the right to take risks as long as the people do not harm other as a result of
taking the risks. As a result, they conclude that it should be each person's decision whether or not to
wear a seat belt. Which of the following, if true, most seriously weakens the conclusion drawn above?
A. Many new cars are built with seat belts that automatically fasten when someone sits in the front seat.
B. Automobile insurance rates for all automobile owners are higher because of the need to pay for the
increased injuries or deaths of people not wearing seat belts.
C. Passengers in airplanes are required to wear seat belts during takeoffs and landings.
D. The rate of automobile fatalities in states that do not have mandatory seat belt laws is greater than
the rate of fatalities in states that do have such laws.
E. In automobile accidents, a greater number of passengers who do not wear seat belts are injured than
are passengers who do wear seat belts.

153

S-ar putea să vă placă și